Calculus Sem1 Solutions

You might also like

Download as pdf or txt
Download as pdf or txt
You are on page 1of 148

Contents

1 Functions and limits 1


1.1 The idea of a limit . . . . . . . . . . . . . . . . . . . . . . . . . . . . . . . . . . . . . . . . . . . 1
1.2 Limit Laws . . . . . . . . . . . . . . . . . . . . . . . . . . . . . . . . . . . . . . . . . . . . . . 9
1.3 Infinite limits and asymptotes . . . . . . . . . . . . . . . . . . . . . . . . . . . . . . . . . . . . 13
1.4 Limits through One-sided Limits . . . . . . . . . . . . . . . . . . . . . . . . . . . . . . . . . . . 15
1.5 The Sandwich Theorem & Trigonometric Limits . . . . . . . . . . . . . . . . . . . . . . . . . . 16

2 Continuity of Functions 22
2.1 Continuity at a Point . . . . . . . . . . . . . . . . . . . . . . . . . . . . . . . . . . . . . . . . . 22
2.2 Left and Right Continuity at a Point . . . . . . . . . . . . . . . . . . . . . . . . . . . . . . . . . 25
2.3 Continuity on an Interval . . . . . . . . . . . . . . . . . . . . . . . . . . . . . . . . . . . . . . . 26
2.4 Theorems on Continuity . . . . . . . . . . . . . . . . . . . . . . . . . . . . . . . . . . . . . . . 26
2.5 Discontinuities . . . . . . . . . . . . . . . . . . . . . . . . . . . . . . . . . . . . . . . . . . . . 31

3 Differentiation 34
3.1 First Principles . . . . . . . . . . . . . . . . . . . . . . . . . . . . . . . . . . . . . . . . . . . . 34
3.2 Tangents . . . . . . . . . . . . . . . . . . . . . . . . . . . . . . . . . . . . . . . . . . . . . . . . 37
3.2.1 Non-vertical tangents . . . . . . . . . . . . . . . . . . . . . . . . . . . . . . . . . . . . . 37
3.2.2 Vertical tangents . . . . . . . . . . . . . . . . . . . . . . . . . . . . . . . . . . . . . . . 38
3.3 Differentiability vs. Continuity . . . . . . . . . . . . . . . . . . . . . . . . . . . . . . . . . . . . 39
3.4 The derivative as a function . . . . . . . . . . . . . . . . . . . . . . . . . . . . . . . . . . . . . . 41
3.5 Derivative Rules . . . . . . . . . . . . . . . . . . . . . . . . . . . . . . . . . . . . . . . . . . . . 43
3.6 Trigonometric Derivatives . . . . . . . . . . . . . . . . . . . . . . . . . . . . . . . . . . . . . . 53
3.7 Implicit Differentiation . . . . . . . . . . . . . . . . . . . . . . . . . . . . . . . . . . . . . . . . 57
3.8 Derivatives of Exponential and Logarithmic Functions . . . . . . . . . . . . . . . . . . . . . . . 60
3.9 Linear Approximation and Differentials . . . . . . . . . . . . . . . . . . . . . . . . . . . . . . . 66

4 Introduction to Integration 68
4.1 Antidifferentiation and the Indefinite Integral . . . . . . . . . . . . . . . . . . . . . . . . . . . . 68
4.2 The Definite Integral & FTC . . . . . . . . . . . . . . . . . . . . . . . . . . . . . . . . . . . . . 75
4.3 The Substitution Rule . . . . . . . . . . . . . . . . . . . . . . . . . . . . . . . . . . . . . . . . . 77
4.4 Integration by Parts . . . . . . . . . . . . . . . . . . . . . . . . . . . . . . . . . . . . . . . . . . 81

5 Differentiation Techniques 86
5.1 Higher Order Derivatives . . . . . . . . . . . . . . . . . . . . . . . . . . . . . . . . . . . . . . . 86
5.2 Continuity and Differentiation of Inverse Functions . . . . . . . . . . . . . . . . . . . . . . . . . 89
5.3 Inverse Trigonometric Derivatives and Integrals . . . . . . . . . . . . . . . . . . . . . . . . . . . 90
5.4 Logarithmic Differentiation . . . . . . . . . . . . . . . . . . . . . . . . . . . . . . . . . . . . . . 99

6 Applications of Differentiation 102


6.1 Indeterminate Forms and l’Hôpital’s Rule . . . . . . . . . . . . . . . . . . . . . . . . . . . . . . 102
6.2 Related Rates . . . . . . . . . . . . . . . . . . . . . . . . . . . . . . . . . . . . . . . . . . . . . 107
6.3 Extrema and the Extreme Value Theorem . . . . . . . . . . . . . . . . . . . . . . . . . . . . . . 111

i
ii 1st Semester Calculus Lecture Manual 2017 MATH1036

6.4 Rolle’s Theorem and the Mean Value Theorem . . . . . . . . . . . . . . . . . . . . . . . . . . . 117


6.5 Curve Sketching . . . . . . . . . . . . . . . . . . . . . . . . . . . . . . . . . . . . . . . . . . . . 120
6.5.1 Increasing and Decreasing Functions and The First Derivative Test . . . . . . . . . . . . . 121
6.5.2 Concavity and The Second Derivative Test . . . . . . . . . . . . . . . . . . . . . . . . . 125
6.5.3 A summary of Curve Sketching . . . . . . . . . . . . . . . . . . . . . . . . . . . . . . . 132
6.6 Optimization . . . . . . . . . . . . . . . . . . . . . . . . . . . . . . . . . . . . . . . . . . . . . 140

7 Hyperbolic Functions 146


Chapter 1

Functions and limits

LEARNING OUTCOMES:
On completion of this chapter you should (tick the checkbox when you have mastered the skill)
 1. understand the concept of a limit of a function at a point intuitively,
 2. be able to estimate the value of a limit of a function at a given point by calculating function values close
to the given point,
 3. be able to evaluate the limit of a function at a given point graphically,
 4. understand and be able to calculate one-sided limits,
 5. know the behaviour of a function with infinite limits,
 6. understand and be able to determine vertical and horizontal asymptotes of a function,
 7. be able to compute limits of exponential functions,
 8. be able to use the appropriate Limit Laws to find the limit of a function at a given point,
 9. know how to prove that the limit of a function at a given point exists using one-sided limits,
 10. be able to use factorization and rationalization to calculate limits of rational functions,
 11. be able to use the Squeeze Theorem to calculate limits,
 12. know, understand and be able to prove the theorems on trigonometric limits given in these notes on this
section and
 13. know how to solve the examinable tutorial problems and the worked out problems from these notes on
this chapter.

The single most important concept in all of calculus is that of a limit. Every single notion of calculus is encap-
sulated in one sense or another to that of a limit. In this course, we will only consider an intuitive and informal
discussion of a limit of a function at a point. The precise approach to limits, using the standard ε-δ definition is
reserved for the second year Analysis course and will not be considered here.

1.1 The idea of a limit


Consider a function f defined near the point x = a, not necessarily at a itself. We wish to investigate the behaviour
of f for values x in the domain of f that are close to a. We first consider this numerically, then graphically.
x−9
Worked Example 1.1.1. Consider the function f (x) = √ . What happens to the values of f (range values
x−3
of f ) when x (domain values of f ) is close to 9?
Solution. A table of values will conjecture the behaviour of f . We want to approach 9 and can do this in a
direction closely from the left (x < 9, i. e. from values below 9) and a direction closely from the right (x > 9, i. e.
from values above 9).

1
2 1st Semester Calculus Lecture Manual 2017 MATH1036

x−9
x<9 f (x) = √
x−3
8 5.828427
8.5 5.915476
8.9 5.983287
8.95 5.991655
8.99 5.998333
8.999 5.999833
8.9999 5.999983
8.99999 5.999998

It appears that as x approaches 9 from below (the left), the closer f (x) is to 6 , the limiting value of f .

To say “x approaches 9 from below” or “x approaches 9 from the left” we write symbolically x → 9− . We then
write lim− f (x) (called the left-hand limit) for the value that f(x) approaches as x → 9− . In the above calculations
x→9
we thus have
lim f (x) = 6 .
x→9−

Approaching 9 from the right gives the following table of values.

x−9
x>9 f (x) = √
x−3
9.1 6.016620
9.05 6.008322
9.001 6.000167
9.0001 6.000017
9.00001 6.0000017

Again, it can be observed that the closer that x is to 9 from above, the closer f (x) is to 6 . We write x → 9+
to denote “x approaches 9 from above” or “x approaches 9 from the right”. We then write lim+ f (x) (called the
x→9
right-hand limit) for the value that f (x) approaches as x → 9+ . In our calculations

lim f (x) = 6 .
x→9+

lim f (x) and lim+ f (x) are one-sided limits. We have


x→9− x→9

lim f (x) = 6 = lim+ f (x)


x→9− x→9

and we phrase this combined result as “the limit of f (x) as x approaches 9 exists and is equal to 6 ” and write

lim f (x) = 6 .
x→9

Both the left-hand limit lim− f (x) and the right-hand limit lim+ f (x) of a function must exist and must both
x→a x→a
be equal for the limit of the function lim f (x) to exist i. e.,
x→a

lim f (x) = L ⇔ lim− f (x) = L = lim+ f (x).


x→a x→a x→a

Graphically: The one-sided limits can be determined easily from the graph. Consider the graph of
x−9
f (x) = √ .
x−3
MATH1036 1st Semester Calculus Lecture Manual 2017 3

f (x)
6

9 x

It is clear that x = 9 is inadmissible since f (9) does not exist. However lim− f (x) = 6 = lim+ f (x) both exist,
x→9 x→9
easily “read off” from the graph of f so that lim f (x) = 6 .
x→9

Worked Example 1.1.2. (The number e) Let


1
f (x) = (1 + x) x .
Use the tabular method to estimate lim f (x) (if it exists) to five decimal places.
x→0
Solution. From the left at x = 0:
1
x<0 f (x) = (1 + x) x
-0.01 2.731999
-0.001 2.719642
-0.0001 2.718418
-0.00001 2.718295
-0.000001 2.718283

We see that to five decimal places lim− f (x) = 2.71828.


x→0
From the right at x = 0:

1
x>0 f (x) = (1 + x) x
0.01 2.704814
0.001 2.716924
0.0001 2.718146
0.00001 2.718262
0.000001 2.718280

We see that to five decimal places lim+ f (x) = 2.71828.


x→0

Hence, lim− f (x) = lim+ f (x) so that lim f (x) exists, and to five decimal places
x→0 x→0 x→0
1
lim f (x) = lim(1 + x) x ≈ 2.71828.
x→0 x→0

The exact value of this limit is given the symbol e (called Eulers number, accredited to the Swiss mathematician
Leonhard Euler (1707-1783) who was among the first to study its properties extensively). We thus have

1
lim(1 + x) x = e
x→0

The number e is an irrational number (like π) and is used frequently in all of Mathematics. 2 < e < 3 and to five
decimals e ≈ 2.71828. In the sequel, e will refer to this irrational number of Euler.
4 1st Semester Calculus Lecture Manual 2017 MATH1036

Note.
1. The function y = e x is called the natural exponential function.
2. Recall that the exponential functions given as y = a x with a > 0 have the inverse y = loga x. Particularly,
the logarithmic function to the base e, loge x, is called the natural logarithmic function y = loge x which we
denote in future as y = ln x (logarithmus naturalis), i. e.,

ln x = loge x.

3. y = ln x has all the logarithmic properties:


(a) y = ln x ⇔ x = ey ,
(b) eln x = x and ln e x = x,
(c) ln xr = r ln x,
loge x ln x
(d) loga x = = ,
loge a ln a
(e) ln xy = ln x + ln y,
Å ã
x
(f) ln = ln x − ln y.
y
You will encounter y = ln x in particular in the chapters on integration. Note the properties above remembering
that ln x = loge x.

Note: If f does not approach a unique value as x approaches a, then lim f (x) does not exist, i. e.,
x→a

lim f (x) , lim+ f (x) ⇒ lim f (x) DOES NOT EXIST.


x→a− x→a x→a

It is obvious that it is tedious to determine lim− f (x), lim+ f (x) and eventually the existence of lim f (x) using the
x→a x→a x→a
tabular approach. More importantly, the tabular approach does not prove the existence of the limit nor does
it give the exact value of the limit. The graphical way (if we know how to sketch the graph) is more succinct. A
computer algebra program may assist you in graphical sketching.

Worked Example 1.1.3. Sketch the graph of the given function and determine graphically whether the corre-
sponding limit exists; if so, find the limit.

1. f (x) = 4 − x2 . Find lim f (x).
x→2
√ √
Solution. From the graph of f , lim− 4 − x2 = 0 and 4 − x2 is not defined when x > 2. In this case we say
√ √ x→2
lim 4 − x2 exists, and lim 4 − x2 = 0.
x→2 x→2

√

 −x if x < 0,

2. Let g(x) = 3 − x if 0 ≤ x < 3,

if x > 3.

(x − 3)2

Find lim g(x) and lim g(x) if they exist.


x→0 x→3

Solution. From the graph of g,


lim g(x) = 0 and lim g(x) = 3.
x→0− x→0+

Since lim− g(x) , lim+ g(x), lim g(x) does not exist.
x→0 x→0 x→0
Since lim− g(x) = 0 and lim+ g(x) = 0, lim g(x) exists and lim g(x) = 0.
x→3 x→3 x→3 x→3
MATH1036 1st Semester Calculus Lecture Manual 2017 5

|x|
3. h(x) = . Find lim h(x) if it exists.
x x→0

Solution. From the graph of h we see that lim− h(x) = −1 and lim+ h(x) = 1. Since lim− h(x) , lim+ h(x), lim h(x)
x→0 x→0 x→0 x→0 x→0
does not exist.
(
1 if x ∈ Q, i. e., x is rational,
4. Let d(x) =
0 if x ∈ R \ Q, i. e., x is irrational.
Find for any a ∈ R, lim d(x) if it exists. The function d is called the Dirichlet function.
x→a

Solution. Let a ∈ R. As x → a− or x → a+ , x passes through both rational and irrational numbers. (The rationals
and irrationals are said to be dense in R. Between any two real numbers, there is a rational and an irrational
number.) Consequently as x approaches a, the values of the Dirichlet function jump back and forth between 0 and
1 and thus d(x) cannot approach and stay close to a fixed number L. Therefore lim d(x) does not exist.
x→a

5. Consider the greatest integer function defined as

bxc = greatest integer less than or equal to x.

Find limbxc if it exists.


x→4

Solution. We see from the graph that lim− bxc = 3 whilst lim+ bxc = 4. Thus lim− bxc , lim+ bxc and so limbxc does
x→4 x→4 x→4 x→4 x→4
not exist.
In general, we have for each integer n

lim bxc = n − 1 and lim bxc = n.


x→n− x→n+

The Dirichlet function defined in example 4 is one in which lim d(x) does not exists for any a ∈ R, whilst for the
x→a
greatest integer function limbxc does not exist for any a ∈ Z.
x→a

The graphs of the remaining problems have been sketched with a drawing program. We evaluate the limits of the
given functions below accordingly using the graphs. All trigonometric functions are defined in radian measure
(refer to the Algebra Lecture Guide).
6 1st Semester Calculus Lecture Manual 2017 MATH1036

Å ã
1
Worked Example 1.1.4. 1. Let f (x) = sin . Find lim f (x). (In this example, using the tabular method to
x x→0
conjecture the value of the limit is misleading.)

0.5

0
−1 −0.5 0 0.5 1

−0.5

−1

1
Solution. As x → 0, f frantically fluctuates between 1 and −1 never approaching a fixed value. Thus lim− sin
x→0 x
1 1
does not exist. The case is similar for x → 0+ so that lim+ sin also does not exist. Hence lim sin does not
x→0 x x→0 x
exist.

sin x
2. Let g(x) = . Find lim g(x).
x x→0

0.8

0.6

0.4

0.2

−10 −5 5 10

−0.2

Solution.
From the graph we see lim g(x) = 1. Later we will give a rigorous proof of this result.
x→0
MATH1036 1st Semester Calculus Lecture Manual 2017 7

1 − cos2 x
3. Let h(x) = . Find lim h(x).
x x→0

0.8

0.6

0.4

0.2

−10 −5 5 10

−0.2

−0.4

−0.6

−0.8

Solution.
From the graph we see lim h(x) = 0. Later we will give a rigorous proof of this result.
x→0

Tutorial 1.1.1. 1. Thomas Exercises 11th ed. 2.1 pp. 75–76: 1, 2, 4, 7, 8 or 12th ed. 2.2 p. 73: 1, 2, 4, 7, 8.
2. If f is a function such that f (2) = 4, can you conclude anything about lim f (x)? Explain your answer and
x→2
provide a graph for f to substantiate.
3. If g is a function such that lim g(x) = 4, can you conclude anything about g(2)? Explain your answer and
x→2
provide a graph for g to substantiate.
4. Let f (x) = x − bxc. For each integer n, find lim− f (x) and lim+ f (x) if they exist.
x→n x→n

In the above examples we showed that either a limit of a function f exists at x = a ( f (x) approaches a unique
value L ∈ R) or it does not exist ( f (x) does not approach a unique value).

The limit of a function f can also be unbounded, i. e., as x → a+ or x → a− , the values of f (x) become positively
large or negatively large. We use the infinity symbol ∞ to denote unboundedness. +∞, or shortly ∞, will denote
positively large values and −∞ will denote negatively large values. ∞ is a symbolic representation of having no
bound/limit and is not a real number. You cannot algebraically operate with ∞ as if it were an element of the reals.

The following infinite limit cases arise


1. lim− f (x) = ∞ = lim+ f (x) ⇒ lim f (x) = ∞ means that f (x) is arbitrarily positively large for x sufficiently
x→a x→a x→a
close to a. lim f (x) in fact does not exist here but the infinity notation describes the behaviour of f close to a.
x→a
2. lim− f (x) = −∞ = lim+ f (x) ⇒ lim f (x) = −∞ means that f (x) is arbitrarily negatively large for x sufficiently
x→a x→a x→a
close to a. lim f (x) in fact does not exist here but the infinity notation describes the behaviour of f close to a.
x→a
3. lim− f (x) = ∞ and lim+ f (x) = −∞ (or vice versa). lim f (x) in fact does not exist here but the infinity notation
x→a x→a x→a
describes the behaviour of f close to a− and a+ .

The following examples illustrate infinite (unbounded) limits graphically. In such cases lim f (x) DOES NOT
x→a
EXIST.
8 1st Semester Calculus Lecture Manual 2017 MATH1036

1
Worked Example 1.1.5. 1. Let f (x) = . Find lim− f (x), lim+ f (x), lim f (x).
x x→0 x→0 x→0
Solution. lim− f (x) = −∞, lim+ f (x) = ∞, lim f (x) does not exist.
x→0 x→0 x→0

1
2. Let g(x) = − 2 . Find lim− g(x), lim+ g(x), lim g(x).
x x→0 x→0 x→0
Solution. lim− g(x) = −∞, lim+ g(x) = −∞, lim g(x) = −∞.
x→0 x→0 x→0

3. Find limπ − sec x and limπ + sec x.


x→ 2 x→ 2

Solution. limπ − sec x = ∞ and limπ + sec x = −∞.


x→ 2 x→ 2

Vertical asymptotes arise when limits are infinite. These are auxiliary vertical lines that the function values
approach (but never attain). We say that the line x = a is a vertical asymptote of f if at least one of the following
limits hold

lim f (x) = ∞ or lim f (x) = ∞ or lim f (x) = −∞ or lim f (x) = −∞


x→a− x→a+ x→a− x→a+
(or lim f (x) = ∞ or lim f (x) = −∞).
x→a x→a

The previous worked example illustrates the concept of a vertical asymptote.


Horizontal asymptotes arise when we consider the limit of a function f for unbounded values of x, i. e., lim f (x)
x→∞
and lim f (x). If, for arbitrary positively or negatively large values of x, the function values approach a unique
x→−∞
value b ∈ R, i. e., if lim f (x) = b or lim f (x) = b, then the line y = b is called a horizontal asymptote.
x→∞ x→−∞

Horizontal and vertical asymptotes are useful in sketching graphs, as you will discover in Chapter 6 of this guide.
1
Worked Example 1.1.6. 1. Let f (x) = 2 + . Find lim f (x) and lim f (x) from the graph of f .
x x→∞ x→−∞
Solution. lim f (x) = 2 and lim f (x) = 2.
x→∞ x→−∞
1
2. Let g(x) = 2 . Find lim g(x) and lim g(x) from the graph of g.
x x→∞ x→−∞
Solution. lim g(x) = 0 and lim g(x) = 0.
x→∞ x→−∞
2x2
3. Let h(x) = . From the graph evaluate lim− h(x), lim+ h(x), lim − h(x), lim + h(x), lim h(x) and lim h(x).
9 − x2 x→3 x→3 x→−3 x→−3 x→∞ x→−∞
What are the horizontal and vertical asymptotes (if any)?
Solution. lim− h(x) = ∞, lim+ h(x) = −∞, lim − h(x) = −∞, lim + h(x) = ∞, lim h(x) = −2 and lim h(x) = −2.
x→3 x→3 x→−3 x→−3 x→∞ x→−∞
y = −2 is a horizontal asymptote, and x = 3 and x = −3 are vertical asymptotes.

f (x)
For functions of the form , we may possibly argue with the behaviour of the numerator and denominator near
g(x)
f (x)
a to calculate lim .
x→a g(x)
Worked Example 1.1.7. Evaluate the following limits without the use of a sketch.
2 2
1. lim+ and lim− .
x→3 x − 3 x→3 x − 3

Solution: The numerator remains unchanged at 2 as x → 3− and x → 3+ . However the denominator (x − 3) → 0+


2 2 2
as x → 3+ (i. e., x > 3). We may write symbolically → + = ∞ as x → 3+ . Thus lim+ = ∞.
x−3 0 x→3 x − 3
2 2
Also, as x → 3− (x < 3), the denominator (x − 3) → 0− . Thus → − = −∞ as x → 3− . Hence
x−3 0
2
lim− = −∞.
x→3 x − 3
From the above analysis, the line x = 3 is a vertical asymptote.
MATH1036 1st Semester Calculus Lecture Manual 2017 9

x−1
2. lim + .
x→−2 x2 (x + 2)
Solution: As x → −2+ (0 > x > −2 but close), the numerator (x − 1) → −3 whilst x2 → 4 and (x + 2) → 0+ so that
x−1 −3 x−1
the denominator x2 (x+2) → 0+ . Thus x → −2+ implies that 2 → + = −∞. Thus lim + 2 = −∞.
x (x + 2) 0 x→−2 x (x + 2)
Also, x = −2 is a vertical asymptote.
3. lim− cosec x.
x→π
1 1
Solution: lim− cosec x = lim− . As x → π− , the denominator sin x → 0+ . Consequently, cosec x = →
x→π sin x x→π sin x
1
= ∞ as x → π− . Hence, lim− cosec x = ∞. Of course, x = π is a vertical asymptote of cosec x.
0+ x→π

Tutorial 1.1.2. 1. Thomas Exercises 11th ed. 2.5 pp. 117–118: 1, 2, 6, 11, 13, 16, 21, 42 or 12th ed. 2.6 pp.
114-115: 37, 38, 42, 47, 49, 52, 57, 72.

1.2 Limit Laws


We now will make use of the following Limit Laws to calculate a limit of a function rather than the approaches
used in Section 1.1.

Theorem 1.1 (Limit Laws). Let c ∈ R and suppose that lim f (x) = L and lim g(x) = M both exist. Then
x→a x→a

1. lim[ f (x) + g(x)] = lim f (x) + lim g(x) = L + M.


x→a x→a x→a

2. lim[ f (x) − g(x)] = lim f (x) − lim g(x) = L − M.


x→a x→a x→a

3. lim[c f (x)] = c lim f (x) = cL.


x→a x→a
h ih i
4. lim[ f (x)g(x)] = lim f (x) lim g(x) = LM.
x→a x→a x→a

f (x) lim
x→a
f (x) L
5. if M , 0, lim = = .
x→a g(x) lim g(x) M
x→a

f (x)
6. if L , 0 and M = 0, lim does not exist.
g(x) x→a

 h in
7. if n ∈ Z+ , lim f (x)n = lim f (x) = Ln .

x→a x→a

8. lim c = c.
x→a

9. lim x = a.
x→a

10. if n ∈ Z+ , lim xn = an .
x→a
√n √n
11. if n ∈ Z+ , lim x= a. If n is even, we assume that a > 0.
x→a
√n √n
12. if n ∈ Z+ , lim f (x) = lim f (x) =
»
n L.
x→a x→a

13. if lim |h(x)| = 0, then lim h(x) = 0.


x→a x→a

Worked Example 1.2.1. Use the above Limit Laws to calculate the following limits if they exist.
1. lim(5x2 − 9x − 8).
x→4
10 1st Semester Calculus Lecture Manual 2017 MATH1036

Solution.

lim(5x2 − 9x − 8) = lim 5x2 − lim 9x − lim 8


x→4 x→4 x→4 x→4
= 5 lim x2 − 9 lim x − lim 8
x→4 x→4 x→4
= 5(42 ) − 9(4) − 8
= 80 − 36 − 8
= 36.
2 √
x3 + 3 x
2. lim .
x→8 4 − 16
x
Solution.

î 2 √ ó

x 3 + 3 x lim
2 x3 + 3 x
x→8
lim =
x→8 4 − 16 lim 4 − 16x
 
x x→8
2 √
lim x 3 + lim 3 x
x→8 x→8
=
16
lim 4 − lim
x→8 x→8 x
√3 √
lim x2 + 3 lim x
x→8 x→8
=
lim 16
x→8
lim 4 −
x→8 lim x
x→8
q √
3 lim x2 + 3 8
x→8
=
16
4−
8
√3 √
64 + 3(2 2)
=
2

4 + 2(3 2)
=
2

= 2 + 3 2.

4x − 3
3. lim .
x2 − 4
x→2

Solution.
lim(4x − 3) = 4(2) − 3 = 5 , 0
x→2

whilst
lim(x2 − 4) = 22 − 4 = 0.
x→2

4x − 3
Thus lim does not exist (Theorem 1.1,6.).
x→2 x2 − 4

Recall that a polynomial function is of the form

f (x) = bn xn + bn−1 xn−1 + · · · + b2 x2 + b1 x + b0

p(x)
with bi ∈ R for i = 1, 2, . . . , n and n any non-negative integer. A rational function is of the form f (x) = with
q(x)
p(x) and q(x) polynomials. We then have the following as a consequence of Theorem 1.1 (1,8,10,5).
MATH1036 1st Semester Calculus Lecture Manual 2017 11

Theorem 1.2. If f is a polynomial or a rational function and a is in the domain of f , then lim f (x) = f (a).
x→a

2x2 + 1
Worked Example 1.2.2. Evaluate lim .
x→2 x2 + 6x − 4
Solution. Since we are finding the limit of a rational function, we can do direct substitution by Theorem 1.2:

2x2 + 1 2(22 ) + 1 9 3
lim = = = .
x→2 x + 6x − 4 2 + 6(2) − 4 12 4
2 2

p(x)
For rational functions f (x) = where both the numerator and denominator tend to zero (said to be of the
q(x)
0
indeterminate form ), we use factorization to simplify the limit and then use the Limit Laws.
0
Worked Example 1.2.3. Evaluate the following limits if they exist.
x2 − 4
1. lim .
x→2 x − 2

Solution.

x2 − 4 (x − 2)(x + 2)
lim = lim
x→2 x−2 x→2 x−2
= lim(x + 2) ∵ x , 2 we may cancel the factor (x − 2)
x→2
= 4.

x3 + 1
2. lim .
x→−1 x+1
Solution.

x3 + 1 (x + 1)(x2 − x + 1)
lim = lim
x→−1 x+1 x→−1 x+1
= lim (x2 − x + 1) ∵ x , −1 we may cancel the factor (x + 1)
x→−1
= 3.

x+1
3. lim .
x→−1 (2x2 + 7x + 5)2
Solution.

x+1 x+1
lim = lim
x→−1 (2x2 + 7x + 5)2 x→−1 [(2x + 5)(x + 1)]2
x+1
= lim
x→−1 (2x + 5)2 (x + 1)2
1
= lim ∵ x , −1 we may cancel the factor (x + 1).
x→−1 (2x + 5)2 (x + 1)

1
Since lim (2x + 5)2 (x + 1) = (−2 + 5)2 (0) = 0, by Theorem 1.1.6, lim does not exist. Hence the
x→−1 x→−1 (2x + 5)2 (x + 1)
x+1
original limit lim does not exist.
x→−1 (2x2 + 7x + 5)2
12 1st Semester Calculus Lecture Manual 2017 MATH1036

x4 − 16
4. lim .
x→2 x3 − 8
Solution. Substitution fails since 2 is not in the domain of the given function. However, factorization produces

x4 − 16 (x − 2)(x + 2)(x2 + 4)
lim = lim
x→2 x3 − 8 x→2 (x − 2)(x2 + 2x + 4)
(x + 2)(x2 + 4)
= lim ∵ x , 2 we may cancel the factor (x − 2)
x→2 x2 + 2x + 4
(x + 2)(x2 + 4)
= lim by Theorem 1.1.12
x→2 x2 + 2x + 4

(2 + 2)(22 + 4)
= by Theorems 1.1 and 1.2
22 + 2(2) + 4

4(8)
=
4+4+4

8
= .
3
0
Certain quotients are also of the form where the Quotient Limit Law cannot be applied immediately. However,
0
rationalizing the numerator or denominator may allow us to proceed with the Limit Laws.

Worked Example 1.2.4. Evaluate the following limits.



x+1−1
1. lim .
x→0 x
Solution.
√ Ç√ åÇ √
x+1−1 x+1−1 x+1+1
å
lim = lim √ (rationalizing the numerator)
x→0 x x→0 x x+1+1
(x + 1) − 1
= lim √
x→0 x( x + 1 + 1)
x
= lim √
x→0 x( x + 1 + 1)

1
= lim √ ∵ x , 0 we may cancel the factor x
x→0 x+1+1
1 1 1 1
√ = » = = .
lim( x + 1 + 1) lim x + 1 + 1 1 + 1 2
x→0 x→0

x
2. lim √ .
x→0 1 + 3x − 1
Solution.
ãÇ √
1 + 3x + 1
Å å
x x
lim √ = lim √ √ (rationa1izing the denominator)
x→0 1 + 3x − 1 x→0 1 + 3x − 1 1 + 3x + 1

x( 1 + 3x + 1)
= lim
x→0 (1 + 3x) − 1

x( 1 + 3x + 1)
= lim
x→0 3x

1 + 3x + 1
= lim ∵ x , 0 we may cancel x
x→0 3
Ä√ ä 1 Åq 1 √
ã
1 2
= lim 1 + 3x + 1 = lim(1 + 3x) + 1 = ( 1 + 1) = .
3 x→0 3 x→0 3 3
MATH1036 1st Semester Calculus Lecture Manual 2017 13

Tutorial 1.2.1. 1. Thomas Exercises 11th ed. 2.2 pp. 83–85: 1, 3, 4, 5, 10, 15, 16, 20, 28, 31, 36, 39, 42, 55, 58
or 12th ed. 2.2 pp. 74–76: 11–14, 18, 21, 22, 24, 34, 37, 42, 53, 56, 79, 82.
2. Given that lim f (x) = 2, lim g(x) = 0 and lim h(x) = −3 find the limits that exist. If the limit does not exist,
x→a x→a x→a
explain why. √
a. lim[ f (x) − h(x)], b. lim( f (x))3 , c. lim 3 2 f (x) − 4h(x),
x→a x→a x→a
g(x) f (x)
d. lim , e. lim .
x→a h(x) x→a g(x)
3. Find the limit, if it exists.
√ √
3 + x x2 + 4 − 5
1 1 Å ã
1 1
a. lim , b. lim √ , c. lim √ − .
x→−3 3 + x x→1 x+3−2 x→0 x 1 + x x

1.3 Infinite limits and asymptotes


Horizontal asymptotes of graphs of functions correspond to limits of functions as x → ∞ and x → −∞. The
following theorems and techniques allow calculating the limits algebraically.

Theorem 1.3 (Horizontal aymptote: x-axis). If r is any positive rational number, then

1 1
lim =0 and lim = 0,
x→∞ xr x→−∞ xr
p
where in the case of x → −∞ we have to assume that xr is defined for negative x, i. e., r can be written as r =
q
where p and q are positive integers with q odd.

Worked Example 1.3.1.


1
1. lim 3 = 0.
x→∞ x
1
2. lim √3 = 0.
x→−∞ x2
If f is a rational function, then lim f (x) and lim f (x) may be found by first dividing the numerator and denomi-
x→∞ x→−∞
nator by the highest power of x in the denominator and then apply Theorem 1.3.

Worked Example 1.3.2. Evaluate the following limits.


2x − 3
1. lim 2 .
x→∞ x + 7x − 1

Solution.
2x 3
2x − 3 2
− 2
lim = lim 2 x x
x→∞ x2 + 7x − 1 x→∞ x 7x 1
+ −
x2 x2 x2
2 3
− 2
= lim x x
x→∞ 7 1
1+ − 2
x x
1 1
2 lim − 3 lim 2
x→∞ x x→∞ x
=
1 1
1 + 7 lim − lim 2
x→∞ x x→∞ x
2(0) − 3(0)
=
1 + 7(0) − 0
0
= = 0.
1
14 1st Semester Calculus Lecture Manual 2017 MATH1036

2x − 3 x−3
Thus lim = 0 so that y = 0, the x-axis, is a horizontal asyrnptote of the function 2 .
x→∞ x2 + 7x − 1 2x + 7x − 1

9x2 + 2
2. lim .
x→∞ 3 − 4x

Solution.
Ç√ Ç√
9x2 + 2 9x2 + 2
å å
√ √
9x2 + 2 x x2 √
lim = lim = lim ∵ x > 0, x2 = x
x→∞ 3 − 4x x→∞ 3 4x x→∞ 3
− −4
x x x
9x2

2 2
+ 2 9+ 2
x2 x x
= lim = lim
x→∞ 3 x→∞ 3
−4 −4
…x … x
2 2
lim 9 + 2 9 + lim 2
x→∞ x x→∞ x
= ã =
3
Å
3
lim −4 lim − 4
x→∞ x x→∞ x

9+0 3
= =− .
0−4 4

3 9x2 + 2
Thus y = − is a horizontal asyrnptote of .
4 3 − 4x
Theorem 1.4 (Vertical asymptote: x = a).
1
1. If n is an even positive integer, then lim = ∞.
(x − a)n
x→a
1 1
2. If n is an odd positive integer, then lim+ = ∞ and lim− = −∞.
x→a (x − a)n x→a (x − a)n

1 1
Worked Example 1.3.3. Evaluate lim+ and lim− .
x→3 (x − 3)3 x→3 (x − 3)3

1 1
Solution. lim+ = ∞ and lim− = −∞. Thus the line x = 3 is a vertical asyrnptote of the function
x→3 (x − 3)3 x→3 (x − 3)3
1
f (x) = .
(x − 3)3

Theorem 1.5 (Limits of exponential functions). For the exponential function a x ,


1. if a > 1, then
lim a x = ∞ and lim a x = 0.
x→∞ x→−∞

2. if 0 < a < 1, then


lim a x = 0 and lim a x = ∞.
x→∞ x→−∞

Worked Example 1.3.4. Evaluate the following exponential limits.


1. lim e x and lim e x .
x→∞ x→−∞

Solution. Since e > 1, lim e x = ∞ and lim e x = 0. Thus y = 0, the x-axis is a horizontal asymptote of y = e x .
x→∞ x→−∞
MATH1036 1st Semester Calculus Lecture Manual 2017 15

e x − e−x e x − e−x
2. lim and lim .
x→∞ e x + e−x x→−∞ e x + e−x

Solution.
e x − e−x e x (1 − e−2x )
lim = lim
x→∞ e x + e−x x→∞ e x (1 + e−2x )

1 − e−2x
= lim
x→∞ 1 + e−2x
lim 1 − lim e−2x
x→∞ x→∞
=
lim 1 + lim e−2x
x→∞ x→∞
1−0
= ∵ 0 < e−2 < 1
1+0
=1

and
e x − e−x e−x (e2x − 1)
lim = lim
x→−∞ e x + e−x x→−∞ e−x (e2x + 1)

e2x − 1
= lim −2x
x→−∞ e +1
lim e2x − lim 1
x→−∞ x→−∞
=
lim e2x + lim 1
x→−∞ x→∞
0−1
= ∵ e2 > 1
0+1
= −1.
e x − e−x
Thus y = 1 and y = −1 are horizontal asymptotes of the curve y = . This particular curve is called the
e x + e−x
e x − e−x
hyperbolic tangent function and denoted by tanh(x) = x . (You will encounter this function and the other
e + e−x
hyperbolic functions in your self-study of chapter 7.)

Tutorial 1.3.1. 1. Thomas Exercises 11th ed. 2.4 p. 108: 52, 58, 59, 63, 64, 65 or 12th ed. 2.6 p. 114: 14, 20,
21, 29, 30, 31.
2. Thomas Exercises 12th ed. 2.6 p. 116: 80, 82, 85.

1.4 Limits through One-sided Limits


The following theorem allows us to prove algebraically the existence (or non-existence) of limits and to calculate
their value. We used this theorem graphically in section 1.1.

Theorem 1.6. lim f (x) = L ⇔ lim− f (x) = L = lim+ f (x).


x→a x→a x→a

Worked Example 1.4.1. 1. Prove that lim |x| = 0.


x→0

Proof. We recall that (


x if x ≥ 0,
|x| =
−x if x < 0.
Then

lim |x| = lim− (−x) ∵ x → 0− → x < 0 → |x| = −x


x→0− x→0
=0
16 1st Semester Calculus Lecture Manual 2017 MATH1036

and

lim |x| = lim+ (x) ∵ x → 0+ → x > 0 → |x| = x


x→0+ x→0
= 0.

Thus lim− |x| = 0 = lim+ |x| so that by Theorem 1.6, lim |x| = 0, 
x→0 x→0 x→0

|x + 4|
2. Determine algebraically whether lim exists.
x→−4 x + 4

Solution. Since

x+4 if x + 4 ≥ 0
(
|x + 4| =
−(x + 4) if x + 4 < 0
x+4
(
if x ≥ −4
=
−(x + 4) if x < −4,

we have the left-hand limit


|x + 4| −(x + 4)
lim = lim −
x→−4− x+4 x→−4 x+4
= lim − (−1)
x→−4
= −1

and the right-hand limit


|x + 4| x+4
lim = lim +
x→−4+ x+4 x→−4 x + 4
= lim 1
x→−4−

= 1.

|x + 4| |x + 4| |x + 4|
Thus lim − , lim + and hence by Theorem 1.6, lim does not exist.
x→−4 x+4 x→−4 x + 4 x→−4 x + 4

Tutorial 1.4.1. 1. Thomas Exercises 11th ed. 2.4 p. 108–109: 12–15, 17–20, 82 or 12th ed. 2.4 pp. 91–92:
12–15, 17–20, 52.
2. For the following functions f (x), determine if the one-sided limits at the indicated point a exist, find whether
the limit at a exists, and compare with the value of f (a) if f is defined at a.
1 1 x2 − 1
a. f (x) = − at a = 0, b. f (x) = at a = 1, c. f (x) = bxc + b−xc at a = −3.
x |x| |x − 1|

1.5 The Sandwich Theorem & Trigonometric Limits


The next important theorem allows us to compute the limits of functions that are bounded (squeezed, pinched or
sandwiched) between two other functions that have the same limit.

Theorem 1.7 (Sandwich Theorem). If f (x) ≤ g(x) ≤ h(x) when x is near a (except possibly at a) and
lim f (x) = L = lim h(x), then lim g(x) = L.
x→a x→a x→a

Note. The Sandwich Theorem is also called Squeeze Theorem.

Worked Example 1.5.1. 1. Use the Sandwich Theorem to prove that lim(x2 + 1) = 1 using the fact that
x→0
lim(|x| + 1) = 1.
x→0
MATH1036 1st Semester Calculus Lecture Manual 2017 17

Proof. For −1 ≤ x ≤ 1 it follows that 0 ≤ x2 ≤ |x|. Thus

1 ≤ x2 + 1 ≤ |x| + 1.

Since lim 1 = 1 and it is given that lim(|x| + 1) = 1, by the Sandwich Theorem lim(x2 + 1) = 1. 
x→0 x→0 x→0

2. Use the Sandwich theorem to prove that lim sin θ = 0 and hence conclude that lim cos θ = 1. (Note that θ is in
θ→0 θ→0
radian measure.)

Proof. Consider the sector OAC with central angle θ in the unit circle for θ > 0 (first quadrant) and θ < 0 (fourth
quadrant).

O D C
1
θ

| sin θ|
A
|θ|

|θ|
| sin θ|

θ
−1
O D C

θ>0 θ<0

For θ , 0,

length of AD < length of AC < length of AC


ˆ ⇒ length of AD < length of AC
ˆ
⇒ | sin θ| < (OC)|θ| (∵ Arc length = θ).

Then for such θ we have

0 < | sin θ| < (1)|θ| ⇒ 0 < | sin θ| < |θ|.

Since lim 0 = 0 and lim |θ| = 0, by the Sandwich Theorem, lim | sin θ| = 0. Consequently,
θ→0 θ→0 θ→0

lim sin θ = 0.
θ→0


Since cos2 θ + sin2 θ = 1 and for θ close to 0, cos θ > 0, we have cos θ = 1 − sin2 θ. Hence
p
lim cos θ = lim 1 − sin2 θ
θ→0 θ→0
q
= lim(1 − sin2 θ)
θ→0
q
= 1 − lim sin2 θ
θ→0
q
= 1 − (lim sin θ)2
θ→0

= 1−0
= 1. 
18 1st Semester Calculus Lecture Manual 2017 MATH1036

1
3. Prove that lim x sin = 0 (note that x is in radians).
x→0 x

1 1
Proof. Since −1 ≤ sin t ≤ 1 for all t ∈ R, it follows that for x , 0, −1 ≤ sin ≤ 1, i. e., sin ≤ 1. Thus
x x

1 1
x sin = |x| sin ≤ |x|(1) = |x|
x x

so that
1
0 ≤ x sin ≤ |x|.
x
Since lim 0 = 0 and lim |x| = 0, by the Sandwich Theorem
x→0 x→0

1
lim x sin = 0.
x→0 x

Consequently,
1
lim x sin = 0. 
x→0 x
sin x
4. Determine lim .
x→∞ x
1 sin x 1 sin x
Solution. Since −1 ≤ sin x ≤ 1, it follows that for x > 0 (x → ∞), − ≤ ≤ , i. e., the graph of is
Å ã x x x x
1 1 1 1
bounded by those of − and . Since lim − = 0 and lim = 0, we have by the Sandwich Theorem that
x x x→∞ x x→∞ x

sin x
lim = 0.
x→∞ x

The Sandwich Theorem has fundamental applications in trigonometric limit calculations as seen in the examples
above. We will now consider evaluating trigonometric limits with all calculations based on radian measure.

We recall from the above example, part 2, that

lim sin θ = 0 and lim cos θ = 1


θ→0 θ→0

which will be used in Theorem 1.8.

The next theorem is a crucial result that will be used in Chapter 3 of these notes. You will need to recall the Area
of sector calculation in the Algebra Guide. The area of a sector is given by

1 2
A= r θ
2
where θ is the central angle of the sector with radius r. Again we must stress that all angle measurement is in
radians.
MATH1036 1st Semester Calculus Lecture Manual 2017 19

sin θ
Theorem 1.8. lim = 1.
θ→0 θ
Proof. Consider the sector OAC in the first quadrant (0 < θ < π2 ) of a circle of radius 1.

1
B
A

tan θ
sin θ
θ
O D C

θ>0

We see that

Area of 4OAC < Area of sector OAC < Area of 4OBC


1 1 1
⇒ (OC)(AD) < (OC)2 θ < (OC)(BC)
2 2 2
1 1 2 1
⇒ (1) sin θ < (1) θ < (1) tan θ
2 2 2
1 1 1
⇒ sin θ < θ < tan θ
2 2 2
⇒ sin θ < θ < tan θ
sin θ
⇒ sin θ < θ <
cos θ
θ 1
⇒1< <
sin θ cos θ
π
since for 0 < θ < we have sin θ > 0, and so dividing by sin θ leaves the inequalities unchanged. Taking inverses
2
sin θ
we have 1 > > cos θ or equivalently
θ
sin θ
cos θ < < 1. (∗)
θ
Now if − π2 < θ < 0, then 0 < −θ < π2 so that by the result just established
sin(−θ)
cos(−θ) < < 1,
−θ
which reduces again to (∗) since sin(−θ) = − sin θ and cos(−θ) = cos θ. Thus the inequality (∗) also holds for
− π2 < θ < 0. Hence for all θ ∈ (− π2 , π2 ) except θ = 0,
sin θ
cos θ < < 1.
θ
sin θ
Since lim cos θ = 1 and lim 1 = 1, by the Sandwich Theorem lim = 1. 
θ→0 θ→0 θ→0 θ

By the above
sin A A
lim =1 or lim =1
A→0 A A→0 sin A
20 1st Semester Calculus Lecture Manual 2017 MATH1036

Together with the above theorem and the Limit Laws we also have the following important limit.

1 − cos θ
Theorem 1.9. lim = 0.
θ→0 θ
Proof.

1 − cos θ 1 − cos θ 1 + cos θ


Å ãÅ ã
lim = lim
θ→0 θ θ→0 θ 1 + cos θ
1 − cos θ2
= lim
θ→0 θ(1 + cos θ)

sin2 θ
= lim
θ→0 θ(1 + cos θ)
sin θ sin θ
Å ãÅ ã
= lim
θ→0 θ 1 + cos θ
sin θ sin θ
Å ãÅ ã
= lim lim
θ→0 θ θ→0 1 + cos θ
ã lim sin θ
sin θ
Å
θ→0
= lim
θ→0 θ lim(1 + cos θ)
θ→0
Å
sin θ
ã lim sin θ
θ→0
= lim
θ→0 θ 1 + lim cos θ
θ→0
0
= (1)
1+1
= 0.

Worked Example 1.5.2. Evaluate the following trigonometric limits.


sin 5x
1. lim .
x→0 x
Solution.
sin 5x 5 sin 5x sin 5x
lim = lim = 5 lim = 5(1) = 5.
x→0 x x→0 5x x→0 5x

sin2 x
2. lim .
x→0 x
Solution.

sin2 x
Å ã Å ã
sin x sin x sin x sin x  
lim = lim = lim sin x = lim lim sin x = (1)(0) = 0.
x→0 x x→0 x x→0 x x→0 x x→0

sin(x2 )
3. lim .
x→0 x
Solution.
sin(x2 ) x sin x2  Å sin x2
ã
lim = lim = lim x lim = (0)(1) = 0.
x→0 x x→0 x2 x→0 x→0 x2

sin2 x
4. lim .
x→0 x2

Solution.
sin2 x sin x 2 sin x 2
Å ã Å ã
lim 2 = lim = lim = (1)2 = 1.
x→0 x x→0 x x→0 x
MATH1036 1st Semester Calculus Lecture Manual 2017 21

sin θ
5. lim .
θ→0 θ + tan θ
Solution.
sin θ
sin θ lim 1
lim = lim sin θ = lim
1
=
θ→0
=
1
=
1
.
θ→0 θ + tan θ θ→0 θ tan θ θ→0 θ 1 θ 1 1+ 1 2
+ + lim + 1
sin θ sin θ sin θ cos θ θ→0 sin θ lim cos θ
θ→0

Tutorial 1.5.1. 1. Thomas Exercises 11th ed. 2.2 p. 84: 49, 50, 51a, 52a or 12th ed. 2.2 p. 75: 63, 64, 65a, 66a.
2. Thomas Exercises 11th ed. 2.4 p. 108: 22, 25, 28, 29, 31, 33, 36 or 12th ed. 2.4 p. 92: 22, 25, 28, 29, 31, 33,
35, 40.
3. Thomas Chapter 2 Additional and Advanced Exercises 11th ed. p. 143: 27–30 or 12th ed. p. 121: 27–30.
(
x if x is rational,
4. Let f (x) = Show that lim f (x) = 0.
0 if x is irrational. x→0
Chapter 2

Continuity of Functions

LEARNING OUTCOMES:
On completion of this chapter you should (tick the checkbox when you have mastered the skill)

 1. know and understand the limit definitions of continuity of a function at a point, continuity of a function
over an interval and a continuous function,
 2. be able to establish if a given function is continuous at a point using the definition,
 3. know and understand left-continuity (continuous from the left) and right-continuity (continuous from the
right),
 4. know when and be able to test whether a function is continuous over an interval,
 5. know and understand the various theorems on continuity given in these notes of this section,
 6. know the statement of and understand the Intermediate Value Theorem (IVT),
 7. apply the IVT in determining existence of roots of a given function over an interval,
 8. know, understand and identify the various types of discontinuities and
 9. know how to solve the examinable tutorial problems and the worked out problems from these notes on
this chapter.

The usual meaning of the word continuous in everyday language is that of going on all the time or proceeding
without a break or interruptions and without abrupt changes.

From a mathematics viewpoint, this can also be taken as a loose meaning of continuity. A curve is continuous
between two points a and b if you can draw it in one go from a without lifting up your pencil until you reach b.

We will now present the precise meaning of continuity, which is closely related to the concept of the limit described
in the previous chapter.

2.1 Continuity at a Point


We have seen in the previous chapter that it is not necessary for a function f to be defined at a point a in order
for f to have a limit at a as in Worked Example 1.1.3.3 or all the examples of Worked Example 1.1.4. That is,
lim f (x) may exist even though a is not in the domain of f .
x→a

On the other hand, if f (a) exists, i. e., f is defined at a, and lim f (x) exists it may not be the case that both f (a)
x→a
and lim f (x) are equal as illustrated in the following example.
x→a

22
MATH1036 1st Semester Calculus Lecture Manual 2017 23

Worked Example 2.1.1. Let (


1 if x , 0,
f (x) =
2 if x = 0.
Find f (0) and lim f (x).
x→0
Solution.
f (0) = −2 whilst lim− f (x) = 1 = lim+ f (x). Thus lim f (x) = 1.
x→0 x→0 x→0

The above example clearly shows that f (0) , lim f (x). However, in many cases we do have equality of these two
x→0
quantities f (a) and lim f (x). This leads to the definition of continuity at a point given below.
x→a

The function f is continuous at the point a if and only if f (a) = lim f (x).
x→a

Equivalently (called the slot-machine test),

f is continuous at a if and only if the following three conditions are satisfied:


1. f (a) is defined, i. e., a is in the domain of f ,
2. lim f (x) exists, i.e., lim− f (x) = lim+ f (x), and
x→a x→a x→a
3. f (a) = lim f (x).
x→a

In most cases we will use the above three conditions to determine continuity at a point algebraically. If just one of
the three conditions above fails, then f is not continuous at the given point. If such is the case, then f is said to be
discontinuous at the given point.

Worked Example 2.1.2. 1. Determine whether

 |x + 3|

if x , −3,
f (x) = x+3
1 if x = −3,

is continuous at x = −3.
Solution. −3 is certainly in the domain of f and f (−3) = 1 by definition of f . Also,
|x + 3| −(x + 3)
lim f (x) = lim − = lim − = lim − (−1) = −1
x→−3− x→−3 x+3 x→−3 x+3 x→−3

whilst
|x + 3| (x + 3)
lim f (x) = lim + = lim + = lim + 1 = 1.
x→−3+ x+3x→−3 x→−3 x + 3 x→−3

Thus lim− f (x) , lim+ f (x) so that lim f (x) does not exist. Hence f is discontinuous at x = −3 since condition 2
x→3 x→3 x→3
of the slot machine test is violated.
2. Is 

 x2 + 4 if x < 2,

g(x) = 5 if x = 2,

if x > 2,

 3
x
continuous at x = 2?
24 1st Semester Calculus Lecture Manual 2017 MATH1036

Solution. Certainly g(2) = 5 is defined. Also


poly
lim g(x) = lim− (x2 + 4) = 22 + 4 = 8
x→2− x→2

and
poly
lim g(x) = lim+ x3 = 23 = 8
x→2+ x→2
so that lim g(x) exists and
x→2
lim g(x) = 8.
x→
However g(2) , lim g(x) so that condition 3 in the definition of continuity at a point is violated. Hence, g is
x→2
discontinuous at x = 2.
3. Show that 

 sin x if x > 0,

h(x) = 0 if x = 0,

if x < 0,

x cos x

is continuous at x = 0.
Solution. h is defined at x = 0 with h(0) = 0. Also
  
lim− h(x) = lim− x cos x = lim− x lim− cos x = (0)(1) = 0
x→0 x→0 x→0 x→0

and
lim h(x) = lim+ sin x = 0
x→0+ x→0
so that lim h(x) exists and
x→0
lim h(x) = 0.
x→0
Since h(0) = lim h(x), h is continuous at x = 0 since h satisfies all three conditions of continuity at a point.
x→0

4. Let 

 ax − b if x ≤ 1,

f (x) = 3x if 1 < x < 2,


 2
bx − a if x ≥ 2.
Find the value(s) of a and b so that f is continuous at x = 1 but not continuous at x = 2.
Solution. If f is to be continuous at x = 1, lim− f (x) = lim+ f (x) = f (1). Now f (1) = a(1) − b = a − b,
x→1 x→1

lim f (x) = lim− (ax − b) = a(1) − b = a − b


x→1− x→1

and
lim f (x) = lim+ 3x = 3(1) = 3.
x→1+ x→1

Thus a−b = 3. Since f (2) = b(2 )−a = 4b−a and f is not continuous at x = 2, we require 4b−a = f (2) , lim f (x).
2
x→2
But
lim f (x) = lim− 3x = 3(2) = 6
x→2− x→2
whilst
lim f (x) = lim+ (bx2 − a) = 4b − a.
x→2+ x→2
Thus 4b − a , 6 so that
a , 4b − 6 = 4(a − 3) − 6.
Hence a , 4a − 12 − 6 and thus 3a , 18. Consequently, a , 6. Hence f is continuous at x = 1 but not continuous
at x = 2 if and only if b = a − 3 and a , 6.
MATH1036 1st Semester Calculus Lecture Manual 2017 25

Tutorial 2.1.1. 1. Find the values of a and b which make the function


 x−1 if x ≤ −2,

f (x) = ax2 + c if − 2 < x < 1,

x+1

if x ≥ 1,

continuous at x = −2 and x = 1.

2.2 Left and Right Continuity at a Point


From the examples in the previous section it is apparent that we can speak of continuity from the left or right at a
point. We thus have

A function f is continuous from the right at a if lim+ f (x) = f (a).


x→a

and
A function f is continuous from the left at a if lim− f (x) = f (a).
x→a

Worked Example 2.2.1. 1. Let


 |x| + x

if x , 0,
f (x) = 2x
0 if x = 0.

Determine the right and left continuity of f at x = 0.


Solution. f (0) = 0 whilst
|x| + x −x + x
lim f (x) = lim− = lim− = lim− 0 = 0
x→0− x→0 2x x→0 2x x→0
and
|x| + x x+x
lim+ f (x) = lim+ = lim+ = lim− 1 = 1.
x→0 x→0 2x x→0 2x x→0
Since f (0) = 0 = lim− f (x), f is continuous from the left at x = 0. Since f (0) = 0 , 1 = lim+ f (x), f is not
x→0 x→0
continuous from the right at x = 0.
2. Show that at each integer n, the greatest integer function f (x) = bxc is right-continuous at n, but discontinuous
from the left at n.
Solution. Since
lim f (x) = lim+ bxc = n = f (n),
x→n+ x→n
bnc is continuous from the right at n. However,
lim f (x) = lim− bxc = n − 1 , n = f (n),
x→n− x→n

so that bxc is not continuous from the left at n.

It can be observed that

f is continuous at a if and only if f is right-continuous and left-continuous at a.

Tutorial 2.2.1. 1. Consider the function



 bxc if x , 0,
f (x) = x
−1 if x = 0.
Investigate continuity from the left and the right at x = 0, x = π and x = 1.
26 1st Semester Calculus Lecture Manual 2017 MATH1036

2.3 Continuity on an Interval


We can also now consider continuity on an interval.

f is continuous on an interval if and only if f is continuous at each point in the interval.

In particular,
1. f is continuous on an open interval (a, b) if and only if f is continuous at each point a < x < b.
2. f is continuous on the interval [a, b) if and only if f is continuous on the open interval (a, b) and f is right-
continuous at a.
3. f is continuous on the interval (a, b] if and only if f is continuous on the open interval (a, b) and f is left-
continuous at b.
4. f is continuous on the closed interval [a, b] if and only f is continuous on the open interval (a, b) and f is
right-continuous at a and left-continuous at b.
5. f is a continuous function if and only if f is continuous at each point of its domain.
6. f is continuous everywhere if and only if f is continuous at each point in R, i. e., f is continuous on the open
interval (−∞, ∞).

Worked Example 2.3.1. Show that f (x) = x − 4 is continuous on the interval [4, 8].
Solution. For 4 < a < 8, using Theorem 1.1.11,
√ q √
lim f (x) = lim x − 4 = lim x − 4 = a − 4 = f (a).
x→a x→a x→a

Also √ q √ √
lim+ f (x) = lim+ x−4= lim+ x − 4 = 4−4= 0 = 0 = f (4)
x→4 x→4 x→4

and √ q √ √
lim− f (x) = lim− x−4= lim x − 4 = 8−4= 4 = 2 = f (8).
x→8 x→8 x→8−

Thus f is continuous at each x ∈ (4, 8), continuous from the right at x = 4 and continuous from the left at x = 8.
Hence f is continuous on the closed interval [4, 8].

2.4 Theorems on Continuity


By the definition of continuity, f is continuous at a provided that lim f (x) = f (a). Thus where direct substitution
x→a
applies in calculating a limit of a function, the function is necessarily continuous at the given point. We have the
following with regards to the algebra of functions that are continuous at a given point a.

Theorem 2.1. If f and g are continuous at a and c ∈ R, then


1. the sum f + g,
2. the difference f − g,
3. the product f g,
f
4. the quotient if g(a) , 0 and
g
5. the scalar multiple c f
are functions that are also continuous at a.
MATH1036 1st Semester Calculus Lecture Manual 2017 27

Theorem 2.2. The following functions are continuous on their domains.

1. Polynomials p(x) = an xn + an−1 xn−1 + · · · + a2 x2 + a1 x + a0 , ai ∈ R, n ∈ N.

p(x)
2. Rational functions , p and q , 0 polynomials.
q(x)

3. Root functions.

4. The trigonometric functions sin x, cos x, tan x, cosec x, sec x and cot x.

5. The exponential functions a x and the natural exponential function e x .

6. The logarithmic functions loga x and the natural logarithmic function ln x.

7. The absolute value function |x|.

Worked Example 2.4.1. 1. Let




 4x if x ≤ −1,

h(x) = cx + d if − 1 < x ≤ 2,

if x > 2.

−5x

Determine the values of c and d so that h is a continuous function. For these values, sketch h.

Solution. By Theorem 2.2, f is continuous on (−∞, −1], (−1, 2] and (2, ∞]. For continuity at x = −1 and x = 2
we require lim − h(x) = lim + h(x) and lim− h(x) = lim+ h(x). Since
x→−1 x→−1 x→2 x→2

lim h(x) = lim − 4x = 4(−1) = −4


x→−1− x→−1

and
lim h(x) = lim + (cx + d) = c(−1) + d = −c + d,
x→−1+ x→−1

we have = −c + d = −4. Since


lim h(x) = lim− (cx + d) = 2c + d
x→2− x→2

and
lim h(x) = lim+ (−5x) = −5(2) = −10,
x→2+ x→2

we have 2c + d = −10.
Thus solving simultaneously
−c + d = −4
´
⇒ 3c = −6 ⇒ c = −2.
2c + d = −10
Thus d = −6. Hence h is a continuous function if c = −2 and d = −6 and then h is the function

4x
 if x ≤ −1,

h(x) = −2x − 6 if − 1 < x ≤ 2,

if x > 2.

−5x

28 1st Semester Calculus Lecture Manual 2017 MATH1036

2 . Describe the intervals on which the function f (x) = tan x is continuous.


Solution.

x
−π π 0 π π 3π
− 2π
2 2 2
−1

y = tan x

π
f has vertical asymptotes for x = 2 + nπ, n ∈ Z. This means that for each n ∈ Z, lim
π
f (x) does not exist. Hence
x→ 2 +nπ

π sin x
f (x) = tan x is discontinuous at + nπ for each n ∈ Z. At all other points f is continuous since f (x) =
2 is the
cos x
quotient of continuous functions. Thus f is continuous in each open interval in its domain, i. e., on the intervals

3π π  π π  π 3π
Å ã Å ã
..., − ,− , − , , , ,...
2 2 2 2 2 2

Theorem 2.3 (Composite functions). If g is continuous at a and f is continuous at g(a), then the composite
function f ◦ g is continuous at a and

lim f (g(x)) = f (lim g(x)) = f (g(a)).


x→a x→a

The next example illustrates the continuity of the composite of functions together with the above theorems on
continuity.

Worked Example 2.4.2. Use the above theorems on continuity to evaluate the following limits.
1. lim |1 − 2x − 3x2 |.
x→3

Solution.

lim |1 − 2x − 3x2 | = | lim(1 − 2x − 3x2 )| ∵ |x| is continuous on R


x→3 x→3
= |1 − 2(3) − 3(3)2 | ∵ 1 − 2x − 3x2 is a polynomial, hence continuous at 3
= |1 − 6 − 27|
= | − 32|
= 32.
MATH1036 1st Semester Calculus Lecture Manual 2017 29

x2 − 4
2. lim .
x→2 x−2
Solution.

(x − 2)(x + 2)

x2 − 4
lim = lim
x→2 x−2 x→2 x−2

= lim x + 2
x→2
q
= lim(x + 2) ∵ the root function is continuous at all positive values
x→2

= 4
= 2.

3. limπ sin2 x.
x→ 3

Solution.
lim sin2 x = limπ (sin x)2
x→ π3 x→ 3
Å ã2
= limπ sin x ∵ square function is continuous at every point
x→ 3
 π 2
= sin ∵ sin x is continuous at every point
3
Ç √ å2
3
=
2
3
= .
4
4. limπ ecos 2x .
x→ 6

Solution.
lim cos 2x
π
lim ecos 2x = ex→ 6 ∵ the natural exponential function e x is continuous on R
x→ π6
Å ã
cos lim 2x
x→ π
=e 6 ∵ the cosine function is continuous on R
cos( 2π
6 )
=e
1
= e2

= e.
√ √3
5. lim ln( 2x + 3 4 − 3x).
x→0
Solution.
√ √3 √ √3
lim ln( 2x + 3 4 − 3x) = lim[ln 2x + 3 + ln 4 − 3x]
x→0 x→0
ï ò
1 1
= lim ln(2x + 3) + ln(4 − 3x)
x→0 2 3
1 1
= lim ln(2x + 3) + lim ln(4 − 3x) (sum of continuous functions)
2 x→0 3 x→0
1 1
= ln(lim[2x + 3]) + ln(lim[4 − 3x]) (ln is continuous on the positive reals)
2 x→0 3 x→0
1 1
= ln 3 + ln 4.
2 3
30 1st Semester Calculus Lecture Manual 2017 MATH1036

Extra example (not in study guide) Describe the intervals on which the following functions are continuous.

sin 1 if x , 0
(a) g(x) = x
0 if x = 0.
1
Solution. We showed in Worked Example 1.1.4.1 that lim sin does not exist. Thus g is not continuous at x = 0.
x→0 x
However, g is continuous on (−∞, 0) and (0, ∞).

 x sin 1 if x , 0,
(b) h(x) = x
0 if x = 0.
1
Solution. In Worked Example 1.5.1.3 we used the Sandwich Theorem to show that lim x sin = 0. Since
x x→0
h(0) = 0 = lim h(x), h is continuous at x = 0. Thus h is everywhere continuous, i. e., h is continuous on
x→0
R = (−∞, ∞).
The following important theorem on continuous functions tells us that the graph of a continuous function cannot
jump from one side of a horizontal line y = N to the other without intersecting the line at least once.

f (b)

y=N
f (a)

a c b

Theorem 2.4 (Intermediate Value Theorem (IVT)). Suppose that f is continuous on the closed interval [a, b] and
let N be a number between f (a) and f (b), where f (a) , f (b). Then there exists a number c in the open interval
(a, b) such that f (c) = N.

The above theorem gives a bisection method for locating roots or zeros of a continouus function within an interval.
(IVT gives an existence method for finding values of x within an interval for which f (x) = 0). However, it does
not specify what the root is if it exists. As a special case of the IVT we have:

If f is continous on the closed interval [a, b] and either f (a) < 0 < f (b) or f (b) < 0 < f (a), then there is at least
one number c in the open interval (a, b) such that f (c) = 0.

Therefore, if the sign of a continuous function f changes within a closed interval [a, b], then we are guarenteed
the existence of at least one root of f within the open interval (a, b).

The IVT is of course not true in general for discontinuous functions as the following illustrates.

f (b)

y=N
f (a)

a b
MATH1036 1st Semester Calculus Lecture Manual 2017 31

In the above f is discontinuous on [a, b] and there is no c ∈ (a, b) such that f (c) = N. Thus continuity is essential
in the IVT.

Worked Example 2.4.3. 1. Show that the equation cos x = x has a solution in the closed interval [0, π2 ].
Solution. Let f (x) = cos x − x. Then f is a difference of continous functions on R, and hence f is continuous on
R. In particular, f is continuous on [0, π2 ]. Furthermore,

f (0) = cos 0 − 0 = 1

and π π π π π
f = cos − =0− =− .
2 2 2 2 2
π
Thus f < 0 < f (0). Hence by the IVT there exists c ∈ (0, π2 ) such that f (c) = cos c − c = 0. Hence c is a
2
solution to cos x = x with c ∈ (0, π2 ),
2. The equation x3 − 4x2 + 2x + 2 = 0 has three solutions. Find intervals between successive integers that contain
the solutions.
Solution. Here we wish to locate the solutions of g(x) = 0 where g(x) = x3 − 4x2 + 2x + 2. We only need to
evaluate g at various integers until we find successive integers for which the sign of g(x) changes. Then the IVT
will guarantee a solution in the interval of the sign change of g.
Since g(−1) = −5 and g(0) = 2, g(−1) < 0 < g(0) so that g changes sign in the closed interval [−1, 0]. Hence by
the IVT there is c1 ∈ (−1, 0) such that g(c1 ) = 0. Also g(1) = 1 and g(2) = −2 and so g(1) > 0 > g(2) implies
that g changes sign in [1, 2]. By the IVT there exists c2 ∈ (−1, 2) such that g(c2 ) = 0. Finally, g(3) = −1 whilst
g(4) = 10. Thus g(3) < 0 < g(4) so that g changes sign in [3, 4]. Hence by the IVT there exists c3 ∈ (3, 4) such
that g(c3 ) = 0. Hence g has its zeros in the intervals (−1, 0), (1, 2) and (3, 4).

Tutorial 2.4.1. 1. Thomas Exercises 11th ed. 2.6 pp. 129–130: 13–20, 23–33, 40, 47–50, 53–57, 59 or 12th ed.
2.5 pp. 101–103: 13–20, 23–35, 47, 55–58, 61–64, 65, 67.
2. Thomas Chapter 2 Additional and Advanced Exercises 11th ed. p. 142: 19 or 12th ed. p. 120: 19.
3. The equation x3 − 12x + 8 = 0 has three solutions. Find intervals between successive integers that contain these
solutions.

2.5 Discontinuities
If one of the three conditions for continuity of a function f (the slot-machine test) fails at a given point a, we know
then that f is discontinuous at a. The following are the various types of discontinuities.
1. Removable discontinuity
If lim f (x) exists but f fails to be continuous at x = a because either
x→a
(a) f (a) is not defined or
(b) f (a) is defined, but a number different from lim f (x),
x→a
then f is said to have a removable discontinuity at x = a.

a a
32 1st Semester Calculus Lecture Manual 2017 MATH1036

This discontinuity at a can be removed by redefining f (a) = lim f (x).


x→a

2. Jump discontinuity

If lim+ f (x) and lim− f (x) exist but f is not continuous at a because lim f (x) does not exist, then lim− f (x) ,
x→a x→a x→a x→a
lim f (x) and f is said to have a jump discontinuity at x = a. The function jumps from one value to another one.
x→a+

3. Infinite discontinuity

If f is not continuous at a because lim− f (x) = ±∞ or lim+ f (x) = ±∞, then f is said to have an infinite disconti-
x→a x→a
nuity at a.

Below are some illustrations for infinite discontinuities.

a a a

a a a
MATH1036 1st Semester Calculus Lecture Manual 2017 33

Worked Example 2.5.1. Let  3



 x if x ≤ −1,

x2 − 2 if − 1 < x < 0,






if 0 ≤ x < 2,

3 − x



f (x) = 4x − 1 if 2 ≤ x < 4,
 x−1



 15



 if 4 < x < 7,



 7−x
5x + 2 if x ≥ 7.

Fill out the following table and then discuss the continuity of f at a. At each point of discontinuity, classify the
discontinuity. List the intervals on which f is continuous.

a f (a) lim f (x) lim f (x) lim f (x) Continuos or Discontinuity


x→a− x→a+ x→a
discontinuous at a? type

−1 −1 −1 −1 −1 Continuous None

0 3 −2 3 Does not Discontinuous Jump


exist

2 7 1 7 Does not Discontinuous Jump


exist

4 Does not 5 5 5 Discontinuous Removable


exist

7 37 ∞ 37 Does not Discontinuous Infinite


Does not exist
exist

Intervals on which f is continuous: (−∞, 0), (0, 2), (2, 4), (4, 7) and (7, ∞).

Tutorial 2.5.1. 1. Thomas Exercises 11th ed. 2.6 p. 130: 51, 52 or 12th ed. 2.5 p. 102: 59, 60.
2. Thomas Chapter 2 Practice Exercises 11th ed. p. 139: 1, 2 or 12th ed. p. 117: 1, 2. Also indicate the type of
discontinuities.
Chapter 3

Differentiation

LEARNING OUTCOMES:
On completion of this chapter you should (tick the checkbox when you have mastered the skill)

 1. know the limit definition of the derivative of a function at a point,


 2. be able to find the derivative of a function using the definition (first principles),
 3. understand and be able to explain the concept of differentiability on an interval,
 4. be able to interpret and find the derivative at a point as the slope of a tangent line at that point,
 5. be able to interpret graphically non-differentiability of functions at a point,
 6. understand the relationship between differentiable and continuous functions,
 7. know how to find algebraically the derivative funtion f from the given formula of f ,
0

 8. know how to find and sketch the derivative funtion f from the graph of f ,
0

 9. know the proofs of the Derivative Rules,


 10. know and be able to calculate the derivative of a function using the Derivative Rules,
 11. know the derivatives of the trigonometric functions and know how to prove these derivatives,
 12. know the derivatives of the exponential functions a and the logarithmic functions log x,
x
a

 13. know the derivative of the natural exponential function e and the natural logarithmic function ln x,
x

 14. know and be able to apply the Chain Rule in differentiating composite functions,
 15. know how to approximate function values using linearization,
 16. know the concept of and know how to evaluate differentials,
 17. know how to solve the examinable tutorial problems and the worked out problems from these notes on
this chapter.

3.1 First Principles


In evaluating limits, functional notation plays a convenient and important role.

1
Worked Example 3.1.1. Let f (x) = . Evaluate
(x + 1)2

f (5 + h) − f (5)
lim .
h→0 h

34
MATH1036 1st Semester Calculus Lecture Manual 2017 35

Solution.

1 1

f (5 + h) − f (5) [(5 + h) + 1]2 (5 + 1)2
lim = lim
h→0 h h→0 h
1 1

(6 + h) 2 36
= lim
h→0 h
36 − (6 + h)2
= lim
h→0 36h(6 + h)2

36 − (36 + 12h + h2 )
= lim
h→0 36h(6 + h)2
−12h − h2
= lim
h→0 36h(6 + h)2
−12 − h
= lim ( we may cancel h)
h→0 36(6 + h)2
−12
= (continuous function)
36(6)2
1
=− .
108

The derivative of the function f at a, denoted f 0 (a) is defined as

f (a + h) − f (a)
f 0 (a) = lim
h→0 h

or equivalently,

f (x) − f (a)
f 0 (a) = lim
x→a x−a

provided the limit exists. The above limit calculations provide the derivative of the function f at a by first
principles.

Worked Example 3.1.2. Find the derivative of the following by first principles.
1
1. f (5) if f (x) = .
(x + 1)2

Solution. From first principles,


f (5 + h) − f (5) 1
f 0 (5) = lim =− ,
h→0 h 108
as calculated in Worked Example 3.1.1.
36 1st Semester Calculus Lecture Manual 2017 MATH1036


2. g0 (4) if g(x) = x2 + 9.
Solution.
g(x) − g(4)
g0 (4) = lim
x→4 x−4
√ √
x2 + 9 − 16 + 9
= lim
x→4 x−4

x2 + 9 − 5
= lim
x→4 x−4
Ç√ åÇ √
x2 + 9 − 5 x2 + 9 + 5
å
= lim √
x→4 x−4 x2 + 9 + 5
(x2 + 9) − 25
= lim √
x→4 (x − 4)( x2 + 9 + 5)

x2 − 16
= lim √
x→4 (x − 4)( x2 + 9 + 5)

(x − 4)(x + 4)
= lim √
x→4 (x − 4)( x2 + 9 + 5)

x+4
= lim √
x→4 x +9+5
2

4+4 8
= √ =
16 + 9 + 5 10
4
= .
5

If f 0 (a) exists we say that the function f is differentiable at a or f has a derivative at a.

Now given a function f and the point a we define the left-hand derivative of f at a as
f (a + h) − f (a)
f−0 (a) = lim−
h→0 h
and the right-hand derivative of f at a as
f (a + h) − f (a)
f+0 (a) = lim+ .
h→0 h
Equivalently,
f (x) − f (a) f (x) − f (a)
f−0 (a) = lim− and f+0 (a) = lim+ .
x→a x−a x→a x−a
Then certainly

f is differentiable at a provided that f−0 (a) and f+0 (a) both exist and f−0 (a) = f+0 (a)

so that f 0 (a) = f−0 (a) = f+0 (a).


Of course f is not differentiable at a provided that f−0 (a) , f+0 (a).

Worked Example 3.1.3. 1. Show that f (x) = |x| is not differentiable at x = 0.


Solution.
f (0 + h) − f (0) |h| − |0| |h| (−h)
f−0 (0) = lim− = lim− = lim− = lim− = lim− (−1) = −1
h→0 h h→0 h h→0 h h→0 h h→0

whilst
f (0 + h) − f (0) |h| − |0| |h| h
f+0 (0) = lim+ = lim+ = lim− = lim+ = lim+ 1 = 1.
h→0 h h→0 h h→0 + h h→0 h h→0

Thus f−0 (0) , f+0 (0) so that f 0 (a) does not exist. Hence, f is not differentiable at a.
MATH1036 1st Semester Calculus Lecture Manual 2017 37

2. Is the function
if x < 1,
(
4x
g(x) =
2x + 2
2
if x ≥ 1,
differentiable at x = 1?
Solution.
g(x) − g(1)
g0+ (1) = lim+
x→1 x−1
(2x2 + 2) − 4
= lim+
x→1 x−1
2
2x − 2
= lim+
x→1 x−1
2(x2 − 1)
= lim+
x→1 x−1
(x + 1)(x − 1)
= 2 lim+
x→1 x−1
= 2 lim+ (x + 1)
x→1
= 2(2) = 4

whilst
g(x) − g(1) 4x − 4 4(x − 1)
g0− (1) = lim− = lim− = lim− = lim− 4 = 4,
x→1 x−1 x→1 x − 1 x→1 x−1 x→1

so that g0− (1) = 4 = g0+ (1). Hence g0 (1) does exist and g0 (1) = 4.

We say that f is differentiable on an open interval (a, b) if f is differentiable for all c ∈ (a, b). In like manner,
we refer to functions that are differentiable on intervals of the form (a, ∞), (−∞, b) or (−∞, ∞) = R.
A function f is differentiable on the closed interval [a, b] if f is differentiable on the open interval (a, b) and the
one-sided limits f+0 (a) and f−0 (b) both exist.
Differentiability on an interval of the form [a, b), [a, ∞), (a, b] or (−∞, b] is defined in the obvious way, using the
one-sided limit at the endpoint.
A function f is called a differentiable function if f is differentiable at each point of its domain.

Tutorial 3.1.1. 1. Thomas Exercises 11th ed. 3.1 pp. 152–153 or 12th ed. 3.2 p. 131: 1, 2, 4, 6, 12.
2. Find f 0 (a) if it exists, where
(
x3 if x ≤ 1
(a) f (x) = and a = 1.
cx + b if x > 1
(
3x2 if x ≤ 1
(b) f (x) = and a = 1.
2x + 1 if x > 1
3

3. Thomas Chapter 2 Practice Exercises 11th ed. p. 139: 13–16 or 12th ed. p. 118: 13–16. Each limit represents
the limit of some function f at some number a. State such an f and a in each case.

3.2 Tangents
The derivative of a function at a point also has a geometric interpretation.

3.2.1 Non-vertical tangents


Consider the continuous function y = f (x) with point P(a, f (a)) and arbitrary point Q(x, f (x)) approaching P
along f as x → a.
38 1st Semester Calculus Lecture Manual 2017 MATH1036

Q
Q
P
Q

a x

The slope of the secant line PQ is


f (x) − f (a)
mPQ = .
x−a
As x → a, we see geometrically that the point Q tends to the point P and the secant line PQ rotates around P.
Intuitively it seems as though PQ approaches a limiting line as x → a and this limiting line must be the tangent
line to f at x = a. If the function f is differentiable at x = a, then
f (x) − f (a)
m = lim mPQ = lim = f 0 (a).
x→a x→a x−a
In other words, the slopes of the secant lines PQ, mPQ , tend to a limiting slope m which is the derivative of the
function f at P corresponding to the slope of the tangent line of f at P. Thus the tangent line to the curve y = f (x)
at the point P(a, f (a)) is the line through P with slope
f (x) − f (a)
m = lim = f 0 (a).
x→a x−a
The equation of the tangent line to the graph of f at x = a is then

y = f (a) + f 0 (a)(x − a).

Worked Example 3.2.1. Find the equation of the tangent line to the graph of f (x) = x2 at x = −3.
Solution. For x = −3, f (−3) = (−3)2 = 9, so that we seek the equation of the tangent line at the point P(−3, 9) of
the graph of f . The equation of the tangent line is thus

y = f (−3) + f 0 (−3)(x − (−3)),

i. e.,
y = 9 + f 0 (−3)(x + 3).
It remains to find f 0 (−3). Now

f (x) − f (−3) x2 − 9 (x − 3)(x + 3)


f 0 (−3) = lim = lim = lim = lim (x − 3) = −3 − 3 = −6.
x→−3 x − (−3) x→−3 x+3 x→−3 x+3 x→−3

Hence, the equation of the tangent line is

y = 9 + f 0 (3)(x + 3) = 9 + (−6)(x + 3) = 9 − 6x − 18 = −6x − 9.

Tutorial 3.2.1. 1. Thomas Exercises 11th ed. 2.7 pp. 136–137: 5, 8, 11, 16, 17, 31, 32 or 12th ed. 3.1 p. 125: 5,
8, 11, 16, 17, 31, 33, 34.

3.2.2 Vertical tangents


The function f has a vertical tangent at a provided that f is continuous at a, differentiable near a, i. e., f 0 (x) exists
for x , a with x close to a, and lim | f 0 (x)| = ∞.
x→a
MATH1036 1st Semester Calculus Lecture Manual 2017 39

Vertical tangent Vertical tangent

Vertical tangents at endpoints

cusp

a a a b

The definition for vertical tangents may be modified to include vertical tangent lines at an endpoint of the domain
of the function. Thus, if f is continuous on [a, b] but is undefined outside [a, b], then f has a vertical tangent at
x = a if lim+ | f (x)| = ∞ and f has a vertical tangent at x = b if lim− | f (x)| = ∞.
x→a x→b
√3
Worked Example 3.2.2. Let f (x) = x. Show that f has a vertical tangent at x = 0.
Solution. Certainly f is continuous at x = 0 since f (0) = 0 = lim f (x). Now for each a , 0,
x→0

f (x) − f a)
f 0 (a) = lim
x→a x−a
√3 √
x− 3a
= lim
x→a x−a
√3 √
x− 3a
= lim √ 3 √ 3
x→a 3
x − 3a
√3 √
x− 3a
= lim √ √   √3 2 √3 √3
Ä  √ 2 ä (difference of two cubes)
x→a 3
x− 3a x + x a+ 3a
1
= lim  √ 2 √ √ √ 
x→a 3 x + 3 x 3 a + 3 a 2

1
=  √ 2 √ √  √ 2
3
a + a 3a+ 3a
3

1
=  √ 2
3 3a
1
= 2 .
3a 3
1
Thus for each x , 0, f 0 (x) = 2 . Then
3x 3
1 1
lim | f 0 (x)| = lim 2 = lim √3 2 = ∞
x→0 x→0 3x 3 x→0 3 x
by Theorem 1.1 and Theorem 1.4. Hence f has a vertical tangent at x = 0.

Tutorial 3.2.2. 1. Thomas Exercises 11th ed. 2.7 pp. 137–138: 33, 39, 44 or 12th ed. 3.1 p. 126: 35, 41, 46.

3.3 Differentiability vs. Continuity


Since both continuity and differentiability involve limits, we can investigate the relationship between these two
concepts.

1. A function can be continuous at a given point without being differentiable there.


40 1st Semester Calculus Lecture Manual 2017 MATH1036

Worked Example 3.3.1. Let


(
x2 if x ≤ 1,
f (x) =
x if x > 1.
Investigate the continuity and differentiability of f at x = 1.
Solution.
Continuity: f (1) = (1)2 = 1,
lim f (x) = lim− x2 = (1)2 = 1,
x→1− x→1

whilst
lim f (x) = lim+ x2 = 1.
x→1+ x→1

Hence lim− f (x) = lim+ f (x) = f (1), so that f is continuous at x = 1.


x→1 x→1

Differentiability:

f (1 + h) − f (1) (1 + h)2 − 1 h2 + 2h
f−0 (1) = lim− = lim− = lim− = lim− (h + 2) = 2
h→0 h h→0 h h→0 h h→0

whilst
f (1 + h) − f (1) (1 + h) − 1 h
f+0 (1) = lim+ = lim+ = lim− = lim− 1 = 1.
h→0 h h→0 h h→0 h h→0

Thus f−0 (1) , f+0 (1) so that f (1) does not exist.
Hence f is not differentiable at x = 1 even though f is continuous at x = 1.

The above example shows that

CONTINUOUS ; DIFFERENTIABLE

2. Every function that is differentiable at a point is continuous there.

Theorem 3.1. If a function f is differentiable at x = a, then f is continuous at x = a.

Proof. Suppose that f is differentiable at x = a. We must show that f is continuous at x = a, i .e., we need to
show that lim f (x) = f (a).
x→a
If x is in the domain of f and x , a, then f (x) may be written as
Å ã
f (x) − f (a)
f (x) = f (a) + (x − a).
x−a

Then
ï Å ã ò
f x) − f (a)
lim f (x) = lim f (a) + (x − a)
x→a x→a x−a
ïÅ ã ò
f x) − f (a)
= lim f (a) + lim (x − a)
x→a x→a x−a
Å ã
f x) − f (a)
= f (a) + lim lim(x − a)
x→a x−a x→a

= f (a) + f (a) lim(x − a)


0
x→a
= f (a) + f 0 (a) · 0
= f (a).

Hence f is continuous at x = a. 
MATH1036 1st Semester Calculus Lecture Manual 2017 41

The above theorem shows that

DIFFERENTIABLE ⇒ CONTINUOUS

or contrapositively,
NOT CONTINUOUS ⇒ NOT DIFFERENTIABLE

Worked Example 3.3.2. Consider the greatest integer funtion f (x) = bxc at x = 0. Discuss the continuity and
differentiability of f at x = 0.
Solution. By Worked Example 1.1.3.5, f is not continuous at 0. By the contrapositive of Theorem 3.1, f is not
differentiable at x = 0.

Cases in which a function f fails to be differentiable at x = a are:

1. f is discontinuous at a. Graphically, if we observe a removable, jump or an infinite discontinuity at a.

2. f is continuous at a and the graph of f is pointed (e. g. |x|) or makes a sharp turn or corner (e.g. Worked
Example 3.3.1),

3. f has a vertical tangent at a (e.g. Worked Example 3.2.2).

3.4 The derivative as a function


We observe that if for a given function f we let X = {a ∈ R : f 0 (a) exists}, then we can associate with each x ∈ X
the number f 0 (x) and we obtain a function f 0 with domain X:

f 0 : X→R
x 7→ f 0 (x).

The function f 0 is called the derivative of f and, by definition, the values of f 0 at each x are given by the following
(or an appropriate one-sided limit)
f (x + h) − f (x)
f 0 (x) = lim .
h→0 h

Worked Example 3.4.1. Let f (x) = |x|. Find the derivative function f 0 of f .
Solution. The domain of f 0 is the set X = {a ∈ R : f 0 (a) exists}. We know that f is not differentiable at 0 but
differentible at all other points. Thus X = R \ {0} = (−∞, 0) ∪ (0, ∞). Indeed, for each x < 0,

f (x + h) − f (x) |x + h| − |x| −(x + h) − (−x) −h


f 0 (x) = lim = lim = lim = lim = lim(−1) = −1,
h→0 h h→0 h h→0 h h→0 h h→0

and for x > 0,

f (x + h) − f (x) |x + h| − |x| (x + h) − (x) h


f 0 (x) = lim = lim = lim = lim = lim 1 = 1.
h→0 h h→0 h h→0 h h→0 h h→0

Hence the derivative function is defined by

if x > 0,
(
1
f 0 (x) =
−1 if x < 0.

The graphs of f and the derivative function f 0 are given below.


42 1st Semester Calculus Lecture Manual 2017 MATH1036

f (x) = |x| f 0 (x)

−1

Worked Example 3.4.2. Let g(x) = |x2 − 4|. Sketch the graph of g. Use the graph of g to sketch the graph of g0 .

Solution.

g(x) = |x2 − 4|

−2 2

g0 (x) < 0 for x < −2. g0 (x) does not exist at x = 2 (sharp corner). g0 (x) > 0 for −2 < x < 0, whereas g has a
horizontal tangent at x = 0, i. e., g0 (0) = 0. For 0 < x < 2, g0 (x) < 0. At x = 2, g0 (x) does not exist (sharp corner).
For x > 2, g0 (x) > 0.

g0 4

−2 2

−4

Tutorial 3.4.1. 1. Thomas Exercises 11th ed. 3.1 pp. 153–154: 27–32 or 12th ed. 3.2 p. 132: 27–32.
MATH1036 1st Semester Calculus Lecture Manual 2017 43

3.5 Derivative Rules


It is quite cumbersome on each occasion to calculate the derivative of a differentiable function at a given point
using first principles. We will now develop formulas that give the derivatives of known functions that we will
encounter. The proofs of these results use the definition of the derivative and are examinable.

The symbols
df
, D x f or f 0
dx
are commonly used to denote the first derivative function.

Theorem 3.2 (Derivative of constants). If c ∈ R and f (x) = c for each x, then f 0 (x) = 0.

Proof.
f (x + h) − f (x) c−c
f (x) = lim = lim = lim 0 = 0. 
h→0 h h→0 h h→0

The above theorem says that the derivative of constant functions are zero, i. e.,

d
∀c ∈ R (c) = 0.
dx

Theorem 3.3 (Constant Multiple Rule, factorization of constants). If c ∈ R and f is differentiable, then c f is
differentiable and (c f )0 (x) = c f 0 (x).

Proof.

(c f )(x + h) − (c f )(x) c( f (x + h)) − c( f (x)) f (x + h) − f (x)


(c f )0 (x) = lim = lim = c lim = c f 0 (x). 
h→0 h h→0 h h→0 h
Thus
d d
∀c ∈ R [c f (x)] = c f (x).
dx dx

Theorem 3.4 (Power Rule for positive integer power functions). If f (x) = xn where n ∈ Z+ , then f 0 (x) = nxn−1 .

Proof. We must show that if f (x) = xn where n is a positive integer, then f 0 (a) = nan−1 for every a. We will use
the factorization formula

xn − an = (x − a)(xn−1 + axn−2 + a2 xn−3 + a3 xn−4 + · · · + an−2 x + an−1 ).

We then have
f (x) − f (a)
f 0 (a) = lim
x→a x−a
x n − an
= lim
x→a x − a
(x − a)(xn−1 + axn−2 + a2 xn−3 + a3 xn−4 + · · · + an−2 x + an−1 )
= lim
x→a x−a
= lim(xn−1 + axn−2 + a2 xn−3 + a3 xn−4 + · · · + an−2 x + an−1 ) (polynomial with n terms)
x→a
=a n−1
+ aan−2 + a2 an−3 + a3 an−4 + · · · + an−2 a + an−1 ) (polynomials are continuous)
=a n−1
+a n−1
+a
n−1
+a n−1
+ ··· + a
n−1
+a n−1
) n( times)
= na n−1
. 
44 1st Semester Calculus Lecture Manual 2017 MATH1036

Theorem 3.4 says

d n
∀n ∈ N x = nxn−1 .
dx

d d d d 1 d √
Worked Example 3.5.1. 1. π = 0, e = 0, (−4) = 0, = 0 and 2 = 0.
dx dx dx dx 2 dx
d d
2. (3x15 ) = 3 x15 = 3(15)x15−1 = 45x14 .
dx dx
Theorem 3.5 (Sum Rule). If f and g are differentiable functions, then f + g is differentiable and

( f + g)0 (x) = f 0 (x) + g0 (x).

Proof.
( f + g)(x + h) − ( f + g)(x)
( f + g)0 (x) = lim
h→0 h
[ f (x + h) + g(x + h)] − [ f (x) + g(x)]
= lim
h→0 h
[ f (x + h) − f (x)] + [g(x + h) − g(x)]
= lim
h→0 h
f (x + h) − f (x) g(x + h) − g(x)
Å ã
= lim +
h→0 h h
f (x + h) − f (x) g(x + h) − g(x)
= lim + lim (Sum Law for limits)
h→0 h h→0 h
= f (x) + g (x).
0 0


Hence
d d d
[ f (x) + g(x)] = f (x) + g(x).
dx dx dx

Theorem 3.6 (Difference Rule). If f and g are differentiable functions, then f − g is differentiable and

( f − g)0 (x) = f 0 (x) − g0 (x).

Proof. We can apply the Sum Rule and the Constant Multiple Rule.

( f − g)0 (x) = ( f + [−g])0 (x) = f 0 (x) + (−g)0 (x) = f 0 (x) − g0 (x). 

Hence
d d d
[ f (x) − g(x)] = f (x) − g(x).
dx dx dx

Worked Example 3.5.2. Find the first derivatives of the following functions.
1. x3 − 4x2 + 7.

Solution.
d 3 d 3 d 2 d
(x − 4x2 + 7) = x − 4x + 7
dx dx dx dx
d
= 3x2 − 4 x2 + 0
dx
= 3x2 − 4(2x)
= 3x2 − 8x.
MATH1036 1st Semester Calculus Lecture Manual 2017 45

2. 3t2 (t2 + 2).

Solution.

d 2 2 d
3t (t + 2) = 3 t2 (t2 + 2)
dt dt
d
= 3 (t4 + 2t2 )
dt
= 3(4t3 + 2(2t))
= 12t3 + 12t.

Å ãÅ ã
d 2 2 d 2 d 2
Note that (3t (t + 2)) , (3t ) (t + 2) . In general
dt dt dt

( f g)0 , f 0 g0 .

THE DERIVATIVE OF A PRODUCT IS NOT EQUAL TO THE PRODUCT OF THE DERIVATIVES!

w3 − 7w
3. .
2w
Solution.

w3 − 7w d w3 7w
Å ã Å ã
d
= −
dw 2w dw 2w 2w
Å ã
d 1 2 7
= w −
dw 2 2
1
= (2w) − 0
2
= w.

d 3
d
Å
w3 − 7w
ã (w − 7w)
Note that , dw .
dw 2w d
(2w)
dw
In general

Å ã0
f f0
, 0.
g g

THE DERIVATIVE OF A QUOTIENT IS NOT EQUAL TO THE QUOTIENT OF THE DERIVATIVES!

Theorem 3.7 (Product Rule). If f and g are differentiable functions then f g is differentiable and

( f g)0 (x) = f 0 (x)g(x) + f (x)g0 (x).

Proof. We will use the algebraic trick of adding and subtracting f (x)g(x + h) in the numerator to evaluate the limit
46 1st Semester Calculus Lecture Manual 2017 MATH1036

in the definition of the derivative.


( f g)(x + h) − ( f g)(x)
( f g)0 (x) = lim
h→0 h
f (x + h)g(x + h) − f (x)g(x)
= lim
h→0 h
f (x + h)g(x + h) − f (x)g(x + h) + f (x)g(x + h) − f (x)g(x)
= lim
h→0 h
[ f (x + h) − f (x)]g(x + h) + f (x)[g(x + h) − g(x)]
= lim
h→0 h
[ f (x + h) − f (x)]g(x + h) f (x)[g(x + h) − g(x)]
Å ã
= lim +
h→0 h h
[ f (x + h) − f (x)]g(x + h) f (x)[g(x + h) − g(x)]
Å ã Å ã
= lim + lim
h→0 h h→0 h
[ f (x + h) − f (x)] g(x + h) − g(x)
Å ã Å ã
= lim lim g(x + h) + lim f (x) lim
h→0 h h→0 h→0 h→0 h
= f (x)g(x) + f (x)g (x).
0 0


Hence
Å ã
d d d
[ f (x)g(x)] = f (x) g(x) + f (x) g(x).
dx dx dx

or
(Derivative of product)= (Derivative of first factor)× (Second factor)+ (First factor) × (Derivative of second factor)

It is sometimes convenient to write the Product Rule in the form

(uv)0 = uv0 + vu0

Worked Example 3.5.3. Use the product rule to find the first derivatives of the following functions.
1. (x3 + 1)(2x2 + 8x − 5).
Solution.
Å ã
d d 3 d
[(x + 1)(2x + 8x − 5)] =
3 2
(x + 1) (2x2 + 8x − 5) + (x3 + 1) (2x2 + 8x − 5)
dx dx dx
= 3x2 (2x2 + 8x − 5) + (x3 + 1)(4x + 8)
= 6x4 + 24x3 − 15x2 + 4x4 + 8x3 + 4x + 8
= 10x4 + 32x3 − 15x2 + 4x + 8.

The Product Rule can be extended to three (and more) factors

( f gh)0 = f 0 gh + f g0 h + f gh0

2. (8w − 1)(w2 + 4w + 7)(w3 − 5).


Solution. Using the above formula,

d
(8w − 1)(w2 + 4w + 7)(w3 − 5)
dw
= 8(w2 + 4w + 7)(w3 − 5) + (8w − 1)(2w + 4)(w3 − 5) + (8w − 1)(w2 + 4w + 7)(3w2 ).
MATH1036 1st Semester Calculus Lecture Manual 2017 47

f
Theorem 3.8 (Quotient Rule). If f and g are differentiable functions and g(x) , 0, then is differentiable and
g
Å ã0
f f 0 (x)g(x) − f (x)g0 (x)
(x) = .
g [g(x)]2

Proof.
Å ã Å ã
f f
(x + h) − (x)
g g
Å ã
f
(x) = lim
g h→0 h
f (x + h) f (x)

g(x + h) g(x)
= lim
h→0 h
f (x + h)g(x) − f (x)g(x + h)
Å ã

g(x)g(x + h)
= lim
h→0 h
f (x + h)g(x) − f (x)g(x + h)
= lim
h→0 hg(x)g(x + h)
f (x + h)g(x) − f (x)g(x) + f (x)g(x) − f (x)g(x + h)
= lim
h→0 hg(x)g(x + h)
f (x + h) − f (x) g(x + h) − g(x)
Å ã
g(x) − f (x)
h h
= lim
h→0 g(x)g(x + h)
f (x + h) − f (x) g(x + h) − g(x)
Å ã Å ã
lim lim g(x) − lim f (x) lim
h→0 h h→0 h→0 h→0 h
=
lim g(x) lim g(x + h)
h→0 h→0
f 0 (x)g(x) − f (x)g0 (x)
= .
[g(x)]2

Hence
Å ã
d d
Å ã f (x) g(x) − f (x) g(x)
d f (x) dx dx
=
dx g(x) [g(x)]2

Worked Example 3.5.4. Use the Quotient Rule to evaluate the first derivative of the following functions.
2x − 7
1. 2
x +9
Solution.
Å ã
d d
Å ã (2x − 7) (x2 + 9) − (2x − 7) (x2 + 9)
d 2x − 7 dx dx
=
dx x2 + 9 (x2 + 9)2
(2)(x2 + 9) − (2x − 7)(2x)
=
(x2 + 9)2
2x2 + 18 − 4x2 + 14x
=
(x2 + 9)2
−2x2 + 18 + 14x
= .
(x2 + 9)2
48 1st Semester Calculus Lecture Manual 2017 MATH1036

v3 − 1
2.
v3 + 1
Solution.

v3 − 1 (3v2 )(v3 + 1) − (v3 − 1)(3v2 )


Å ã
d
=
dv v3 + 1 (v3 + 1)2
3v5 + 3v2 − 3v5 + 3v2
=
(v3 + 1)2
6v2
= 3 .
(v + 1)2

Alternatively,

v3 − 1 d (v3 + 1) − 2
Å ã Å ã
d
=
dv v3 + 1 dv v3 + 1
Å ã
d 2
= 1− 3
dv v +1
−(2)(3v2 )
=− 3
(v + 1)2
6v2
= 3 .
(v + 1)2

We can now extend the Power Rule to the case in which the exponent is a negative integer using the Quotient Rule.

Theorem 3.9 (Power Rule for negative integer power functions). If f (x) = xn and n ∈ Z− , then f 0 (x) = nxn−1 .

Proof. Since n ∈ Z− , n = −m for some positive integer m. Then


Å ã
d n d −m d 1
f 0 (x) = x = x =
dx dx dx xm
Å ã
d d
1 xm − (1) xm
dx dx
=
(xm )2
(0)xm − (1)(mxm−1 )
=
x2m
m−1
−mx
=
x2m
= −mxm−1−2m
= −mx−m−1
= nxn−1 (∵ n = −m) 

Hence
d −n
∀ n ∈ Z+ x = −nx−n−1
dx

The Power Rule is also extended to include rational exponents, the proof of which will be done as a consequence
of section 3.7 on Implicit Differentiation of these notes. The Power Rule holds in fact for all reals.

Theorem 3.10 (General Power Rule). If r is any real number, then

d r
x = rxr−1 .
dx
MATH1036 1st Semester Calculus Lecture Manual 2017 49

Worked Example 3.5.5. Use the generalized Power Rule to find the derivatives of the following functions.
1
1. f (x) = .
x
1 1
Solution. f (x) = ⇒ f (x) = x−1 ⇒ f 0 (x) = −x−2 = − 2 .
x x

2. g(x) = x.
√ 1 1 1 −1 1 − 1 1
Solution. g(x) = x ⇒ g(x) = x 2 ⇒ g0 (x) = x2 = x 2 = √ .
2 2 2 x
The square root function occurs frequently and it is therefore useful to REMEMBER THAT

d √ 1
x= √
dx 2 x

√ 1
instead of keep changing x to its rational power function x 2 and then differentiating the latter using the Power
Rule.
3. h(x) = xe+π .
Solution. h(x) = xe+π ⇒ h0 (x) = (e + π)xe+π−1 .
√ √ √
4. v(t) = t 2 + t + 2.
√ √ 1
Solution. v0 (t) = 2t 2−1 + √ .
2 t
Tutorial 3.5.1. 1. Thomas Exercises 11th ed. 3.2 pp. 167–169: 9, 10, 12, 13, 15, 17, 20, 22, 43, 46, 52, 55 or
12th ed. 3.3 pp. 143–145: 9, 10, 12, 13, 15, 17, 20, 22, 57, 60, 74, 77.
2. Thomas Chapter 3 Practice Exercises 11th ed. pp. 255–257: 4, 9, 99, 101, 107, 109 or 12th ed. pp. 213–215:
4, 9, 99, 101, 107, 109.

The next critically important rule is for composite functions.

Theorem 3.11 (Chain Rule). If f and g are both differentiable functions, then so is the composite function f ◦ g
and
( f ◦ g)0 (x) = f 0 (g(x))g0 (x).

Proof. On the domain of f define the function

 f (t) − f (g(x))

if t , g(x),
F(t) = t − g(x)
if t = g(x).
 0
f (g(x))

Since f is differentiable at g(x), by definition of the derivative we have


f (t) − f (g(x))
f 0 (g(x)) = lim .
t→g(x) t − g(x)
But by definition of F,
f (t) − f (g(x))
lim = lim F(t).
t→g(x) t − g(x) t→g(x)

Therefore
lim F(t) = f 0 (g(x)) = F(g(x)),
t→g(x)

which shows that F is continuous at g(x).


Since g is differentiable, g is continuous at x so that the composite function F ◦ g is continuous at x. Hence

lim F(g(t)) = F(g(x)). (*)


t→x
50 1st Semester Calculus Lecture Manual 2017 MATH1036

Now for t , x, if g(t) , g(x) we have

g(t) − g(x) f (g(t)) − f (g(x)) g(t) − g(x)


ï ò ï ò
F(g(t)) =
t−x g(t) − g(x) t−x
f (g(t)) − f (g(x))
= .
t−x
The above equation also holds if g(t) = g(x) since in this case LHS = 0 = RHS. So we have for all t , x in the
domain of f ◦ g that
g(t) − g(x)
ï ò
f (g(t)) − f (g(x))
= F(g(t)) .
t−x t−x
Therefore
g(t) − g(x)
Å ï òã
f (g(t)) − f (g(x))
lim = lim F(g(t))
t→x t−x t→x t−x
g(t) − g(x)
ï ò
= lim F(g(t)) lim
t→x t→x t−x
g(t) − g(x)
ï ò
= F(g(x)) lim by (*)
t→x t−x
= F(g(x))g0 (x)
= f 0 (g(x))g0 (x) by definition of F.

Thus, by definition of the derivative,


( f ◦ g)0 (x) = f 0 (g(x))g0 (x).


The Chain Rule may also be expressed in the following form. If y = f (u) and u = g(x) are both differentiable
functions of x, then
Å ãÅ ã
dy dy du
=
dx du dx

Worked Example 3.5.6. Use the Chain Rule to find the first derivatives of the following.
1. (x2 − 1)100 .
Solution.
d 2 d
(x − 1)100 = 100(x2 − 1)99 (x2 − 1) = 100(x2 − 1)99 (2x) = 200x(x2 − 1)99 .
dx dx

1
2. .
(4w2 + 6w − 7)3
Solution.
Å ã
d 1 d
= (4w2 + 6w − 7)−3
dw (4w2 + 6w − 7)3 dw
d
= −3(4w2 + 6w − 7)−4 (4w2 + 6w − 7)
dw
= −3(4w2 + 6w − 7)−4 (8w + 6)
−3(8w + 6)
=
(4w2 + 6w − 7)4
−6(4w + 3)
= .
(4w2 + 6w − 7)4
MATH1036 1st Semester Calculus Lecture Manual 2017 51


3. v2 + v + 4.

Solution.
d √2 1 d 2 2v + 1
v +v+4= √ (v + v + 4) = √ .
dv 2 v2 + v + 4 dv 2 v2 + v + 4


4. x x2 + 1.

Solution.

d Ä √ 2
ã√
d √ 2
Å
ä d
x x +1 = x x2 + 1 + x x +1
dx dx dx
√ Å
1 d 2
ã
= (1) x2 + 1 + x √ (x + 1)
2 x2 + 1 dx
√ Å
2x
ã
= x2 + 1 + x √
2 x2 + 1
√ x2
= x2 + 1 + √
x2 + 1
(x + 1) + x
2 2
= √
x2 + 1
2x + 1
2
= √ .
x2 + 1

 x  23
5. .
x2 + 16
Solution.

d  x  32 2  x  23 −1 d  x 
=
dx x2 + 16 3 x2 + 16 dx x2 + 16
2  x  − 3 (1)(x2 + 16) − x(2x)
1

= (Quotient Rule)
3 x2 + 16 (x2 + 16)2
2  x  − 3 16 − x2
1

= .
3 x2 + 16 (x2 + 16)2

It is apparent that the Chain Rule gives the following neat calculations.

d
∀r ∈ R [ f (x)]r = r[ f (x)]r−1 f 0 (x)
dx

and
d p f 0 (x)
f (x) = √
dx 2 f (x)
52 1st Semester Calculus Lecture Manual 2017 MATH1036

The following is a summary of the theorems and pertinent results of this section.

dy
Function y Its derivative
dx
constant c, c ∈ R 0

f +g f 0 + g0

f −g f 0 − g0

cf, c ∈ R cf0

fg f 0 g + f g0

f f 0 g − f g0
g g2

xr , r ∈ R rxr−1
√ 1
x √
2 x

f ◦g ( f 0 ◦ g)g0

Worked Example 3.5.7. Use the Derivative Rules in the following questions.
5
1. Find the equation of the normal line to the graph of f (x) = at x = −2.
1 + x2
5
Solution. The tangent line at x = −2 has slope mt = f 0 (−2). Since f (x) = = 5(1 + x2 )−1 ,
1 + x2

−10x
f 0 (x) = −5(1 + x2 )−2 (2x) = .
(1 + x2 )2

Then
(−10)(−2) 20 4
mt = f 0 (−2) = = = .
(1 + (−2)2 )2 25 5
If mn denotes the slope of the normal, then mn mt = −1 so that

1 5
mn = − =− .
mt 4

5 5
At x = −2, f (−2) = = = 1 so that we seek the normal line at the point (−2, 1) on the graph of f . The
1 + (−2)2 5
equation of the normal is then
5
yn − 1 = − (x − (−2)) ,
4
5 5
i. e., yn = − x − + 1. Hence the normal line is the line
4 2

5 3
yn = − x − .
4 2
MATH1036 1st Semester Calculus Lecture Manual 2017 53


2. Let f (x) = x − 3 3 x. Find the points on the graph of f where f has horizontal and vertical tangents.
Solution.
Horizontal tangents:
√3 Horizontal tangents occur at those points in the domain of f for which f 0 (x) = 0.
1
Since f (x) = x − 3 x = x − 3x 3 ,
Å ã 2
1 −2 2 2 2 x3 − 1
f 0 (x) = 1 − 3 x 3 = 1 − x− 3 = x− 3 (x 3 − 1) = 2 .
3 x3
Then
2
x3 − 1
f 0 (x) = 0 ⇔ 2 =0
x3
2
⇔ x3 − 1 = 0
2
⇔ x3 = 1
⇔ x2 = 1
⇔x=1 or x = −1.

Thus f 0 (x) = 0 when x = 1 or x = −1. Since


1
f (1) = 1 − 3(1) 3 = 1 − 3 = −2

and
1
f (−1) = −1 − 3(−1) 3 = −1 − 3(−1) = 2,
f has horizontal tangents at the points (1, −2) and (−1, 2) .
Vertical tangents: Vertical tangents occur at x = a where f is continuous and f 0 exists for x , a but close to
2
x3 − 1
a and lim | f (x)| = ∞. Since f (x) =
0 0
2 for all x , 0, f is differentiable close to x = 0. Futhermore f is
x→a x3
continuous at x = 0 and
2
lim | f 0 (x)| = lim 1 − x− 3 = ∞.
x→0 x→0
1
This shows that f has a vertical tangent x = 0. Since f (0) = 0 − 3(0) 3 = 0, f has a vertical tangent at the origin
(0, 0).

Tutorial 3.5.2. 1. Thomas Exercises 11th ed. 3.5 pp. 199–202: 1, 2, 9, 11, 14, 23, 31, 34, 67, 71, 72–74, 116 or
12th ed. 3.6 pp. 167–169: 1, 2, 9, 11, 13, 23, 31, 34, 79, 83, 84, 87, 88, 104.
2. For the following functions, find the points on the graphs where the tangents are horizontal or vertical.
1
(a) f (x) = x − ,
x
4 1
(b) g(x) = x 3 + 4x 3 ,
3
(c) h(x) = x(x + 2) 5 ,
1
(d) d(x) = xr − x r on [0, ∞], where r > 1.

3.6 Trigonometric Derivatives


To find the derivatives of trigonometric functions we make use of Theorems 1.8 and 1.9 which respectively proves

sin θ 1 − cos θ
lim =1 and lim = 0.
θ→0 θ θ→0 θ
The trigonometric functions are differentiable in their domains and their derivatives are now given using the
limit definition (incorporating the above limits in Theorems 1.8 and 1.9) together with the Derivative Rules and
trigonometric identities.
54 1st Semester Calculus Lecture Manual 2017 MATH1036

d
Theorem 3.12 (Derivative of sin x). sin x = cos x
dx

Proof.
d sin(x + h) − sin x sin x cos h + cos x sin h − sin x
sin x = lim = lim
dx h→0 h h→0 h
Å ã
sin x cos h − sin x cos x sin h
= lim +
h→0 h h
ï Å ã Å ãò
cos h − 1 sin h
= lim sin x + cos x
h→0 h h
 Å cos h − 1
ã  Å sin h
ã
= lim sin x lim + lim cos x lim
h→0 h→0 h h→0 h→0 h
 Å 1 − cos h
ã  Å sin h
ã
= lim sin x − lim + lim cos x lim
h→0 h→0 h h→0 h→0 h

= (sin x)(0) + (cos x)(1)


= cos x. 
Using a similar proof to that of Theorem 3.12 we have the following. We present here an alternate proof using the
aid of a trigonometric identity and the Chain Rule.

d
Theorem 3.13 (Derivative of cos x). cos x = − sin x
dx

Proof.
d d π 
cos x = sin −x
dx dx 2
π  d π 
= cos −x −x (Chain Rule)
π2  dx 2
= cos − x (0 − 1)
2
π 
= − cos −x
2
= − sin x. 

d
Theorem 3.14 (Derivative of tan x). tan x = sec2 x
dx

Proof.
Å ã
d d sin x
tan x =
dx dx cos x
Å ã
d d
sin x cos x − sin x cos x
dx dx
= (Quotient Rule)
cos2 x
cos x cos x − sin x(− sin x)
=
cos2 x
cos2 x + sin2 x
=
cos2 x
= sec2 x. 

d
Theorem 3.15 (Derivative of cosec x). cosec x = − cosec x cot x
dx
MATH1036 1st Semester Calculus Lecture Manual 2017 55

Proof.
Å ã
d d 1
cosec x =
dx dx sin x
d
= (sin x)−1
dx
d
= − (sin x)−2 sin x (Chain Rule)
dx
1
= − 2 (cos x)
sin x
Å ã
1 cos x 
=−
sin x sin x
= − cosec x cot x. 

d
Theorem 3.16 (Derivative of sec x). sec x = sec x tan x
dx

Proof.
Å ã
d d 1
sec x =
dx dx cos x
d
−(1) cos x
= dx (Quotient Rule)
cos2 x
sin x
=
cos2 x
Å ãÅ ã
1 sin x
=
cos x cos x
= sec x tan x. 

d
Theorem 3.17 (Derivative of cot x). cot x = − cosec2 x
dx

Proof.
d d  cos x 
cot x =
dx dx sin x
Å ã
d d
cos x sin x − cos x sin x
dx dx
= (Quotient Rule)
sin2 x
− sin x sin x − cos x cos x
=
sin2 x
− sin2 x − cos2 x
=
sin2 x
= − cosec2 x. 
56 1st Semester Calculus Lecture Manual 2017 MATH1036

Worked Example 3.6.1. Find the derivatives of the following.


1. y = sin x cos x.
Solution.
Å ã
d d
y =
0
sin x cos x + sin x cos x (Product Rule)
dx dx
= cos x cos x + sin x(− sin x)
= cos2 x − sin2 x
= cos 2x.

Alternatively:
1
y = sin x cos x ⇒ y = sin 2x
2
1 d
⇒ y0 = cos 2x (2x) (Chain Rule)
2 dx
1
= cos 2x(2)
2
= cos 2x.
π
2. f 0 if f (x) = x cot x.
4
Solution.
Å ã
d d
f 0 (x) = x cot x + x cot x
dx dx
= cot x + x(− cosec x)
2

= cot x − x cosec2 x
π π π π
⇒ f0 = cot − cosec2
4 4π  Ä √ 4ä2 4
=1− 2
4
π
=1− .
2

3. sec x
Solution.
√ √
d √ √ √ d √ sec x tan x
sec x = sec x tan x x= √ .
dx dx 2 x
4. g(t) = sin3 (4t).
Solution. Since g(t) = (sin 4t)3 ,
d
g0 (t) = 3(sin 4t)2 sin 4t (Chain Rule)
dt
d
= 3 sin2 (4t) cos(4t) (4t) (Chain Rule again)
dt
= 3 sin2 (4t) cos(4t)(4)
= 12 sin2 (4t) cos(4t).

Note: For a double composite f (g(h(x))),


d
f (g(h(x))) = f 0 (g(h(x)))g0 (h(x))h0 (x).
dx
MATH1036 1st Semester Calculus Lecture Manual 2017 57

Thus the problem can be done in one go differentiating from the outside towards the inside:

d
sin3 (4t) = (3 sin2 4t)(cos 4t)(4) = 12 sin2 4t cos 4t.
dt

5. y = x2 + cos3 (sin(3x)).
p

Solution.
dy 1
= p [2x + 3 cos2 (sin 3x)[− sin(sin 3x)](cos 3x)(3)]
dx 2 x2 + cos3 (sin(3x))
2x − 9 cos2 (sin 3x) sin(sin 3x) cos 3x
= .
2 x2 + cos3 (sin(3x))
p

Herewith a summary of the trigonometric derivatives.

dy
y Using Chain Rule with u = f (x)
dx
du
sin x cos x y = sin u ⇒ y0 = (cos u)
dx
du
cos x − sin x y = cos u ⇒ y0 = −(sin u)
dx
du
tan x sec2 x y = tan u ⇒ y0 = (sec2 u)
dx
du
cosec x − cosec x cot x y = cosec u ⇒ y0 = −(cosec u cot u)
dx
du
sec x sec x tan x y = sec u ⇒ y0 = (sec u tan u)
dx
du
cot x − cosec2 x y = cot u ⇒ y0 = −(cosec2 u)
dx

Tutorial 3.6.1. 1. Thomas Exercises 11th ed. 3.4 pp. 186–187: 1, 2, 5, 10, 11, 24, 26 or 12th ed. 3.5 pp.
159–160: 1, 2, 6, 9, 14, 15, 18, 20.
2. Thomas Exercises 11th ed. 3.5 pp. 199–200: 3, 6, 7, 28, 43, 44, 46, 57, 69, 70 or 12th ed. 3.6 pp. 167–168: 3,
6, 7, 28, 45, 46, 48, 63, 81, 82.
3. Thomas Chapter 3 Practice Exercises 11th ed. p. 255: 20, 39 or 12th ed. pp. 213–214: 20, 39.

3.7 Implicit Differentiation



Equations of the form y = f (x) give y explicitly as a function of x, e. g., y = x3 + 1 and y = sin x. Thus far we
have been differentiating functions of this type. In some cases y is not given explicitly as a function of x, of which
dy
we need to find as illustrated in the next example.
dx
Worked Example 3.7.1. Suppose that y is a function of x that satisfies the equation

3x3 y − 4y − 2x + 1 = 0.

dy
Find by first finding y explicitly.
dx
58 1st Semester Calculus Lecture Manual 2017 MATH1036

Solution. The only way that we know to solve for y0 is to somehow write y explicitly and this can be done with
this example.

3x3 y − 4y − 2x + 1 = 0
⇒ (3x3 − 4)y = 2x − 1
2x − 1
⇒y= 3
3x − 4
dy 2(3x3 − 4) − (2x − 1)(9x2 )
∴ =
dx (3x3 − 4)2
9x2 (2x − 1)
Å ã
(3x3 − 4) 2 −
3x3 − 4
=
(3x − 4)2
3
Å ã
2x − 1
2 − 9x2
3x3 − 4
= 3
3x − 4
2 − 9x2 y 2x − 1
= 3
∵ y= 3 .
3x − 4 3x − 4

It is generally not so simple to write such equations explicitly as y = f (x). Such functions (like the above example)
dy
are said to be given implicitly by the equation. When these functions are differentiable, it is possible to find
dx
dy
without solving the equation for y. This method is called implicit differentiation. This technique to find is
dx
given below.

Implicit differentiation: Differentiate both sides of the equation w.r.t x


taking into account the Chain Rule
dy
and then solve the resulting equation for .
dx

In the above example we differentiate implicitly as follows.

3x3 y − 4y − 2x + 1 = 0
d d
⇒ (3x3 y − 4y − 2x + 1) = (0)
dx dx
dy dy
⇒ 3(3x2 )y + 3x3 −4 −2+0=0
dx dx
dy 3
⇒ (3x − 4) = 2 − 9x2 y
dx
dy 2 − 9x2 y
∴ = .
dx 3x3 − 4

When using the method of implicit differentiation it is important to remember that y is a function of x and that we
are differentiating with respect to x. Hence, powers of y are powers of functions and the Chain Rule is required for
their differentiation. Also be prepared to use combinations of the Chain Rule, Product Rule and Quotient Rule.

dy
Worked Example 3.7.2. 1. Find by implicit differentiation if x2 + y2 = 4.
dx
MATH1036 1st Semester Calculus Lecture Manual 2017 59

Solution.

x2 + y2 = 4
d 2 d
⇒ (x + y2 ) = (4)
dx dx
dy
⇒ 2x + 2y =0
dx
dy
⇒ 2y = −2x
dx
dy x
∴ =− .
dx y

We have just found the slope at each point (x, y) on the graph of the implict function x2 + y2 = 4 (circle centered
at the origin, radius 2). Note that at x = ±2 (when y = 0) x2 + y2 = 4 is not differentiable. The circle has vertical
dy x
tangents at the points (−2, 0) and (2, 0). Thus = − are the slopes of the tangent lines at each point on the
dx y
circle x2 + y2 = 4 except at the non-differentiable points (−2, 0) and (2, 0).

2. If p, q ∈ Z and q , 0 prove that


d qp p p
x = x q −1 .
dx q
(This proves the Power Rule for all rationals).
p
Proof. Let y = x q . Then yq = x p . Implicit differentiation then gives:

yq = x p
dy
⇒ qyq−1 = px p−1
dx
dy px p−1
⇒ = q−1
dx qy
px p−1
= Ä p äq−1
q xq
px p−1
= p
qx p− q
p p
= x p−1−p+ q
q
p p
= x q −1 .
q
p d qp p
Hence, since y = x q we have proved that x = qp x q −1 . 
dx
3. Find y0 if xy tan yx = x2 + y3 .
Solution.
y
xy tan = x2 + y3 ⇒ xy tan(yx−1 ) = x2 + y3
x
⇒ y tan(yx−1 ) + x tan(yx−1 )y0 + xy sec2 (yx−1 )[x−1 y0 − yx−2 ] = 2x + 3y2 y0
⇒ y tan(yx−1 ) + x tan(yx−1 )y0 + y sec2 (yx−1 )y0 − x−1 y2 sec2 (yx−1 ) = 2x + 3y2 y0
⇒ x tan(yx−1 )y0 + y sec2 (yx−1 )y0 − 3y2 y0 = 2x − y tan(yx−1 ) + x−1 y2 sec2 (yx−1 )
2x − y tan(yx−1 ) + x−1 y2 sec2 (yx−1 )
⇒ y0 = .
x tan(yx−1 ) + y sec2 (yx−1 ) − 3y2
60 1st Semester Calculus Lecture Manual 2017 MATH1036

4. Find the equation of the tangent line at (x, y) = ( π6 , 5π


6 ) to the curve

π
x sin y + y sin x = .
2

Solution.
π
x sin y + y sin x =
2
⇒ sin y + x(cos y)y0 + y0 sin x + y cos x = 0
⇒ y0 (x cos y + sin x) = − sin y − y cos x
sin y + y cos x
∴ y0 = −
x cos y + sin x

Then at ( π6 , 5π
6 ),

π 5π 6 +
sin 5π 5π
cos π6
Å ã
y0 , =− π
6
π
6 + sin 6
cos 5π
6 6 6
Ä√ ä
2 + 6
1 5π 3
2
=− √
π
Ä ä
6 − 2
3
+ 12

6 + 5π 3
=− √ .
6 − 3π

Thus at ( π6 , 5π
6 ) the equation of the tangent is

0 π 5π π
ï Å ãò 

y− = y , x−
6 6 6 6

i. e., √ ô
6 + 5π 3  π
ñ

y= − √ x− .
6 6 − 3π 6

Tutorial 3.7.1. 1. Thomas Exercises 11th ed. 3.6 pp. 209–210: 19, 22, 25, 28, 30, 31, 35, 47, 54, 56, 59, 61 or
12th ed. 3.7 p. 174: 1, 4, 7, 10, 13, 19, 31, 38, 40, 43, 45.

3.8 Derivatives of Exponential and Logarithmic Functions


Consider the exponential functions f (x) = a x , a > 0. All such functions have the only intercept (0, 1). The
derivative at x = 0 of each exponential function is of great significance. Using the limit definition to calculate
f 0 (0) we get

f (0 + h) − f (0)
f 0 (0) = lim
h→0 h
a0+h − a0
= lim
h→0 h
ah − 1
= lim .
h→0 h
Thus for each exponential function f (x) = a x , the slope of the tangent at x = 0 is given by the limit

ah − 1
f 0 (0) = lim .
h→0 h
MATH1036 1st Semester Calculus Lecture Manual 2017 61

We will use the following fact, without proof: Of all possible exponential functions, the natural exponential
function f (x) = e x is the only one whose tangent line at x = 0 has slope 1.
Thus for f (x) = e x , f 0 (0) = 1, i. e.,
eh − 1
lim =1
h→0 h

Consequently, we have the following important derivative theorem.

Theorem 3.18 (Derivative of e x ).


d x
e = ex
dx

Proof. Using the limit definition

d x e x+h − e x
e = lim
dx h→0 h
e x eh − e x
= lim
h→0 h
Å h ã
x e −1
= lim e
h→0 h
h
e −1
= e x lim
h→0 h
= e (1)
x

= ex . 

Multiples of the natural exponential function are the only functions whose derivatives are themselves, i. e.,

d
Ce x = Ce x
dx

Incorporating the Chain Rule we have


d f (x)
e = e f (x) f 0 (x)
dx

i. e.
Incorporating the Chain Rule we have

Derivative of eFunction = eFunction × (Derivative of Function)

Worked Example 3.8.1. Find y0 in the following.



x2 +1
1. y = e .
Solution.

d √ 2
x2 +1
y0 = e x +1
dx

1
= e x +1 √
2
2x
2 x2 + 1

xe x +1
2

= √
x2 + 1
62 1st Semester Calculus Lecture Manual 2017 MATH1036

2. y = e2x cos(πx).
Solution.

y0 = e2x (2) cos(πx) + e2x (− sin(πx))π


= e2x [2 cos(πx) − π sin(πx)]

3. xey − ye x = 2.
Solution. Implicit differentiation gives

ey + xey y0 − [y0 e x + ye x ] = 0
⇒ (xey − e x )y0 = ye x − ey
ye x − ey
∴ y0 = y .
xe − e x
√ √
4. y = e x
+ ex .
Solution. √
√ 1 1 e x 1√ x
y =e
0 x
√ + √ ex = √ + e.
2 x 2 ex 2 x 2

To find the derivative of y = a x for any exponential function we resort to the properties of exponentials and
logarithms, in particular
eln x = x and ln xr = r ln x.

Theorem 3.19 (Derivative of a x ).


d x
a = a x ln a.
dx

x
Proof. Since a x = eln a = e x ln a , we have

d x d x ln a d
a = e = e x ln a (x ln a)
dx dx dx
= e x ln a ln a
= a x ln a. 

Incorporating the Chain Rule we have

d f (x)
a = a f (x) (ln a) f 0 (x).
dx

Worked Example 3.8.2. Find y0 if


1. y = 5 x
Solution. y0 = 5 x ln 5.
2
2. y = 35x 23x .
Solution.
2 2 2
y0 = 35x (5 ln 3)23x + 35x 23x (6x ln 2) = (5 ln 3 + 6x ln 2)35x 23x .

Tutorial 3.8.1. 1. Thomas Exercises 11th ed. 3.2 p. 167: 24, 27, 28 or 12th ed. 3.3 p. 143: 30, 31, 40.
2. Thomas Exercises 11th ed. 3.6 p. 209: 32 or 12th ed. 3.7 p. 174: 16.
3. Thomas Practice Exercises 11th ed. pp. 255–256: 44, 73, 74 or 12th ed. p. 214: 44, 73, 74.
MATH1036 1st Semester Calculus Lecture Manual 2017 63

Theorem 3.20 (Derivative of ln x).


d 1
ln x =
dx x

Proof. Let y = ln x. We use the properties of exponentials and logarithms, the Chain Rule and incorporate implicit
differentiation.

y = ln x
⇔ ey = x
d y d
⇒ e = x
dx dx
⇒ ey y0 = 1
1
⇒ y0 = y
e
1
⇒y = 0
x
d 1
∴ ln x = . 
dx x
Together with the Chain Rule we have
d f 0 (x)
ln[ f (x)] =
dx f (x)

Worked Example 3.8.3. Find y0 if


1. y = xr for r ∈ R.
r
Solution. Note that y = xr = eln x = er ln x . Then

d
y0 = er ln x (r ln x)
dx
Å ã
1
= er ln x r
x
= x rx
r −1


∴ y0 = rxr−1 .

The above proves the general Power Rule


d r
x = rxr−1
dx

2. y = ln(ln x).
Solution.
Å ãÅ ã
1 d
y =
0
ln x
ln x dx
Å ãÅ ã
1 1
=
ln x x
1
= .
x ln x
64 1st Semester Calculus Lecture Manual 2017 MATH1036


3. y = ln(x + x2 + 1).
Solution.
1 d √
y0 = √ (x + x2 + 1)
x + x + 1 dx
2
Å ã
1 1
= √ 1+ √ (2x)
x + x2 + 1 2 x2 + 1
Å ã
1 x
= √ 1+ √
x+ x +12 x +1
2
Ç√
x2 + 1 + x
å
1
= √ √
x + x2 + 1 x2 + 1
1
= √ .
x2 + 1

4. y3 + x2 ln y = 5x + 3.
Solution. Implicit differentiation.

y0
ãÅ
3y y + 2x ln y + x
2 0 2
=5
y
x2
Å ã
⇒ y0 3y2 + = 5 − 2x ln y
y
3y + x2
Å 3 ã
⇒ y0 = 5 − 2x ln y
y
5y − 2xy ln y
⇒ y0 =
3y3 + x2

x2 (2x − 1)3
5. y = ln .
(x + 5)2
Solution. Direct differentiation would require using the Chain Rule together with the Quotient Rule and the
Product Rule. Instead we use the properties of logarithms to first simplify and then differentiate.

x2 (2x − 1)3
y = ln ⇒ y = ln[x2 (2x − 1)3 ] − ln(x + 5)2
(x + 5)2
⇒ y = ln x2 + ln(2x − 1)3 − 2 ln(x + 5)
⇒ y = 2 ln x + 3 ln(2x − 1) − 2 ln(x + 5)
Å ã Å ã Å ã
1 2 1
⇒ y0 = 2 +3 −2
x 2x − 1 x+5
2 6 2
⇒ y0 = + −
x 2x − 1 x + 5
2(2x − 1)(x + 5) + 6x(x + 5) − 2x(2x − 1)
⇒ y0 =
x(2x − 1)(x + 5)
4x 2
+ 18x − 10 + 6x2 + 30x − 4x2 + 2x
⇒ y0 =
x(2x − 1)(x + 5)
6x 2
+ 50x − 10
⇒ y0 =
x(2x − 1)(x + 5)
2(3x2 + 25x − 5)
⇒ y0 =
x(2x − 1)(x + 5)
MATH1036 1st Semester Calculus Lecture Manual 2017 65

Theorem 3.21 (Derivative of loga x).


d 1
loga x = .
dx x ln a

loge x ln x
Proof. Let y = loga x. Using the change of base property of logarithms, y = = . Then
loge a ln a
Å ã Å ã Å ã
d ln x 1 d 1 1 1
y =
0
= ln x = = . 
dx ln a ln a dx ln a x x ln a
Incorporating the Chain Rule we have

d f 0 (x)
loga f (x) = .
dx f (x) ln a

Worked Example 3.8.4. Find f 0 (x) if


6x + 4
1. f (x) = log5 .
2x − 3
6x + 4
Solution. Since f (x) = log5 = log5 (6x + 4) − log5 (2x − 3),
2x − 3
6 2
f 0 (x) = −
(6x + 4) ln 5 (2x − 3) ln 5
Å ã
1 6 2
= −
ln 5 6x + 4 2x − 3
1 12x − 18 − (12x + 8)
ï ò
26
= =−
ln 5 (6x + 4)(2x − 3) (ln 5)(6x + 4)(2x − 3)

2. f (x) = ln(log x).


Solution. Since log x = log10 x,
Å ã
1 d
f (x) =
0
log x
log x dx
Å ãÅ ã
1 1
=
log x x ln 10
1
= .
x(ln 10) log x

Herewith a summary of exponential and logarithmic derivatives.

dy
y Using Chain Rule with u = f (x)
dx
du
ex ex y = eu ⇒ y0 = eu
dx
du
ax a x ln a y = au ⇒ y0 = au ln a
dx
1 1 du
ln x y = ln u ⇒ y0 =
x u dx
1 1 du
loga x y = loga u ⇒ y0 =
x ln a u ln a dx
66 1st Semester Calculus Lecture Manual 2017 MATH1036

Tutorial 3.8.2. 1. Thomas Exercises 11th ed. 3.7 pp. 221–222: 13, 14, 21, 22, 28, 30, 33, 38, 57–60, 76, 77, 86
or 12th ed. 3.8 pp. 184–185: 13, 14, 21, 22, 28, 30, 33, 38, 57–60, 76, 77, 86.

3.9 Linear Approximation and Differentials


Using the derivative, we will illustrate one of the simplest types of approximation methods, namely linear ap-
proximation. The idea is to use the value of the function and its derivative at a single point to find approximate
values of the function at nearby points.
Assume that f is a differentiable function at x = a and that we are interested in approximate values of f (x) for x
near a. We use the tangent line of f at (a, f (a)) as an approximation of f (x) when x is near a. The tangent line is
a graph of a linear function and the equation of the tangent line at x = a is

y − f (a) = f 0 (a)(x − a),

equivalently,
y = f (a) + f 0 (a)(x − a).
The approximation
f (x) ≈ f (a) + f 0 (a)(x − a) for x ≈ a
is the linear approximation of f at a. The linear function

L(x) = f (a) + f 0 (a)(x − a)

whose graph is the tangent line is called the linearization of f at a.



Worked Example
√ 3.9.1. Find the linearization of the function f (x) = x at x near 1 and use it to approximate
the value of 1.02.
√ 1
Solution. The linearization is the equation of the tangent line at x = 1. Since f (1) = 1 = 1 and f (x) = √ ,
2 x
the linearization of f at x = 1 is

L(x) = f (1) + f 0 (1)(x − 1)


1
= 1 + √ (x − 1)
2 1
1
= 1 + (x − 1).
2
The corresponding linear approximation is
√ 1
f (x) = x ≈ 1 + (x − 1).
2

Then 1.02 can now be approximated as
√ 1
1.02 ≈ 1 + (1.02 − 1) = 1.01.
2

A calculator gives 1.02 ≈ 1.0099505.

In many problems we are interested in the change, or approximate change, of values y = f (x) that correspond to
a change in x. If x changes from x1 to x2 , this amount of change is denoted by ∆x. Similarly ∆y (∆ f ) will denote
the change in the dependent variable y that corresponds to the change ∆x. Thus

∆y = f (x2 ) − f (x1 ) = f (x1 + ∆x) − f (x1 ).

In the linear approximation, if ∆x = x − a represents the change in x, the corresponding change in y = f (x) is
∆y = f (x) − f (a). If f is differentiable at a, then we know that

f (x) ≈ f (a) + f 0 (a)(x − a)


MATH1036 1st Semester Calculus Lecture Manual 2017 67

so that
f (x) − f (a) ≈ f 0 (a)(x − a),
i. e.,
∆y ≈ f 0 (a) ∆x.
We use a notation that suggests the derivative in the linear approximation. We write dy for ∆y and dx for ∆x. The
differential of y = f (x) at x = a is defined as dy where

dy = f 0 (a) dx.

The symbols dy and dx represent variables with dy dependent on dx. dy is called the differential approximation
of the change in y = f (x).

Worked Example 3.9.2. Find the differentials dy of the following functons.



1. y = 1 + x2 .
2x x
Solution. dy = √ dx = √ dx.
2 1 + x2 1 + x2
2. y = (1 + 2r)−4 .
Solution. dy = −4(1 + 2r)−5 (2) dr = −8(1 + 2r)−5 dr.
3. y = t2 et .
Solution. dy = (t2 et + 2tet ) dt = tet (t + 2) dt.

The following are rules for working with differentials. If c ∈ R and u and v are functions of x, then

differential of a constant dc = 0

factorization d(cu) = c du

differential sum d(u + v) = du + dv

differential product d(uv) = v du + u dv


u v du − u dv
differential quotient d =
v v2
differential power d(xn ) = nxn−1 dx

Tutorial 3.9.1. 1. Thomas Exercises 11th ed. 3.10 pp. 250–251: 1, 2, 3, 6, 8, 9, 11, 13, 14, 21, 30, 31, 33, 38, 39,
43, 44 or 12th ed. 3.11 pp. 210–211: 1, 2, 3, 6, 8, 9, 11, 13, 14, 21, 30, 31, 33, 38, 39, 43, 44.
Chapter 4

Introduction to Integration

NOTE: Integration is particularly important to those students studying statistics.

LEARNING OUTCOMES:
On completion of this chapter you should (tick the checkbox when you have mastered the skill)
 1. know the definition of an antiderivative,
 2. be able to verify that a function is an antiderivative of a given function,
 3. understand the indefinite integral as an antiderivative,
 4. know the antiderivatives or integrals of all the known functions,
 5. know the properties of the definite integral,
 6. know how to apply the Fundamental Theorem of Calculus (FTC) in computing a definite integral,
 7. know and understand how to use the Substitution Rule in evaluating a definite or indefinite integral,
 8. know and understand how to use the technique of Integration by Parts (IBP) in evaluating a definite or
indefinite integral,
 9. know how to solve the examinable tutorial problems and the worked out problems from these notes on
this chapter.

For a given function f we have been able to compute the (unique) derivative f 0 . In this Chapter we consider the
following question.

For a given function f , can we find a function (or functions) F such that F 0 (x) = f (x) for each x in an
interval?

4.1 Antidifferentiation and the Indefinite Integral


A function F is called an antiderivative of the function f on an interval I if F 0 (x) = f (x) for all x ∈ I.

Worked Example 4.1.1. Verify that F(x) is an antiderivative of f (x) in the following.
1. F(x) = −x cos x + sin x and f (x) = x sin x.
Solution. F is an antiderivative of f if and only if F 0 (x) = f (x). Since

F 0 (x) = (−1) cos x − x(− sin x) + cos x = x sin x = f (x),

F is indeed an antiderivative of f .
3 √
2. F(x) = 13 (x2 + 1) 2 and f (x) = x x2 + 1.

68
MATH1036 1st Semester Calculus Lecture Manual 2017 69

Solution.
1
Å
3 2 1
ã √
F (x) =
0
(x + 1) (2x) = x x2 + 1 = f (x).
2
3 2
Hence F 0 (x) = f (x) so that F is an antiderivative of f .
3. F(x) = x ln x − x and f (x) = ln x.
Å ã
1
Solution. F (x) = (1) ln x + x
0
− 1 = ln x = f (x). Hence F is an antiderivative of f .
x
4. F(x) = e x and f (x) = e x .
Solution. Since F 0 (x) = e x = f (x), F is an antiderivative of f . In other words, e x is an antiderivative of itself.
ax
5. F(x) = and f (x) = a x .
ln a
Å ã Å ã
1 d x 1
Solution. Since F 0 (x) = a = (a x ln a) = f (x), F is an antiderivative of f .
ln a dx ln a
Antiderivatives are never unique. Since the derivative of a constant is zero, it follows that if F is an antiderivative
of f , then so are the functions G(x) = F(x) + C for every number C. Functions of this type, F(x) + C, are the only
possible antiderivatives of f .

If F is an antiderivative of f , then the most general antiderivative of f on an interval I is F(x) + C, where C is


an arbitrary constant. The most general antiderivative is denoted by the symbol
Z
f (x) dx

R
and called the indefinite integral of f . The function f (x) is called the integrand of the indefinite integral. Here
and dx are merely parts of a symbol and have no individual meaning. Thus if we know that F is an antidervative
of f , then the most general antiderivative or indefinite integral of f is
Z
f (x) dx = F x) + C.

Based on the previous Worked Example we may conclude the following results.
R
Worked Example 4.1.2. 1. x sin x dx = −x cos x + sin x + C.
R √ 3
2. x x2 + 1 dx = 13 (x2 + 1) 2 + C.
R
3. ln x dx = x ln x − x + C.
R
4. e x dx = e x + C.
R ax
5. a x dx = + C.
ln a
The process of indefinite integration is thus the procedure of finding the most general antiderivative of a given
function.
Z
f (x) dx is all solutions to the question “What must I differentiate to get the integrand f (x)?”

Theorem 4.1 (Integral of power functions). Let f (x) = xr where −1 , r ∈ R. Then

Z
1 r+1
xr dx = x + C.
r+1
70 1st Semester Calculus Lecture Manual 2017 MATH1036

1 r+1
Proof. Let F(x) = x . Then
r+1
1
F 0 (x) = (r + 1)xr+1−1 = xr .
r+1
Thus Z
1 r+1
xr dx = x + C. 
r+1

Theorem 4.2 (Integral of 1x ).


Z
1
dx = ln |x| + C.
x

Proof. Let f (x) = ln |x|. Then


if x > 0,
(
ln x
f (x) =
ln(−x) if x < 0.
It follows that 1
 if x > 0,
x

f 0 (x) =
 1 (−1) = 1 if x < 0.

−x x
1 d 1
Thus f 0 (x) = for all x , 0. Therefore, ln |x| = . Hence
x dx x
Z
1
dx = ln |x| + C. 
x
Theorems 4.1 and 4.2 give the integral of the power functions for each r ∈ R:

 1 xr + C

if r , −1,
Z
x dx = r + 1
r

ln |x| + C if r = −1.

d 1 d f 0 (x)
Note. ln |x| = and the Chain Rule give ln | f (x)| = . Thus
dx x dx f (x)

f 0 (x)
Z
dx = ln | f (x)| + C.
f (x)

Worked Example 4.1.3. Find y0 if


1. y = ln | sin x|.
Solution. Å ã
1 d cos x
y0 = sin x = = cot x.
sin x dx sin x
This shows that
d
ln | sin x| = cot x.
dx
Thus ln | sin x| is an antiderivative of cot x, and hence
Z
cot x dx = ln | sin x| + C.
MATH1036 1st Semester Calculus Lecture Manual 2017 71

2. y = ln | sec x|.
Solution. Å ã
1 d sec x tan x
y0 = sec x = = tan x.
sec x dx sec x
This shows that
d
ln | sec x| = tan x.
dx
Thus ln | sec x| is an antiderivative of tan x, and hence
Z
tan x dx = ln | sec x| + C.

3. y = ln | sec x + tan x|.


Solution.
Å ã
1 d
y0 = (sec x + tan x)
sec x + tan x dx
Å ã
1
= (sec x tan x + sec2 x)
sec x + tan x
Å ã
1
= sec x(tan x + sec x)
sec x + tan x
= sec x.

This shows that


d
ln | sec x + tan x| = sec x.
dx
Thus ln | sec x + tan x| is an antiderivative of sec x, and hence
Z
sec x dx = ln | sec x + tan x| + C.

4. y = ln | cosec x − cot x|.


Solution.
Å ã
1 d
y =
0
(cosec x − cot x)
cosec x − cot x dx
Å ã
1
= (− cosec x cot x + cosec2 x)
cosec x − cot x
Å ã
1
= cosec x(− cot x + cosec x)
cosec x − cot x
= cosec x.

This shows that


d
ln | cosec x − cot x| = cosec x.
dx
Thus ln | cosec x − cot x| is an antiderivative of cosec x, and hence
Z
cosec x dx = ln | cosec x − cot x| + C.
72 1st Semester Calculus Lecture Manual 2017 MATH1036

The derivative knowledge acquired in the previous chapter and the examples above provide a table of antideriva-
tives or indefinite integrals.

Z
e x dx = e x + C
Z
ax
a x dx = +C
ln a
Z
ln x dx = x ln x − x + C
Z
sin x dx = − cos x + C
Z
cos x dx = sin x + C
Z
tan x dx = ln | sec x| + C
Z
cosec x dx = ln | cosec x − cot x| + C
Z
sec x dx = ln | sec x + tan x| + C
Z
cot x dx = ln | sin x| + C
Z
sec2 x dx = tan x + C
Z
sec x tan x dx = sec x + C
Z
cosec2 x dx = − cot x + C

Theorem
R 4.3 (Properties
R of the indefinite
R integral).
1. [ f (x) + g(x)]dx = f (x) dx + g(x) dx
R R R
2. [ f (x) − g(x)]dx = f (x) dx − g(x) dx
R R
3. k f (x) dx = k f (x) dx for any k ∈ R.

Worked Example 4.1.4. Find the indefinite integrals of the following functions.
1. f (x) = 10x4 − 6x3 + 5.
Solution.
Z Z
f (x) dx = (10x4 − 6x3 + 5) dx
Z Z Z
= 10x4 dx − 6x3 dx + 5 dx
Z Z Z
= 10 x4 dx − 6 x3 dx + 5 1 dx
Å ã Å ã
1 5 1 4
= 10 x −6 x + 5x + C
5 4
3
= 2x5 − x4 + 5x + C.
2
MATH1036 1st Semester Calculus Lecture Manual 2017 73

2. g(u) = 2eu − 3 sin u + 2 cos u.


Solution.
Z Z
g(u) du = (2eu − 3 sin u + 2 cos u) du
Z Z Z
= 2e du − 3 sin u du + 2 cos u du
u

Z Z Z
= 2 eu du − 3 sin u du + 2 cos u du

= 2eu + 3 cos u + 2 sin u + C.

x2 + 3x + 1
3. h(x) = .
x
Solution.

x2 + 3x + 1
Z Z
h(x) dx = dx
x
Z Å ã
1
= x+3+ dx
x
Z Z Z
1
= x dx + 3 1 dx + dx
x
x2
= + 3x + ln |x| + C.
2
Note.
R
f (x) dx
Z
f (x)
dx , .
g(x)
R
g(x) dx

4. t(x) = x(3x2 + 1)2 .


Solution.
Z Z
t(x) dx = x(3x2 − 1)2 dx
Z
= x(9x4 − 6x2 + 1) dx
Z
= (9x5 − 6x3 + x) dx
Z Z Z
= 9 x5 dx − 6 x3 dx + x dx
Å 6ã Å 4ã
x x x2
=9 −6 + +C
6 4 2
3 3 1
= x6 − x4 + x2 + C.
2 2 2
Note.
Z ÅZ ã ÅZ ã
f (x)g(x) dx , f (x) dx g(x) dx .
74 1st Semester Calculus Lecture Manual 2017 MATH1036

5. s(t) = 2t + 3 sec t − ln t3 .

Solution.
Z Z
s(t) dt = (2t + 3 sec t − ln t3 ) dt
Z Z Z
= 2t dt + 3 sec t dt − ln t3 dt

2t
Z
= + 3 ln | sec t + tan t| − 3 ln t dt
ln 2
2t
= + 3 ln | sec t + tan t| − 3(t ln t − t) + C
ln 2
t
2
= + 3 ln | sec t + tan t| − 3t ln t + 3t + C.
ln 2

It is important to note that


Z
f 0 (x) dx = f (x) + C.

Worked Example 4.1.5. 1. Find f if f 0 (t) = 2 cos t − sec2 t, − π2 < t < π2 .

Solution.
Z Z
f (t) = 2 cos t − sec t ⇒
0 2
f (t) dt = (2 cos t − sec2 t) dt
0

Z Z
⇒ f (t) = 2 cos t dt − sec2 t dt

⇒ f (t) = 2 sin t − tan t + C.

2. Find g if g0 (x) = 12x2 − 6x + 1 and g(1) = 5.

Solution.
Z Z
g0 (x) = 12x2 − 6x + 1 ⇒ g0 (x) dx = (12x3 − 6x + 1) dx
Z Z Z
⇒ g(x) = 12 x dx − 6 x dx + 1 dx
2

Å 3ã Å 2ã
x x
⇒ g(x) = 12 −6 + x+C
3 2
⇒ g(x) = 4x3 − 3x2 + x + C.

Since g(1) = 5, we can solve for the constant C.

g(1) = 5 ⇒ 5 = 4(1)3 − 3(1)2 + 1 + C


= 4−3+1+C
⇒ C = 3.

Hence g(x) = 4x3 − 3x2 + x + 3.

Tutorial 4.1.1. 1. Thomas Exercises 11th ed. 4.8 pp. 338–340: 2, 3, 7, 36, 39, 40, 55, 65, 66, 72, 73, 75, 76, 77,
83, 86 or 12th ed. 4.8 pp. 285–286: 2, 3, 7, 36, 39, 40, 55, 65, 66, 72, 73, 75, 76, 77, 83, 86.
MATH1036 1st Semester Calculus Lecture Manual 2017 75

4.2 The Definite Integral & FTC


A definite integral is a particular limit involving continuous functions that we will encounter and evaluate in more
detail in Chapter 8, a limit of Riemann Sums.
Let f be a continuous function that is defined on a closed interval [a, b]. A partition of [a, b] is any decomposition
of [a, b] into subintervals of the form

[x0 , x1 ], [x1 , x2 ], [x2 , x3 ], . . . , [xn−1 , xn ]

where n is a positive integer and xi are numbers such that

a = x0 < x1 < x2 < x3 < · · · < xn−1 < xn = b.

The length of each subinterval [xi−1 , xi ] is denoted by ∆xi , i. e,

∆xi = xi − xi−1 .

We will mostly partition the interval [a, b] into n subintervals of equal length. Their common length is then
b−a
∆x =
n
and the points of subdivision are given by the formula

xi = a + i∆x.

The symbol for a definite integral and the limit that defines it are

Zb n
X
f (x) dx = lim f (xi∗ )∆x
n→∞
a i=1

where xi∗ is any sample point in the subinterval [xi−1 , xi ]. a and b are called the limits of integration, particularly
Rb
a is the lower limit and b the upper limit. Note that the limit exists and so f (x) dx ∈ R.
a
Rb
The process of finding the number represented by the limit defining f (x) dx is called integration or evaluating
a
the integral. In the notation of the definite integral, the letter x is a dummy variable; the symbol used for the
independent variable is immaterial. Thus

Zb Zb Zb
f (x) dx = f (t) dt = f (r) dr = . . . etc.
a a a

The definite integral and the indefinite integral (an antiderivative) are related by the Fundamental Theorem of
Calculus, part 2 (FTC2). The FTC2 provides a means of calculating the definite integral without evaluating the
limit that defines it. The FTC1 and FTC2 will be encountered in more detail in the second semester.

Theorem 4.4 (FTC2). If f is continuous on [a, b], then

Zb òb
f (x) dx = F(x) = F(b) − F(a),
a
a

where F is any antiderivative of f .

If f is non-negative and continuous on [a, b], then the definite integral corresponds to the area under the graph
of f .
76 1st Semester Calculus Lecture Manual 2017 MATH1036

Worked Example 4.2.1. Evaluate the integrals using FTC2.


R4 1
1. √ dx.
1 x
1
Solution. √ is continuous on the closed interval [1, 4], so the definite integral exists. By the FTC2
x
Z4 Z4
1 1
√ dx = x− 2 dx
x
1 1
1 ò4
x2
= 1
(2) 1
ò4

=2 x
1
√ √
=2 4−2 1
= 2(2) − 2(1) = 2.

R2 4 + x2
2. dx.
1 x3
Solution.
Z2 Z2 Å
4 + x2
ã
1
dx = 4x +
−3
dx
x3 x
1 1
î 4x−2 ó2
= + ln |x|
−2 1
î 2 ó2
= − 2 + ln |x|
x 1
Å ã Å ã
2 2
= − 2 + ln |2| − − 2 + ln |1|
2 1
1
= − + ln 2 + 2
2
3
= + ln 2.
2

π
R4
3. sec2 x dx.
− π4

Solution.
π
Z4 ó π4
sec2 x dx = tan x
− π4
− π4
π  π
= tan − tan −
4 4
= 1 − (−1)
= 2.

The following are properties of the definite integral.


MATH1036 1st Semester Calculus Lecture Manual 2017 77

Zb Za
f (x) dx = − f (x) dx
a b

Za
f (x) dx = 0
a

Zb Zb
c f (x) dx = c f (x) dx for constants c
a a

Zb Zb Zb
( f (x) + g(x)) dx = f (x) dx + g(x) dx
a a a

Zb Zb Zb
( f (x) − g(x)) dx = f (x) dx − g(x) dx
a a a

Tutorial 4.2.1. 1. Thomas Exercises 11th ed. 5.4 p. 392: 1, 2, 14, 23, 31, 32 or 12th ed. 5.4 p. 333: 1, 2, 6, 7,
12, 16, 20, 21, 23, 24, 33, 34.

4.3 The Substitution Rule


R
lt is clear from the FTC2 that the indefinite integral f (x) dx = F(x) + C greatly facilitates the evaluation of the
Rb
definite integral f (x) dx. When we differentiate a composite function we use the Chain Rule. In attempting
a
to calculate an indefinite integral, we often use the Chain Rule in reverse called the Substitution Rule or the
Subsitution Method.
Consider the Chain Rule in differentiation
d
F(g(x)) = F 0 (g(x))g0 (x).
dx
The Chain Rule thus says that F(g(x)) is an antiderivative of F 0 (g(x))g0 (x). This translates to the indefinite integral
Z
F 0 (g(x))g0 (x) dx = F(g(x)) + C.

If we change the variable or make the u-substitution u = g(x), we then reduce the indefinite integral to
Z Z
F 0 (g(x))g0 (x) dx = F(g(x)) + C = F(u) + C = F 0 (u) du.

Therefore a u-substitution may transform the original indefinite integral into one of the known forms in the table
of antiderivatives.

Theorem 4.5 (Indefinite Integral Substitution Rule). If u = g(x) is a differentiable function whose range is con-
tained in an interval I and f is continuous on I, then
Z Z
f (g(x))g (x) dx =
0
f (u) du.

HINT. If in the integrand we can identify functions in which the derivative of one is the other, then the Substitution
Rule is applicable to simplify the integral. We must choose an appropriate u-substitution, evaluate the differential

du = u0 (x) dx,
78 1st Semester Calculus Lecture Manual 2017 MATH1036

replace u0 (x) dx with du, and then substitute everywhere in the integral u = u(x), transforming the integral into one
of only u’s. Then use the indefinite integral table to solve the integral in terms of u and finally give the solution in
terms of x by back substituting for u in terms of x.

Worked Example 4.3.1. Use the Substitution Rule to evaluate the following indefinite integrals.
R
1. (3x + 2)7 dx.
Solution. Let u = 3x + 2. Then du = 3 dx or dx = 13 du. Substituting we get
Z Z
1
(3x + 2) dx = u7 du
7
3
Å 8ã
1 u
= +C
3 8
u8
= +C
24
(3x + 2)8
= + C substituting back for u to the original variable x.
24
R √3
2. x 7 − 6x2 dx.
Solution. Let u = 7 − 6x2 . Then du = −12x dx so that x dx = − 121
du. The integral reduces to
Z √
√3
Z Å ã
3 1
x 7 − 6x2 dx = u − du
12
Z
1 1
=− u 3 du
12
Ç 4å
1 u3
=− 4
+C
12 3
4
u3
=− +C
16
1 4
= − (7 − 6x2 ) 3 + C.
16
R
3. x2 sin x3 dx.
Solution. Let u = x3 . Then du = 3x2 dx and x2 dx = 31 du. Then
Z Z
1
x2 sin3 x dx = sin u du
3
1
= (− cos u) + C
3
1
= − cos x3 + C.
3
R cos x
4. √ dx.
1 + sin x

Solution. Let u = 1 + sin x. Then
1 cos x
du = √ (cos x) dx = √ dx.
2 1 + sin x 2 1 + sin x
cos x
Thus √ dx = 2 du. Hence
1 + sin x
Z
cos x
Z √
√ dx = 2 du = 2u + C = 2 1 + sin x + C.
1 + sin x
MATH1036 1st Semester Calculus Lecture Manual 2017 79


Note. Alternatively we may let u = 1 + sin x and continue. However, whenever a appears in the denominator
of the integrand, a neat substitution may be the square rooted function.
R
5. tan 4x sec2 4x dx.

Solution. Let u = tan 4x. Then du = 4 sec2 4x dx and thus sec2 4x dx = 14 du. Then
Z Z
1
tan 4x sec 4x dx =
2
u du
4
Å 2ã
1 u
= +C
4 2
u2
= +C
8
2
tan 4x
= + C.
8
Alternatively:
Z Z
tan 4x sec2 4x dx = tan 4x sec 4x sec 4x dx.

Let u = sec 4x. Then du = 4 sec 4x tan 4x dx and hence 1


4 du = sec 4x tan 4x dx. Then
Z Z
tan 4x sec2 4x dx = tan 4x sec 4x sec 4x dx
Z
1
= u du
4
Å 2ã
1 u
= +K
4 2
u2
= +K
8
sec2 4x 1 + tan2 4x tan2 4x
Å ã
= +K = +K = + C.
8 8 8

The Substitution Rule gives neat formulas for functions with multiples. For example, if k is a constant, then
Z
ekx dx = 1k ekx + C
Z
sin kx dx = − 1k cos kx + C
Z
cos kx dx = 1
k sin kx + C
Z
sec2 kx dx = 1
k tan kx + C
Z
sec kx tan kx dx = 1
k sec kx + C

The substitution u = kx will give the above solutions.

Worked Example 4.3.2. Evaluate


R R
1. e2x dx. Solution. e2x dx = 12 e2x + C.
R R
2. sin 5x dx. Solution. sin 5x dx = − 51 cos 5x + C.
80 1st Semester Calculus Lecture Manual 2017 MATH1036

Theorem 4.6 (Definite Integral Substitution Rule). If g is differentiable on [a, b], if g0 is continuous on [a, b], and
if f is continuous on the range of g, then

Zb Zg(b)
f (g(x))g (x) dx =
0
f (u) du.
a g(a)

Worked Example 4.3.3. Evaluate the following definite integrals.


R1
1. x(x2 + 1)3 dx.
0

Solution. Let u = x2 + 1. Change the variable and the limits to u. The limits (which apply to x) need to be changed
to limits with respect to u. x = 0 ⇒ u = 1 and x = 1 ⇒ u = 2. Also du = 2x dx so that 12 du = x dx. Then

Z1 Z2
1
x(x + 1) dx =
2 3
u3 du
2
0 1
ò4
1 u4
=
2 4 1
ò4
u4
=
8 1
1 4
= (2 − 14 )
8
15
= .
8
Alternatively: We can evalute the indefinite integral and then solve the definite integral using the original x limits
of integration.
Let u = x2 + 1. Then du = 2 dx and
Z Z
1
x(x2 + 1)3 dx = u3 du
2
1 u4
= +C
2 4
u4
= +C
8
1
= (x2 + 1)4 + C.
8
Z1 ò1
1
⇒ x(x2 + 1)3 dx = (x2 + 1)4
8 0
0
1 4
= (2 − 14 )
8
15
= .
8
This alternatively approach is at times the more preferred as we may need to make more that one subsitution to
resolve the integral.
MATH1036 1st Semester Calculus Lecture Manual 2017 81

π
R3 sin θ
2. dθ.
0 cos2 θ
π
Solution. Let u = cos θ. Then du = − sin θ dθ. Also, θ = 0 ⇒ u = cos 0 = 1 and θ = 3 ⇒ u = cos π3 = 12 . Then
π 1
Z3 Z2
sin θ du
dθ = −
cos2 θ u2
0 1
Å ã 12
1
=− −
u 1
= (2 − 1)
= 1.


3. sin2 x dx.
−π
Solution.
Z Z
1 − cos 2x
sin x dx =
2
dx
2
ÅZ Z ã
1
= dx − cos 2x dx
2
Å ã
1 1
= x − sin 2x + C
2 2
x sin 2x
= − +C
2 4

x sin 2x π
ï ò
⇒ sin x dx =
2

2 4 −π
−π
π sin 2π
Å ã
−π sin(−2π)
= − − −
2 4 2 4
π π
= −0+ −0
2 2
= π.
Tutorial 4.3.1. 1. Thomas Exercises 11th ed. 5.5 pp. 402–403: 2, 7, 10, 19, 24, 34, 40, 55-57, 67 or 12th ed. 5.5
pp. 342–344: 3, 4, 5, 8, 11, 14, 21, 26, 36, 52, 67-69, 79.
2. Thomas Exercises 11th ed. 5.6 pp. 410–414: 1, 2, 4, 5, 8, 13, 23, 29, 31, 32, 37, 40, 111, 112, 115, 116 or 12th
ed. 5.6 pp. 350–353: 1, 2, 4, 5, 8, 13, 23, 29, 31, 32, 37, 40, 111, 112, 113, 115, 116.

4.4 Integration by Parts


By the Product Rule for differentiation, if f and g are differentiable functions, then
( f g)0 (x) = f 0 (x)g(x) + f (x)g0 (x).
This gives that the function f (x)g(x) is an antiderivative of f 0 (x)g(x) + f (x)g0 (x). In terms of the indefinite integral
we have
Z
[ f 0 (x)g(x) + f (x)g0 (x)] dx = f (x)g(x) + C
Z Z
⇒ f 0 (x)g(x) dx + f (x)g0 (x) dx = f (x)g(x) + C
Z Z
⇒ f (x)g (x) dx = f (x)g(x) −
0
f 0 (x)g(x) dx + C.
82 1st Semester Calculus Lecture Manual 2017 MATH1036

R
Since the computation of f 0 (x)g(x) dx will yield its own constant, there is no reason to keep the constant C. We
therefore drop it and write Z Z
f (x)g0 (x) dx = f (x)g(x) − f 0 (x)g(x) dx
or Z Z
f (x)g0 (x) dx = f (x)g(x) − g(x) f 0 (x) dx.

The above formula is called the formula for integration by parts. Integration by parts (IBP) is the integration
analogue of the Product Rule for differentiation. Introducing the notations u = f (x) and v = g(x), the differentials
are du = f 0 (x) dx and dv = g0 (x) dx and we have the following integration by parts formula.

Theorem 4.7 (Integration by parts, IBP). If u = f (x) and v = g(x) where f 0 and g0 are continuous, then
Z Z
u dv = uv − v du.

Alternatively, we observe the product rule for differentials


d(uv) = u dv + v du
implies that Z Z
u dv + v du = uv,

and we get the IBP formula Z Z


u dv = uv − v du.

Note that we still integrate with respect to x. u, du, v, dv are only shortcuts for f (x), f 0 (x) dx, g(x), g0 (x) dx,
respectively.

Worked Example 4.4.1. Use IBP to solve the following integrals.


R
1. xe x dx.
Solution. Let
u=x and dv = e x dx
Z
du = dx and v = e x dx = e x .

The IBP formula yields


Z Z
xe x dx = u dv
Z
= uv − v du
Z
= xe x − e x dx

= xe x − e x + C
= e x (x − 1) + C.

R
2. ln x dx.

Solution. Let
u = ln x and dv = dx
Z
1
⇒ du = dx and v = dx = x.
x
MATH1036 1st Semester Calculus Lecture Manual 2017 83

The IBP formula then gives


Z Z
ln x dx = u dv
Z
= uv − v du
Z Å ã
1
= x ln x − x dx
x
Z
= x ln x − dx

= x ln x − x + C
= x(ln x − 1) + C.

Hence, we proved
Z
ln x dx = x(ln x − 1) + C.

R
3. e x cos x dx.
R
Solution. Let I = e x cos x dx and let

u = ex and dv = cos x dx
Z
⇒ du = e x dx and v = cos x dx = sin x.

IBP gives
Z Z Z Z
I= e x cos x dx = u dv = uv − v du = e x sin x − e x sin x dx.
R
We next consider e x sin x dx. For this integral we let

u = ex and dv = sin x dx
Z
⇒ du = e x dx and v = sin x dx = − cos x.

IBP for this integral gives


Z Z Z Z
e x sin x dx = u dv = uv − v du = −e x cos x − e x (− cos x) dx = −e x cos x + I + K.

Substituting back in the first equation we get


Z
I = e x sin x − e x sin x dx

= e x sin x − [−e x cos x + I + K]


= e x sin x + e x cos x − I − K
⇒ 2I = e x sin x + e x cos x − K
1
⇒ I = e x (sin x + cos x) + C.
2
R
Hence, IBP twice yields e x cos x dx = 21 e x (sin x + cos x) + C.
84 1st Semester Calculus Lecture Manual 2017 MATH1036

R
4. sec3 x dx.
Solution. Let I =
R
sec3 x dx. Then
Z
I= sec3 x dx
Z
= sec x sec2 x dx.

Let

u = sec x and dv = sec2 x dx


Z
⇒ du = sec x tan x dx and v = sec2 x dx = tan x.

IBP gives
Z
I= u dv
Z
= uv − v du
Z
= sec x tan x − sec x tan x tan x dx
Z
= sec x tan x − sec x(sec2 x − 1) dx
Z
= sec x tan x − I + sec x dx

= sec x tan x − I + ln | sec x + tan x| + K.

Thus

2I = sec x tan x + ln | sec x + tan x| + K


1
⇒ I = (sec x tan x + ln | sec x + tan x|) + C.
2
Z
Hence, sec3 x dx = 12 (sec x tan x + ln | sec x + tan x|) + C.

Evaluating a definite integral using IBP leads to the following


Zb òb Zb
u dv = uv − v du.
a
a a

Alternatively, the indefinite integral may first be solved and then using that solution we may evaluate the definite
integral.

Worked Example 4.4.2. Evaluate


π
Z2
x cos x dx.
0

Solution. Let

u=x and dv = cos x dx


Z
⇒ du = dx and v = cos x dx = sin x.
MATH1036 1st Semester Calculus Lecture Manual 2017 85

IBP gives
π π
Z2 ò π2 Z2
x cos x dx = x sin x − sin x dx
0
0 0
π ò π2
π  ï
= sin − 0 sin 0 − − cos x
2 2 0
ò π2
π
= (1) + cos x
2 0
π π
= + cos − cos 0
2 2
π
= +0−1
2
π
= − 1.
2
Tutorial 4.4.1. 1. Thomas Exercises 11th ed. 8.2 pp. 552–554: 1, 6, 9, 12, 14, 25, 26, 29, 30 or 12th ed. 8.1 pp.
459–460: 1, 6, 11, 13, 16, 18, 25, 26, 29, 30.
Chapter 5

Differentiation Techniques

LEARNING OUTCOMES:
On completion of this chapter you should (tick the checkbox when you have mastered the skill)
 1. be able to find the higher order derivatives of a given function,
 2. be able to find the n -degree Taylor poynomial of a given function at a given point,
th

 3. be able to find the derivative of the inverse f of a given invertible function f at a given point without
−1
−1

deriving the formula for f ,


 4. be able to find and know the proofs of the derivatives (and integrals) of the inverse trigonometric func-
tions,
 5. be able to use the technique of logarithmic differentiation to evaluate derivatives involving complicated
products, quotients or powers,
 6. know how to solve the examinable tutorial problems and the worked out problems from these notes on
this chapter.

5.1 Higher Order Derivatives


Since the derivative f 0 of a function f is itself a function, we may consider successive derivatives of functions.
The derivative of the first derivative is called the second derivative, the derivative of the second derivative is the
third derivative, etc. We use the following notations for y = f (x).

dy
1st derivative y0 = f 0 y(1) = f (1)
dx
d2 y
Å ã
d dy
2nd derivative = 2 y00 = f 00 y(2) = f (2)
dx dx dx
d d y d3 y
Å 2 ã
3rd derivative = y000 = f 000 y(3) = f (3)
dx dx2 dx3
.. .. ..
. . .
d d y dn y
Å n−1 ã
nth derivative n−1
= n y(n) = f (n)
dx dx dx

Worked Example 5.1.1. 1. Find the first four derivatives of


3
(a) y = 4x2 − 5x + 8 − .
x

86
MATH1036 1st Semester Calculus Lecture Manual 2017 87

Solution.
3
y = 4x2 − 5x + 8 − ⇒ y = 4x2 − 5x + 8 − 3x−1
x
⇒ y0 = 8x − 5 + 3x−2
⇒ y00 = 8 − 6x−3
⇒ y000 = 18x−4
⇒ y(4) = −72x−5 .

(b) f (x) = sin x.


Solution.

f (x) = sin x ⇒ f 0 (x) = cos x


⇒ f 00 (x) = − sin x
⇒ f 000 (x) = − cos x
⇒ f (4) (x) = sin x

1
(c) h(x) = . Hence find a formula for the nth derivative h(n) (x).
x
Solution.
1
h(x) = ⇒ h(x) = x−1
x
⇒ h(1) (x) = −x−2
⇒ h(2) (x) = 2(1)x−3
⇒ h(3) (x) = −3(2)(1)x−4
⇒ h(4) (x) = 4(3)(2)(1)x−3
⇒ h(5) (x) = −5(4)(3)(2)(1)x−4
..
.
⇒ h(n) (x) = (−1)n n(n − 1)(n − 2)(n − 3) · · · (3)(2)(1)x−(n+1) .

Using the factorial symbol n! for the product of n consecutive positive integers

n! = n(n − 1)(n − 2)(n − 3) · · · (3)(2)(1)

we get the formula for the nth derivative


dn 1 (−1)n n!
= .
dxn x xn+1

2. Find y00 if x2 − 3xy + y2 = 4.


Solution.We differentiate implicitly.

x2 − 3xy + y2 = 4 ⇒ 2x − 3(y + xy0 ) + 2yy0 = 0


⇒ 2x − 3y − 3xy0 + 2yy0 = 0
⇒ y0 (2y − 3x) = 3y − 2x
3y − 2x
⇒ y0 = .
2y − 3x
88 1st Semester Calculus Lecture Manual 2017 MATH1036

Then
(3y0 − 2)(2y − 3x) − (3y − 2x)(2y0 − 3)
y00 =
(2y − 3x)2
6yy − 9xy − 4y + 6x − 6yy0 + 4xy0 + 9y − 6x
0 0
=
(2y − 3x)2
0
5y − 5xy
=
(2y − 3x)2
Å ã
3y − 2x
5y − 5x
2y − 3x 3y − 2x
= ∵ y0 =
(2y − 3x)2 2y − 3x
10y2 − 15xy − 15xy + 10x2
=
(2y − 3x)3
y2 − 3xy + x2
= 10
(2y − 3x)3

If f is any function that has n derivatives at x = a, we can form the nth -degree Taylor polynomial of f about a

f 00 (a)(x − a)2 f 000 (a)(x − a)3 f (n) (a)(x − a)n


T n (x) = f (a) + f 0 (a)(x − a) + + + ··· +
2! 3! n!
n
X f (k) (a)(x − a)k
= .
k=0
k!

Note that the first-order Taylor poynomial of f about a is

T 1 (x) = f (a) + f 0 (a)(x − a)

so that the graph of T 1 is the tangent line at a.

Worked Example 5.1.2. 1. If f (x) = ln x, find the fourth degree Taylor polynomial at x = 1.

Solution.

f (x) = ln x
1
⇒ f 0 (x) = = x−1 ⇒ f 0 (1) = 1
x
⇒ f 00 (x) = −x−2 ⇒ f 00 (1) = −1
⇒ f 000 (x) = 2x−3 ⇒ f 000 (1) = 2
⇒ f (4) (x) = −6x−4 ⇒ f (4) (1) = −6.

Thus the Taylor polynomial of degree 4 of f (x) = ln x at x = 1 is

f 00 (1)(x − 1)2 f 000 (1)(x − 1)3 f (4) (1)(x − 1)4


T 4 (x) = f (1) + f 0 (1)(x − 1) + + +
2! 3! 4!
(−1)(x − 1)2 2(x − 1)3 (−6)(x − 1)4
= ln 1 + (1)(x − 1) + + +
2 6 24
(x − 1)2 (x − 1)3 (x − 1)4
= (x − 1) − + − .
2 3 4
MATH1036 1st Semester Calculus Lecture Manual 2017 89

2. Find the Taylor polynomial of degree n of f (x) = e x at x = 0.

Solution. Since f (n) (x) = e x for each n, f (n) (0) = e0 = 1. Hence the Taylor polynomial of f (x) = e x at x = 0 is

f 00 (0)x2 f 000 (0)x3 f (n) (0)xn


T n (x) = f (0) + f 0 (0)x + + + ··· +
2! 3! n!
x2 x3 xn
=1+x+ + + ··· +
2! 3! n!
n
X xk
= .
k=0
k!

Taylor polynomials will be studied in more detail in the last block of the second semester in Chapter 11 of these
notes.

Tutorial 5.1.1. 1. Thomas Exercises 11th ed. 3.2 p. 167: 29, 31, 33, 35, 36 or 12th ed. 3.3 p. 143: 41, 45, 47,
51, 52.
2. Thomas Exercises 11th ed. 3.4 p. 186: 25 or 12th ed. 3.5 p. 160: 33.
3. Thomas Exercises 11th ed. 3.6 p. 209: 39–44 or 12th ed. 3.7 p. 174: 23–26.
4. Thomas Exercises 11th ed. 11.8 p. 794: 2, 4, 5, 7, 8 or 12th ed. 10.8 p. 606: 2, 4, 6, 7, 9, 10.

5.2 Continuity and Differentiation of Inverse Functions


Recall that a function f is invertible if and only if f is a one-to-one function, i. e.,

f (x) = f (y) ⇒ x = y,

equivalently,
x , y ⇒ f (x) , f (y).

For such functions the inverse f −1 is defined by

f −1 (y) = x ⇔ y = f (x).

In particular, e x and ln x are invertible and are inverses of each other.

Theorem 5.1. If f is a one-to-one continuous function defined on an interval, then its inverse function f −1 is also
continuous.

Given a one-to-one differentiable function f whose derivative is never zero, the inverse function is also differen-
tiable in its domain and its derivative may be found at given points without actually deriving the formula for f −1
and then finding its derivative ( f −1 )0 , as illustrated by the following theorem.

Theorem 5.2. If f is a one-to-one differentiable function with inverse function f −1 and f 0 ( f −1 (a)) , 0, then the
inverse function is differentiable at a and

1
( f −1 )0 (a) = .
f 0 ( f −1 (a))

Worked Example 5.2.1. 1. Let f (x) = x − 2. Find ( f −1 )0 (2) by
(a) first finding a formula for f −1 , then differentiating to get a formula for ( f −1 )0 (x) and finally evaluating ( f −1 )0 (2),

(b) using Theorem 5.2 and confirm that (a) and (b) agree.
90 1st Semester Calculus Lecture Manual 2017 MATH1036

Solution.(a) The domain of f is dom( f ) = {x ∈ R : x ≥ 2} = [2, ∞) and the range of f is [0, ∞). Since f is a
one-to-one function ( f (x) = f (y) ⇒ x = y), f −1 exists with domain [0, ∞) and range [2, ∞). Let y = f −1 (x). Then

y = f −1 (x) ⇒ x = f (y)
⇒ x= y−2
p

⇒ x2 = y − 2
⇒ y = x2 + 2
⇒ f −1 (x) = x2 + 2.

Then ( f −1 )0 (x) = 2x, x > 0. Thus ( f −1 )0 (2) = 2(2) = 4.


(b) Since f −1 exists and 2 ∈ dom( f −1 ),

f −1 (2) = x ⇔ f (x) = 2 ⇔ x − 2 = 2 ⇔ x = 6.

1
Thus f −1 (2) = 6. Since f 0 (x) = √ ,
2 x−2
1 1
f 0 ( f −1 (2)) = f 0 (6) = √ = , 0.
2 4 4
Thus by Theorem 5.2,
1
( f −1 )0 (2) = = 4.
f 0 ( f −1 (2))

2. If g(x) = 2x + e x , show that g is invertible and then find (g−1 )0 (1) without finding the formula for g−1 .
Solution. The function 2x is strictly increasing, and so is the function e x . Thus also the sum, g, is a strictly
increasing function, i. e.,

x < y ⇒ 2x < 2y and e x < ey


⇒ 2x + e < 2y + ey .
x

Hence g is one-to-one, and g−1 exists. Then

1
(g−1 )0 (1) =
g0 (g−1 (1))

if g0 (g−1 (1)) , 0. Now

g−1 (1) = x ⇔ g(x) = 1


⇔ 2x + e x = 1
⇔x=0 (by inspection)

Thus g−1 (1) = 0. Hence


1 1 1 1
(g−1 )0 (1) = = = = .
g0 (g−1 (1)) g0 (0) 2+e0 3
Tutorial 5.2.1. 1. Thomas Exercises 11th ed. 3.7, p. 221: 3, 4, 7, 9 or 12th ed. 3.8 p. 184: 3, 4, 7, 9.

5.3 Inverse Trigonometric Derivatives and Integrals


Recall the definitions and properties of the inverse trigonometric functions done in Algebra. In this section we
derive the derivative formulas for the inverse trigonometric functions arcsin x, arccos x and arctan x. We will also
incorporate the indefinite integrals of these functions. All proofs are examinable.
MATH1036 1st Semester Calculus Lecture Manual 2017 91

Theorem 5.3 (Derivative of arcsin x).

d 1
arcsin x = √ , −1 < x < 1.
dx 1 − x2

Proof. Let y = arcsin x. Then


π π
y = arcsin x ⇔ sin y = x and − ≤y≤
2 2
dy
⇒ cos y =1 by implicit differentiation
dx
π π
Since cos y > 0 for − < y < and sin2 y + cos2 y = 1, we get
2 2
» √
cos y = | cos y| = 1 − sin2 y = 1 − x2 ∵ sin y = x.

Hence
dy dy 1 1
cos y =1⇒ = = √ .
dx dx cos y 1 − x2

d 1
Hence arcsin x = √ with −1 < x < 1. 
dx 1 − x2

Incorporating the Chain Rule gives

d f 0 (x)
arcsin f (x) = p .
dx 1 − [ f (x)]2

1
The above theorem informs that √ is an antiderivative of arcsin x so that the associated indefinite integral
1 − x2
is
Z
1
√ dx = arcsin x + C.
1 − x2

Worked Example 5.3.1. 1. Find y0 if



(a) y = x arcsin 2x + 21 1 − 4x2 .

Solution.
Ç å Å ã
1 1 1
y = arcsin 2x + x
0
p (2) + √ (−8x)
1 − (2x) 2 2 2 1 − 4x2
2x 2x
= arcsin 2x + √ − √
1 − 4x2 1 − 4x2
∴ y0 = arcsin 2x.
92 1st Semester Calculus Lecture Manual 2017 MATH1036

Å ã
x
(b) y = arcsin √ .
1 + x2
Solution.
Å ã
x
y = arcsin √
1 + x2
Å ã
1 d x
⇒ y0 = √
1 + x2
Å ã2 dx
x
1− √
1 + x2
Ü√
2x2
ê
1 + x2 − √
√ 2 1+x
1 2
= …
1 ( 1+x ) 2 2

1 + x2
√ +
Ç å
2 2
2(1 x ) − 2x
= 1 + x2 √
2(1 + x2 ) 1 + x2
1
= .
1 + x2

Alternatively:
Å ã
x
y = arcsin √
1 + x2
x
⇔ sin y = √
1 + x2

⇒ 1 + x2 sin y = x
2x √
⇒ √ sin y + 1 + x2 (cos y)y0 = 1 Implicit differentiation
2 1 + x2

Since − π2 < y < π2 , cos y > 0, and therefore

» x2 1
cos y = 1 − sin2 y = 1− = √ ,
1 + x2 1 + x2

we conclude

x x √ 1
√ √ + 1 + x2 √ y0 = 1
1+x 2 1+x 2 1 + x2
x2
⇒ + y0 = 1
1 + x2
x2
⇒ y0 = 1 −
1 + x2
1
∴ y0 = .
1 + x2
MATH1036 1st Semester Calculus Lecture Manual 2017 93

2. y = arcsin(cos x).
Solution. For | cos x| < 1, i. e., x , kπ, k ∈ Z, we have
dy 1 d
y = arcsin(cos x) ⇒ = √ cos x
dx 1 − cos x 2 dx
1
= √ (− sin x)
sin2 x
sin x
=−
| sin x|
−1 if sin x > 0,
(
=
1 if sin x < 0.

This shows that the graph of arcsin(cos x) is saw-tooth like, i. e., it is composed of line segments with slopes
alternating between 1 and −1.
A direct computation of the graph of y = arcsin(cos x) is given below.

y = arcsin(cos x) ⇒ sin y = cos x


 π
⇔ cos y − = cos x
2
π
⇔ y − = ±x + 2kπ, k ∈ Z.
2
π
Since y − ∈ [−π, 0], we therefore have
2
π
 − x + 2kπ
 if x ∈ [2kπ, (2k + 1)π), k ∈ Z,
2
y=
π
 + x − 2kπ if x ∈ [(2k − 1)π, 2kπ), k ∈ Z.

2
Below is the graph of y = arcsin(cos x).

−3π −2π −π π 2π 3π

3. Evaluate
1
Z4
1
√ dx.
1 − 4x2
0

Solution. First consider the indefinite integral. Let u = 2x. Then du = 2dx so that
Z Z
1 1 1
√ dx = √ du
1 − 4x2 2 1 − u2
1
= arcsin u + C
2
1
= arcsin 2x + C.
2
94 1st Semester Calculus Lecture Manual 2017 MATH1036

Then
1
Z4 ò 14
1 1
√ dx = arcsin 2x
1 − 4x2 2 0
0
ï Å ã ò
1 1
= arcsin − arcsin 0
2 2
1 hπ i
= −0
2 6
π
= .
12
4. Prove that
Z
1  x
√ dx = arcsin + C, a > 0
a2 − x2 a

and then evaluate Z


1
√ dx.
3 − x2
x
Proof. Let u = . Then au = x so that a du = dx. Then
a
Z Z
1 1
√ dx = a √ du
2
a −x 2 a 2 − a2 u2
Z
1
=a p du
a2 (1 − u2 )
Z
1
=a √ du
a 1 − u 2
Z
1
= √ du
1 − u2
= arcsin u + C
 x
= arcsin + C.
a
By the above formula, Z Å ã
1 x
√ dx = arcsin √ + C. 
3 − x2 3
5. Use integration by parts to prove
Z √
arcsin x dx = x arcsin x + 1 − x2 + C.

Proof. Let
u = arcsin x and dv = dx
Z
1
⇒ du = √ dx and v = dx = x.
1 − x2
Then
Z Z
arcsin x dx = u dv
Z
= uv − v du
Z
x
= x arcsin x − √ dx.
1 − x2
MATH1036 1st Semester Calculus Lecture Manual 2017 95

√ −2x
Now let w = 1 − x2 . Then dw =√ dx so that
2 1 − x2
Z
x
Z √
√ dx = − dw = −w + C = − 1 − x2 + C.
1 − x2
Hence
Z Z
x
arcsin x dx = x arcsin x − √ dx
1 − x2
Ä √ ä
= x arcsin x − − 1 − x2 + C

= x arcsin x + 1 − x2 + C.

Note. For arcsin 2x let u = 2x. Then du = 2 dx and


Z Z
1
arcsin 2x dx = arcsin u du
2
1Ä √ ä
= u arcsin u + 1 − u2 + C
2
u 1√
= arcsin u + 1 − u2 + C
2 2
1√
= x arcsin 2x + 1 − 4x2 + C.
2
Compare this with Worked Example 5.3.1.1(a) above!

Theorem 5.4 (Derivative of arccos x).

d 1
arccos x = − √ .
dx 1 − x2

Proof. Instead of proving the derivative by implicit differentiation, we will first prove the identity
π
arcsin x + arccos x = .
2
√ √
Let A = arcsin x and B = arccos x. Then A ∈ [− π2 , π2 ] so that cos A ≥ 0. Thus cos A = 1 − sin2 A = 1 − x2 .
√ √
Since B ∈ [0, π], sin B ≥ 0 so that sin B = 1 − cos2 B = 1 − x2 . Then

sin(arcsin x + arccos x) = sin(A + B)


= sin A cos B + cos A sin B
√ √
= (x)(x) + 1 − x2 1 − x2
= x2 + (1 − x2 )
= 1.

π 3π π
ï ò
Since A + B ∈ − , , arcsin x + arccos x = . Differentiating gives
2 2 2

1 d
√ + arccos x = 0
1− x2 dx
d 1
∴ arccos x = − √ . 
dx 1 − x2
96 1st Semester Calculus Lecture Manual 2017 MATH1036

Worked Example 5.3.2. Find y0 if x2 + x arccos y = ye x .

Solution.

x2 + x arccos y = ye x
Ç å
1
⇒ 2x + arccos y + x − p y = y0 e x + ye x
0
1 − y2
Ç å
x
⇒ 2x + arccos y − ye = px
+ e y0
x
1 − y2
x + e x 1 − y2
Ç p å
⇒ y0 = 2x + arccos y − ye x
1 − y2
p

1 − y2 (2x + arccos y − ye x )
p
⇒ y0 = .
x + e x 1 − y2
p

The integral of arccos x rnay be omitted as integrals of that type can be realised in terrns of arcsin x. The more
preferred derivative and integral is that of arcsin x.

Theorem 5.5 (Derivative of arctan x).

d 1
arctan x = .
dx 1 + x2

Proof.

y = arctan x ⇔ tan y = x
⇒ (sec2 y)y0 = 1
1
⇒ y0 =
sec2 y
1
⇒ y0 =
1 + tan2 y
d 1
∴ arctan x = . 
dx 1 + x2

Incorporating the Chain Rule,

d f 0 (x)
arctan f (x) = ,
dx 1 + [ f (x)]2

and the corresponding indefinite integral is

Z
1
dx = arctan x + C.
1 + x2

Worked Example 5.3.3. 1. Find y0 if


x+1
(a) y = arctan .
x−1
MATH1036 1st Semester Calculus Lecture Manual 2017 97

Solution.

x+1
Å ã
2
y = arctan = arctan 1 +
x−1 x−1
Å ã
1 d 2
⇒y =
0
1+
x + 1 2 dx x−1
Å ã
1+
x−1
(x − 1)2
ï ò
−2
=
(x − 1)2 + (x + 1)2 (x − 1)2
2
=− 2
2x + 2
1
=− 2 .
x +1

(b) y = ln arctan x2 .

Solution.

y = ln arctan x2
Å ã
1 d
⇒ y0 = arctan x2
arctan x2 dx
Å ãÅ ã
1 1
= 2x
arctan x2 1 + (x2 )2
2x
= .
(1 + x4 ) arctan x2

2. Evaluate
Z1
ex
dx.
1 + e2x
0

Solution. Let u = e x . Then du = e x dx, x = 0 ⇒ u = e0 = 1 and x = 1 ⇒ u = e1 = e. Then

Z1 Ze
ex 1
dx = du
1 + e2x 1 + u2
0 1
òe
= arctan u
1
= arctan e − arctan 1
π
= arctan e − .
4

3. Prove that
Z
1 1  x
dx = arctan + C.
a2 + x2 a a
98 1st Semester Calculus Lecture Manual 2017 MATH1036

x
Proof. Let u = . Then au = x so that a du = dx. Then
a
Z Z
1 1
dx = a du
a +x
2 2 a + a2 u2
2
Z
1
=a du
a2 (1 + u2 )
Z
1 1
= du
a 1 + u2
1
= arctan u + C
a
1  x
= arctan + C. 
a a
4. Find
x+9
Z
dx.
x2 + 9
Solution.
x+9
Z Z Z
x 9
dx = dx + dx
x2 + 9 x2 + 9 x2 + 9
ï  x ò
1
= I+9 arctan .
3 3
For I, let u = x2 + 9. Then du = 2x dx and
Z Z
x 1 1 1 1
I= dx = du = ln |u| + C = ln |x2 + 9| + C.
x2 + 9 2 u 2 2
Hence
x+9
Z
1  x
dx = ln(x 2
+ 9) + 3 arctan + C.
x2 + 9 2 3

5. Use integration by parts to prove that


Z
1
arctan x dx = x arctan x − ln(1 + x2 ) + C.
2

Proof. Let
u = arctan x and dv = dx
Z
1
⇒ du = dx and v = dx = x.
1 + x2
Then
Z Z
arctan x dx = u dv
Z
= uv − v du
Z
x
= x arctan x − dx
1 + x2
Z
1 1
= x arctan x − dx let w = 1 + x2
2 w
1
= x arctan x − ln |w| + C
2
1
= x arctan x − ln(1 + x2 ) + C ∵ 1 + x2 > 0 
2
MATH1036 1st Semester Calculus Lecture Manual 2017 99

Tutorial 5.3.1. 1. Thomas Exercises 11th ed. 3.8 pp. 230–231: 49, 50, 61, 62, 64, 65, 67, 68 or 12th ed. 3.9 p.
191: 21, 22, 33, 34, 36, 37, 39, 40.
2. Thomas Exercises 11th ed. 5.5 p. 403: 49, 50, 53, 54 or 12th ed. 5.5 p. 343: 61, 62, 65, 66.
3. Thomas Exercises 11th ed. 8.6 p. 584: 23, 24, 31, 32 or 12th ed. 8.5 p. 485: 17–20. For each integral, use
appropriate methods to find the antiderivates.

5.4 Logarithmic Differentiation


Given y = f (x), y0 can also be found by the process called logarithmic differentiation. This method is especially
useful if f (x) involves complicated products, quotients, or powers. The properties of the (natural) logarithm allows
simplification of f (x).

The following outlines the technique of logarithmic differentiation.

Process to follow in logarithmic differentiation

1. Given:
y = f (x)

2. Take the natural logarithms on both sides and simplify using properties of the natural logarithm:

ln y = ln f (x)

3. Differentiate implicitly with respect to x:

d d y0 d
ln y = ln f (x) ⇒ = ln f (x)
dx dx y dx

4. Solve for y0 by cross-multiplication:


Å ã
d
y =y
0
ln f (x)
dx

Worked Example 5.4.1. Use the method of logarithmic differentiation to find y0 if


x2 (x − 1)3
1. (Complicated quotient): y = .
x+1
Solution.
x2 (x − 1)3
y=
x+1
Å 2
x (x − 1)3
ã
⇒ ln y = ln
x+1
= ln(x (x − 1)3 ) − ln(x + 1)
2

= ln x2 + ln(x − 1)3 − ln(x − 1)


= 2 ln x + 3 ln(x − 1) − ln(x + 1)

Differentiation gives
y0
Å ã Å ã
1 1 1
=2 +3 −
y x x−1 x+1
100 1st Semester Calculus Lecture Manual 2017 MATH1036

Hence
2(x − 1)(x + 1) + 3x(x + 1) − x(x − 1)
ï ò
y =y0
x(x − 1)(x + 1)
4x + 4x − 2
2
ï ò
=y
x(x − 1)(x + 1)
x (x − 1)3 4x2 + 4x − 2
2 ï ò
=
x+1 x(x − 1)(x + 1)
2x(x − 1) (2x2 + 2x − 1)
2
∴ y0 = .
(x + 1)2
√ 2
2. (Complicated product): y = xe x (x2 + 1)10 .
Solution.
√ 2
xe x (x2 + 1)10
y=
√ 2
ln y = ln x + ln e x + ln(x2 + 1)10
1
= ln x + x2 ln e + 10 ln(x2 + 1)
2
1
= ln x + x2 + 10 ln(x2 + 1)
2Å ã
y0 1 1
Å ã
2x
⇒ = + 2x + 10 2
y 2 x x +1
ï ò
1 20x
∴y =y
0
+ 2x + 2
2x x +1
√ x2 2
ï ò
1 20x
= xe (x + 1)10 + 2x + 2 .
2x x +1
x
3. (Complicated power): y = 2 x .
Solution.
x
y = 2x
x
⇒ ln y = ln 2 x
= x x ln 2
y 0
d
⇒ = ln 2 x x .
y dx
Now apply logarithmic differentiation to z = x x .
ln z = ln x x
= x ln x
z0
Å ã
1
⇒ =x + ln x
z x
= 1 + ln x
∴ z0 = z(1 + ln x)
d x
⇒ x = x x (1 + ln x).
dx
Then
y0 d
= ln 2 x x = (ln 2)(x x )(1 + ln x)
y dx
∴ y0 = y(ln 2)(x x )(1 + ln x)
x
= 2 x (ln 2)(x x )(1 + ln x).
MATH1036 1st Semester Calculus Lecture Manual 2017 101

Theorem 5.6 (General Power Rule).


d r
∀r ∈ R x = rxr−1
dx

Proof. Let y = xr . Using logarithmic differentiation we have

ln |y| = ln |xr |
= ln |x|r
= r ln |x|
y0
r
∴ =
y x
ry
⇒ y0 =
x
rxr
=
x
Hence
d r
x = rxr−1 . 
dx
Tutorial 5.4.1. 1. Thomas Exercises 11th ed. 3.7 pp. 221–222: 41, 42, 49, 51–54, 89–96 or 12th ed. 3.8 pp.
184–185: 41, 42, 49, 51–54, 89–96.
Chapter 6

Applications of Differentiation

ln this chapter we will consider some of the applications of the knowledge of the various differentation techniques
that you have acquired in the previous chapters.

6.1 Indeterminate Forms and l’Hôpital’s Rule

LEARNING OUTCOMES:
On completion of this section you should be able to (tick the checkbox when you have mastered the skill)
 1. identify an indeterminate form of a limit,
 2. use l’Hôpitals Rule in evaluating limits of indeterminate forms , , , 0 · ∞, 0 , ∞ , ∞ − ∞,
0
0


1

0 0

 3. know how to solve the examinable tutorial problems and the worked out problems from the notes on this
section.

Theorem 6.1 (l’Hôpital’s Rule). Suppose that f and g are differentiable functions and g0 , 0 near x = a (except
possibly at a). Suppose that
lim f (x) = 0 and lim g(x) = 0
x→a x→a
or that
lim f (x) = ±∞ and lim g(x) = ±∞.
x→a x→a
0 ∞
(In other words, we have an indeterminate form of the type 0 or ∞ ). Then
0
f (x) f (x)
lim = lim 0
x→a g(x) x→a g (x)
if the limit on the right hand side exists (or is ∞ or −∞).

The theorem also applies to one-sided limits as well.

2x2 − 2
Worked Example 6.1.1 (Type 00 ). 1. lim .
x→−1 x + 1
Solution.
2x2 − 2 “ 0”
lim
x→−1 x + 1 0
d 2
(2x − 2)
= lim dx
l’H
x→−1 d
(x + 1)
dx
4x
= lim = lim 4x = −4
x→−1 1 x→−1

102
MATH1036 1st Semester Calculus Lecture Manual 2017 103

e2x − 1
2. lim .
x→0 e x − 1
Solution.
e2x − 1 “ 0”
lim
x→0 e x − 1 0
l’H 2e2x
= lim x
x→0 e
2e0
= 0
e
= 2.

e x + e−x − 2
3. lim .
x→0 1 − cos 2x
Solution.
e x + e−x − 2 “ 0”
lim
x→0 1 − cos 2x 0
l’H
x
e −e −x “ 0”
= lim
x→0 2 sin 2x 0
l’H e x + e−x
= lim
x→0 4 cos 2x
e0 + e0
=
4 cos 0
1+1
=
4(1)
1
= .
2

2x − 1
4. lim .
x→0 x
Solution.
2x − 1 “ 0”
lim
x→0 x 0
l’H 2 x ln 2
= lim
x→0 1
= (ln 2) lim 2 x
x→0
= (ln 2)2 0

= (ln 2)(1)
= ln 2.
2x2 − 2x + 3
Worked Example 6.1.2 (Type ∞
∞ ). 1. lim .
x→∞ 3x2 + 1
Solution.
2x2 − 2x + 3 “ ∞”
lim
x→∞ 3x2 + 1 ∞
l’H 4x − 2 “ ∞”
= lim
x→∞ 6x ∞
l’H 4 2
= lim = .
x→∞ 6 3
104 1st Semester Calculus Lecture Manual 2017 MATH1036

ln x
2. lim √ .
x→∞ x
Solution.
ln x “ ∞”
lim √
x→∞ x ∞
Å ã
1
l’H x
= lim Å ã
x→∞ 1

2 x

2 x
= lim
x→∞ x
2
= lim √
x→∞ x
= 0.
Worked Example 6.1.3 (Type 0 · ∞). Evaluate lim x cosec x.
x→0
Solution.
lim x cosec x “0 · ∞”
x→0
Å ã
1
= lim x
x→0 sin x
x “ 0”
= lim
x→0 sin x 0
l’H 1
= lim
x→0 cos x
1
=
1
= 1.
x
Note that lim = 1. We have proved this earlier, but we have used l’Hôpital’s Rule to evaluate this limit in
x→0sin x
this case. However, for the application of l’Hôpital’s Rule we need the derivative of sin at 0, so that the earlier
sin x
proof of lim = 1 is not redundant.
x→0 x

1 x
Å ã
Worked Example 6.1.4 (Type 1∞ ). 1. Prove that lim 1 + = e, using l’Hôpital’s Rule.
x→∞ x
1 x
Å ã
Proof. Let y = lim 1 + . Then y is an indeterminate of the form 1∞ . So we can rewrite this form into the
x→∞ x
form 00 and use l’Hôpita1’s rule as follows.
1 x
Å ã
y = lim 1 + “1∞ ”
x→∞ x
1 x
Å ã
⇒ ln y = ln lim 1 +
x→∞ x
1 x
Å ã
= lim ln 1 + since the function ln is continuous
x→∞ x
Å ã
1
= lim x ln 1 +
x→∞ x
Å ã
1
ln 1 + “ 0”
x
= lim
x→∞ 1 0
x
MATH1036 1st Semester Calculus Lecture Manual 2017 105

Ö è
Å ã
1 1
− 2
1 x
1+
Åx ã
l’H
= lim
x→∞ 1
− 2
x
1
= lim
x→∞ 1
1+
x
= 1.

Thus we have shown that ln y = 1. By the property of inverse functions, y = eln y = e1 = e. Since y is the original
limit we have proved that
1 x
Å ã
lim 1 + = e. 
x→∞ x

Note. When using l’Hôpital’s rule, as illustrated above, indicate the indeterminate form (in the above case 00 )
l’H
adjacent to the limit and then followed by the = notation indicating that l’Hôpital’s rule is used.
1
2. lim(1 + 3x) 2x .
x→0
1
Solution. Let y = lim(1 + 3x) 2x . Then y is a limit of the indeterminate form 1∞ .
x→0

1
y = lim(1 + 3x) 2x
x→0
1
⇒ ln y = ln lim(1 + 3x) 2x
x→0
1
= lim ln(1 + 3x) 2x
x→0
1
= lim ln(1 + 3x)
x→0 2x
ln(1 + 3x) “ ∞”
= lim
x→0
Å 2x ã ∞
3
l’H 1 + 3x
= lim
x→0 2 Å ã
3 1 3 1 3
= lim = = .
2 x→0 1 + 3x 2 1 + 0 2

3 3
Thus ln y = . Hence, y = e 2 , i. e.,
2
1 3
lim(1 + 3x) 2x = e 2 .
x→0
106 1st Semester Calculus Lecture Manual 2017 MATH1036

Worked Example 6.1.5 (Type 00 ). lim+ (sin x) x .


x→0

Solution. Let y = lim+ (sin x) x . Then y is a limit of the indeterminate form 00 . Then
x→0

y = lim+ (sin x) x
x→0
⇒ ln y = ln lim+ (sin x) x
x→0
= lim+ ln(sin x) x
x→0
= lim+ x ln sin x
x→0

ln sin x “ −∞ ”
= lim+
x→0 1 ∞
x
 cos x 

= lim+ Åsin xã
l’H
x→0 1
− 2
x
 x   
= − lim+ lim+ x lim+ cos x
x→0 sin x x→0 x→0
= −(1)(0)(1)
= 0.

Hence ln y = 0 implies that y = e0 = 1 so that lim+ (sin x) x =1.


x→0

1
Worked Example 6.1.6 (Type ∞0 ). Evaluate lim x x .
x→∞

1
Solution. Let y = lim x x . Then
x→∞

1
ln y = ln lim x x
x→∞
1
= lim ln x x
x→∞
ln x “ ∞”
= lim
x→∞ x ∞
1
l’H
= lim x
1
x→∞
1
= lim
x→∞ x
= 0.

1
Thus ln y = 0 so that lim x x = 1.
x→∞

ã Å
1 1
Worked Example 6.1.7 (Type ∞ − ∞). Evaluate lim x − .
x→0 e − 1 x
MATH1036 1st Semester Calculus Lecture Manual 2017 107

Solution.
Å ã
1 1
lim x − “∞ − ∞”
x→0 e − 1 x
x − (e x − 1) “ 0”
= lim
x→0 xe x − x 0
l’H 1−e x “ 0”
= lim
x→0e x + xe x − 1 0
l’H −e x
= lim x
x→0 e + e x + xe x
−1
= lim
x→0 2 + x
1
=− .
2
Note. ∞ · ∞ and ∞ + ∞ are not indeterminate forms. Do not misuse l’Hôpital’s Rule as the following example
illustrates.
cos 2x
Worked Example 6.1.8 (Misuse of l’Hôpital’s Rule). Evaluate lim .
x→0 x2
1
Solution. This is a limit of the form which is not an indeterminate form. The use of l’Hôpital’s Rule produces
0
the incorrect solution
cos 2x −2 sin 2x −4 cos 2x
lim 2
= lim = lim = −2 WRONG!
x→0 x x→0 2x x→0 2
cos 2x
However, the correct analysis yields lim = ∞.
x→0 x2
Note. Do not get influenced by quotients and immediately use l’Hôpital’s Rule. You must first identify that we
have one of the indeterminate forms 1∞ , 00 , 0 · ∞, 00 , ∞
∞ or ∞ − ∞ in order to use l’Hôpital’s Rule.

Tutorial 6.1.1. 1. Thomas Exercises 11th ed. 4.6 pp. 323-324: 3, 6, 13, 16, 19, 20, 21, 27, 28, 29, 32, 33, 34, 37,
45, 47, 50, 51, 52, 57, 59, 61, 62, 63 or 12th ed. 4.5 p. 261–262: 3, 6, 13, 16, 19, 20, 21, 27, 28, 29, 32, 33, 34,
37, 45, 51, 54, 55, 56, 67, 69, 75, 76, 77.

6.2 Related Rates

LEARNING OUTCOMES:
On completion of this section you should (tick the checkbox when you have mastered the skill)
 1. understand the idea of a related rate,
 2. be able to identify, model and solve a related rate problem using differential calculus,
 3. know how to solve the examinable tutorial problems and the worked out problems from these notes in
this section.
Let f (t) be a differentiable function where t represents time. The

f (t + h) − f (t)
average rate of change of f in the interval [t, t + h] is
h

and the
df f (t + h) − f (t)
rate of change of f with respect to t is f 0 (t) = = lim .
dt h→0 h
108 1st Semester Calculus Lecture Manual 2017 MATH1036

f 0 (t) is often called the instantaneous rate of change of f with respect to t. The standard physical example of the
velocity of a point on a coordinate line is familiar. If s(t) represents the differentiable position function of a point
P on a coordinate line l at time t, then

1. the velocity v(t) of P is v(t) = s0 (t),

2. the speed of P at time t is |v(t)| and

3. the acceleration of P is a(t) = v0 (t) = s00 (t).

f (x + h) − f (x)
Generally, given any differentiable function f (x), we may interpret the ratio as the average rate of
h0
change of f with respect to x (here x is not necessarily representing time). Then f (x) is the rate of change of f
with respect to x.

To solve a related rate problem we use the following strategy.

Related Rate Strategy:

1. Read the problem very carefully.


2. Draw a diagram, where relevant, and introduce notation on the diagram assigning symbols to all
quantities that are functions of time.

3. Express the given information and indicate the quantities that vary on the diagram.
4. Specify, in derivative form, the unknown rate of change that you are looking for.
5. Write an equation that relates the various quantitites of the problem.

6. Use the geometry of the situation, if necessary, to eliminate one of the variables by substitution.
7. Use implicit differentiation, with the Chain Rule, to differentiate the simplified equation w.r.t t.
8. Substitute the given information into the resulting equation and solve for the unknown rate of
change.

9. State the final answer, in coherent form, specifying the units that you are using.

Worked Example 6.2.1 (Sliding ladder, Pythagoras). A ladder 2m long is leaning against the side of a building.
If the bottom of the ladder slides away from the building horizontally at a rate of 0.2m/s, how fast is the ladder
sliding down the building when the top of the ladder is 1.2m above the ground?

Solution. Let x denote the distance from the base of the building to the bottom of the ladder, and y be the distance
dx dy
from the base of the building to the top of the ladder. Then = 0.2m/s. Our objective is to find .
dt dt y=1.2m
MATH1036 1st Semester Calculus Lecture Manual 2017 109

top slides
down

2m
dy ladder
=? y
dt y=1.2m

x
−→
dx
= 0.2m/s bottom slides
dt right

Applying the Theorem of Pythagoras in the resulting right triangle gives the equation
x2 + y2 = 4.
Hence, differentiating implicitly w.r.t. t and incorporating the Chain Rule gives
dx dy
2x + 2y =0
dt dt
dy x dx
⇒ =−
dt y dt
x dx
=− ∵ = 0.2.
5y dt
dy
We require . At y = 1.2 we have
dt y=1.2m

36
x2 + y2 = 4 ⇒ x2 + =4
25
64
⇒ x2 =
25
8
⇒ |x| =
5
8 8
∴x= or x = − .
5 5
8
Since x > 0 at y = 1.2, x = . Hence
5
dy 1 x
=−
dt 5 y
Å ã
8
dy 1 5
⇒ =− Å ã
dt y=1.2 5 6
5
4
=− .
15
Note that the negative sign indicates that the distance from the top of the ladder to the base of the building is
4
decreasing at a rate of m/s.
15
Worked Example 6.2.2 (Cone, Similar Triangles). A conical tank 8m across the top and 6m deep is full of water.
The tank springs a leak at the bottom and loses water at a rate of 2 cubic meters per minute. How fast is the water
level dropping at the instance when the water is exactly 3 meters deep?
110 1st Semester Calculus Lecture Manual 2017 MATH1036

Solution. Let r be the radius of the remaining cone of water and h its corresponding height.

r
6

Then r and h are related by similar triangles,


r 4 2
= = .
h 6 3

dV
Since the cone is losing water, the volume V of the cone decreases at the given rate so that = −2. The objective
dt
dh 1 2 2h
is to find . Since the volume of a cone is V = πr h with r = we have
dt h=3 3 3

1 2
V= πr h
3
Å ã2
1 2h
= π h
3 3
4 3
∴V= πh
27
dV 4 dh
⇒ = π(3)h2
dt 27 dt
4 2 dh
= πh
9 dt
4 2 dh
⇒ −2 = πh
9 dt
4 2 dh
⇒ −2 = π(3 )
9 dt h=3
dh 1
⇒ =− .
dt h=3 2π

1
Hence, at the instant when the water is exactly 3 meters deep, the water level is dropping at a rate of meters

per minute.

Worked Example 6.2.3 (Inclination, Trigonometry). A balloon leaves the ground 50m away from an observer
and rises vertically at a rate of 14m/min. At what rate is the angle of inclination of the observer’s line of sight
increasing at the instant when the balloon is exactly 50m above the ground?

Solution. Let x be the altitude of the balloon and θ be the angle of inclination of the observer’s line of sight. We
dx dθ
then have = 14m/min and we need to find .
dt dt x=50m
MATH1036 1st Semester Calculus Lecture Manual 2017 111

x
dx
= 14
dt

=?
dt x=50
θ
observer 50 m

In the right triangle we have


x
tan θ = .
50
Then
x
tan θ =
50
2 dθ 1 dx
⇒ sec θ =
dt 50 dt
1
= (14)
50
dθ 7
∴ = cos2 θ.
dt 25
π
At x = 50, the triangle is isosceles so that θ = when x = 50. Hence
4

π 1 2
Å ã
dθ 7 7 7 1 7
= cos2 = √ = = = 0.14.
dt x=50m 25 4 25 2 25 2 50

Thus at the the instant when the balloon is 50m above the ground, the inclination of the observer’s line of sight is
increasing at a rate of 0.14 radians per minute.

Tutorial 6.2.1. 1. Thomas Exercises 11th ed. 3.3 pp. 177-178: 1, 4, 6 or 12th ed. 3.4 p. 152: 1, 4, 6.
2. Thomas Exercises 11th ed. 3.9 pp. 236–240: 3, 8, 10, 17, 19, 20, 21, 23, 30 or 12th ed. 3.10 pp. 198–200: 11,
13, 18, 20, 27, 29, 30, 31, 33, 39.

6.3 Extrema and the Extreme Value Theorem

LEARNING OUTCOMES:
On completion of this section you should (tick the checkbox when you have mastered the skill)
 1. know and be able to explain the definition of an absolute (or global) maximum or minimum of a function,
 2. know be able to and explain the definition of a local (or relative) maximum or minimum of a funcion,
 3. know how to determine extrema graphically,
 4. know under which condition a given function will attain absolute extreme values on a closed interval (the
Closed Interval Method),
112 1st Semester Calculus Lecture Manual 2017 MATH1036

 5. know the statements and applications of the Extreme Value Theorem (EVT) and Fermats Theorem,
 6. know how to solve the examinable tutorial problems and worked out problems from these notes on this
section.

Let f be a function and c ∈ dom( f ).

f has an absolute (or global) maximum at c if f (c) ≥ f (x) for all x ∈ dom( f ).

f (c) is then the maximum of f in dom( f ). (c, f (c)) is the highest point of the graph of f .

f has an absolute (or global) minimum at c if f (c) ≤ f (x) for all x ∈ dom( f ).

f (c) is then the minimum of f in dom( f ). (c, f (c)) is the lowest point of the graph of f .
The minimum and maximum of a function f on an interval are called the extreme values of f or extrema of f .

Worked Example 6.3.1. Find the absolute maximum and absolute minimum of the following functions graphi-
cally.
1. f (x) = x2 + 1, 1 ≤ x ≤ 2.
Solution. Since dom( f ) = [1, 2], from the graph of f , f (1) = 2 is the absolute minimum whilst f (2) = 5 is the
absolute maximum of f in [1, 2].
Note. Extrema occur at the endpoints of the interval. f has an absolute minimum and an absolute maximum.
2. g(x) = x2 + 1, −1 < x < 2.
Solution. g does not have an absolute maximum in (−1, 2). However, g has an absolute minimum at x = 0 so that
f (0) = 1 is the absolute minimum.
Note. Extrema occur within the interval (not at endpoints). g has an absolute minimum but does not have an
absolute maximum.
x2 + 1 if x , 0,
(
3. h(x) = − 1 ≤ x ≤ 2.
2 if x = 0,
Solution. We note that h is not continuous in dom(h) = [−1, 2]. h(2) = 5 is the absolute maximum. h does not
have an absolute minimum.
x2 + 1 if x , 0,
(
4. t(x) = − 1 < x < 2.
2 if x = 0,
Solution. t is not continuous in dom(t) = [−1, 2]. t does not have an absolute maximum nor an absolute minimum.

Let c ∈ dom( f ). Then f has a local maximum at c if there exists an open interval (a, b) such that c ∈ (a, b) and
f (c) ≥ f (x) for all x ∈ (a, b) ∩ dom( f ). f (c) is also called a relative maximum.
Also, f has a local minimum at c if there exists an open interval (a, b) such that c ∈ (a, b) and f (c) ≤ f (x) for all
x ∈ (a, b) ∩ dom( f ). f (c) is also called a relative minimum.
The local maxima and minima are called local extrema.

Worked Example 6.3.2. Find the local extrema of the following functions graphically.
1. f (x) = x2 + 1, −1 ≤ x ≤ 2.
Solution. Since dom( f ) = [−1, 2], from the graph of f , f (0) is a local minimum whilst f (−1) = 2 and f (2) = 5
are local maxima of f in [−1, 2].
2. g(x) = x2 + 1, −1 < x < 2.
Solution. g does not have any local maxima in (−1, 2). However, g has a local minimum at x = 0 so that f (0) = 1
is a local minimum. In fact, f (0) = 1 is the absolute minimum, see previous example.
MATH1036 1st Semester Calculus Lecture Manual 2017 113

x2 + 1
(
if x , 0,
3. h(x) = − 1 ≤ x ≤ 2.
2 if x = 0,
Solution. dom(h) = [−1, 2]. h(−1) = 2, h(0) = 2 and h(2) = 5 are local maxima. h does not have any local
minima.
x2 + 1 if x , 0,
(
4. t(x) = − 1 < x < 2.
2 if x = 0,
Solution. dom(t) = (−1, 2). t(0) = 2 is a local maximum. There are no local minima.
5. f (x) = |x|.
Solution. f has a local minimum at x = 0. f (0) = 0 is also the absolute minimum.
6. g(x) = sin x.
π 3π
Solution. g has local maxima at x = + 2πn for n ∈ Z and local minima at x = + 2πn for n ∈ Z. Since
π 2 Å ã 2
 3π
g + 2πn = 1, all local maxima equal 1 whilst g + 2πn = −1 shows that all local minima equal −1.
2 2
Theorem 6.2 (Extreme Value Theorem – EVT). If f is continuous on a closed interval [a, b], then f attains an
absolute maximum value f (c) at some c ∈ [a, b] and an absolute minimum value f (d) at some d ∈ [a, b].

Note. In the EVT, we cannot omit continuous or closed as Worked Example 6.3.1.3 (not continuous) and 6.3.1.2
(open interval, no absolute maximum) illustrates. Continuity on a closed interval guarantees the existence of
BOTH absolute maximum AND absolute minimum.

Theorem 6.3 (Fermat’s Theorem). If has a local maximum or minimum at an interior point c of dom( f ) and if f
is differentiable at c, then f 0 (c) = 0.

Proof. Assume f has a local maximum at c. Let a, b be such that the interval (a, b) belongs to the domain of f ,
a < c < b and f (x) ≤ f (c) for x ∈ (a, b), i. e.,

f (x) − f (c) ≤ 0.

Hence
f (x) − f (c)
≥0 if x ∈ (a, c)
x−c
and
f (x) − f (c)
≤0 if x ∈ (c, b).
x−c
Since f is differentiable at c, it follows that

f−0 (c) ≥ 0 and f+0 (c) ≤ 0


⇒ f 0 (c) = f−0 (c) = f+0 (c) = 0.

A similar proof holds if f has a minimum at c. 

Note that the converse of Fermat’s Theorem is false, i. e., f 0 (c) = 0 ; f (c) is a local maximum or a local
minimum.

Worked Example 6.3.3 (Converse of Fermat’s Theorem is false). Prove that f (x) = x3 has no local extremum.

Proof. f 0 (x) = 3x2 and f 0 (0) = 0. However, f (0) is neither a local maximum nor a local minimum. For x < 0,
f (x) < 0, i. e., f (x) is negative, whereas if x > 0, f (x) > 0. 

A critical number of a function f is a number c ∈ dom( f ) such that either f 0 (c) = 0 or f 0 (c) does not exist.
114 1st Semester Calculus Lecture Manual 2017 MATH1036

f 0 (c) does not exist

f 0 (c) = 0 Horizontal
tangent

Fermats Theorem may be restated as

lf f has a local maximum of minimum at c in the interior of dom( f ), then c is a critical number of f.

Thus the local extrema occur only at the critical numbers or at the endpoints of the domain. However, the critical
numbers of a function need not produce relative extrema.

√3
Worked Example 6.3.4. Find the critical number(s) of f (x) = (x + 5)2 x − 4.

Solution. We find f 0 (x).


1
f (x) = (x + 5)2 (x − 4) 3
1 1 2
⇒ f 0 (x) = 2(x + 5)(x − 4) 3 + (x + 5)2 (x − 4)− 3
ï 3ò
1 2
= 2(x + 5)(x − 4) + (x + 5)2 (x − 4)− 3
3
(x + 5)[6(x − 4) + (x + 5)]
= 2
3(x − 4) 3
(x + 5)(7x − 19)
∴ f 0 (x) = 2
3(x − 4) 3

Consequently, the critical numbers occur when

19
f 0 (x) = 0 ⇔ x = −5 or x =
7

and
f 0 (x) does not exist ⇔ x = 4.

19
Hence f has three critical numbers −5, and 4.
7
To find an absolute maximum or minimum of a continuous function on a closed interval, we note that either it
is local (in which case it occurs at a critical number by Fermat’s Theorem) or it occurs at an endpoint of the
closed interval. We thus have the following method (The Closed Interval Method) to find absolute extrema of
continuous functions on closed intervals which we will exploit in Section 6.6.
MATH1036 1st Semester Calculus Lecture Manual 2017 115

The Closed Interval Method:

To find the absolute maximum and minimum values of a continuous function f on a closed interval [a, b],
use the following steps:

1. Find the critical numbers c ∈ (a, b) of f by finding x where f 0 (x) = 0 or where f 0 (x) does not exist.
2. Evaluate f at each critical number c ∈ (a, b), i. e., evaluate f (c) for critical numbers c ∈ (a, b).
3. Evaluate f at each endpoint of [a, b], i. e., evaluate f (a) and f (b).

4. Compare the values of all f (c), f (a) and f (b).

The smallest of f (c), f (a) or f (b) is the minimum. The greatest of these is the maximum.

Worked Example 6.3.5. Find the absolute maximum and minimum values of f (x) = 2 sin x − cos 2x on [0, 2π].

Solution. Since f is differentiable everywhere in [0, 2π], the critical numbers only occur when f 0 (x) = 0. Thus

f 0 (x) = 0 ⇔ 2 cos x − (−2 sin 2x) = 0


⇔ 2 cos x + 2 sin 2x = 0
⇔ cos x(1 + 2 sin x) = 0
1
⇔ cos x = 0 or sin x = − .
2

Since x ∈ (0, 2π), the critical points are

π 3π 7π 11π
x= , or x= , .
2 2 6 6

Hence f has four critical numbers. Comparing the function values at these critical numbers and at the endpoints
116 1st Semester Calculus Lecture Manual 2017 MATH1036

we get the following.

f (0) = 2 sin 0 − cos 0 = −1.


π π  π
f = 2 sin − cos 2 = 2(1) − (−1) = 3.
Å 2ã Å2 ã Å2 ã
3π 3π 3π
f = 2 sin − cos 2 = 2(−1) − (−1) = −1.
2 2 2
Å ã Å ã Å ã
7π 7π 7π
f = 2 sin − cos 2
6 6 6
π
Å ã
1  1
=2 − − cos 2π + ∵ sin x = − for this critical number x
2 3 2
π
= −1 − cos
3
1
= −1 −
2
3
=− .
Å ã 2 Å ã Å ã
11π 11π 11π
f = 2 sin − cos 2
6 6 6
π
Å ã
1  1
=2 − − cos 4π − ∵ sin x = − for this critical number x
2 3 2
π
= −1 − cos
3
3
=− .
2
f (2π) = 2 sin(2π) − cos(2(2π)) = −1.

7π 11π
Hence we have the absolute minimum at x = and at (occuring at critical numbers). The absolute maximum
6 6
π
occurs at x = (a critical number). Note that the endpoints x = 0 and x = π are local minima.
2
2
Worked Example 6.3.6. Find the absolute maximum and minimum of g(x) = 2x − 3x 3 on [−1, 3].
Solution. We have Ç 1
å
− 13 2 x3 − 1
g (x) = 2 − 2x
0
=2− 1 =2 1 .
x3 x3
The critical values are
1
g0 (x) = 0 ⇔ x 3 − 1 = 0 ⇔ x = 1
1
and g0 (x) does not exist ⇔ x 3 = 0 ⇔ x = 0.

Then comparing
2
g(−1) = 2(−1) − 3(−1) 3 = −2 − 3 = −5,
g(0) = 0,
g(1) = 2 − 3 = −1,
√3
g(3) = 6 − 3 9 ≈ −0.24

we see that we have an absolute maximum at x = 0 (critical point) and an absolute minimum at x = −1 (interval
endpoint).

Tutorial 6.3.1. 1. Thomas Exercises 11th ed. 4.1 pp. 272–273: 1, 5, 6, 7, 10, 14, 17, 20, 29, 33, 41, 45, 46, 48,
49, 52, 53 or 12th ed. 4.1 pp. 227–229: 1, 5, 6, 7, 10, 14, 18, 23, 26, 35, 39, 49, 52, 55, 59, 60, 62, 63, 66, 67.
MATH1036 1st Semester Calculus Lecture Manual 2017 117

6.4 Rolle’s Theorem and the Mean Value Theorem

LEARNING OUTCOMES:
On completion of this section you should (tick the checkbox when you have mastered the skill)
 1. know Rolle’s Theorem and its proof,
 2. be able to use the IVT and apply Rolle’s Theorem,
 3. know the Mean Value Theorem (MVT) and its proof,
 4. be able to apply the MVT,
 5. know how to solve the examinable tutorial problems and the worked out problems from these notes on
this section.

Theorem 6.4 (Rolle’s Theorem). Let f be a function that satisfies the following three hypotheses:
1. f is continuous on the closed interval [a, b],
2. f is differentiable on the open interval (a, b),
3. f (a) = f (b).
Then there is a number c ∈ (a, b) such that f 0 (c) = 0.

Proof. There are three possible cases for f .


1. f is a constant function. Suppose that f (x) = k where k is some constant. Then clearly f 0 (x) = 0 for each x.
Thus for each c ∈ (a, b), f 0 (c) = 0.
2. f (x) > f (a) for some x ∈ (a, b).

f 0 (c) = 0 f 0 (c) = 0

a c b a c b

By the first hypothesis, f is continuous on [a, b] and thus we may apply the Extreme Value Theorem. By the
EVT f has a maximum value somewhere in [a, b]. Since f (a) = f (b), f must attain its maximum value at some
c ∈ (a, b). Then f has a local maximum at c. Since f is differentiable in (a, b) by hypothesis 2, and c ∈ (a, b),
f 0 (c) exists. Consequently, by Fermat’s Theorem, f 0 (c) = 0.
3. f (x) < f (a) for some x ∈ (a, b).

f 0 (c) = 0 f 0 (c) = 0

a c b a c b

This case is similar to case 2, by the EVT f has a minimum value somewhere in [a, b]. Since f (a) = f (b), f
attains its minimum value at some c ∈ (a, b). Again, f 0 (c) = 0 by Fermat’s Theorem. 
118 1st Semester Calculus Lecture Manual 2017 MATH1036

We can see that in particular Rolle’s Theorem implies that if a differentiable curve crosses the x-axis twice ( f (a) =
f (b) = 0), there must be a point between successive crossings where the tangent to the curve is parallel to the
x-axis.
Rolle’s Theorem requires that f is continuous on [a, b] and differentiable on (a, b) with f (a) = f (b), i. e., all three
hypotheses MUST be satisfied to guarantee at least one c ∈ (a, b) for which f has a horizontal tangent (at c). Of
course, there could be more than one point c in (a, b) for which the tangent lines are horizontal at these points c.

Worked Example 6.4.1. Prove that the equation 2x − 1 − sin x = 0 has exactly one real root.
Solution. We will use the IVT to conclude that the equation has at least one real root and then apply Rolle’s
theorem to show that the root is unique.
Consider the closed interval [0, π2 ] with f (x) = 2x − 1 − sin x. Since the functions 2x, −1 and − sin x are continuous
and differentiable on R, the sum of these functions which is f (x) is also continuous and differentiable on R. In
particular f is continous on [0, π2 ]. Since f (0) = 2(0) − 1 − sin 0 = −1 < 0 and f ( π2 ) = 2 π2 − 1 − sin π2 = π − 2 > 0,
with f continuous on [0, π2 ], by the IVT there is c ∈ (0, π2 ) such that f (c) = 0. Hence c is a root of f and thus the
given equation has at least one real root.
Now suppose that the equation has distinct roots c and d in R. We may suppose that c < d. Then f (c) = 0 = f (d)
with f continuous on [c, d] and differentiable on (c, d). Thus by Rolle’s Theorem there is t ∈ (c, d) such that
f 0 (t) = 0. But f 0 (x) = 2 − cos x > 0 for each x since | cos x| ≤ 1. In particular, f 0 (t) > 0 which contradicts
f 0 (t) = 0. This contradiction shows that the given equation cannot have distinct real roots. Hence c is the unique
real root of the equation.

Worked Example 6.4.2. Let f (x) = |x − 2|. Then f satisfies f (1) = 1 = f (3), but there is no c ∈ (1, 3) such that
f 0 (c) = 0. Why does this not contradict Rolle’s Theorem?
Solution. f is not differentiable on (1, 3). Hence, one of the hypotheses of Rolle’s Theorem is violated, so Rolle’s
Theorem cannot apply in this case.

Theorem 6.5 (Mean Value Theorem). Let f be a function that satisfies the following conditions:
1. f is continuous on the closed interval [a, b],
2. f is differentiable on the open interval (a, b).
Then there is a number c ∈ (a, b) such that
f (b) − f (a)
f 0 (c) = ,
b−a
or equivalently,
f (b) − f (a) = f 0 (c)(b − a).

Proof. Consider the equation of the secant line AB.

f (x)

h(x)
A y = f (x)

B
yAB

a x b

The slope of the secant line AB is


f b) − f (a)
mAB = .
b−a
MATH1036 1st Semester Calculus Lecture Manual 2017 119

Thus the equation of the secant line AB is


yAB − f (a) = mAB (x − a)
Å ã
f (b) − f (a)
= (x − a)
b−a
Å ã
f (b) − f (a)
∴ yAB = f (a) + (x − a).
b−a
We now apply Rolle’s Theorem to the function
Å ã
f (b) − f (a)
h(x) = f (x) − yAB = f (x) − f (a) − (x − a)
b−a
defined as the difference between f and the function whose graph is the secant line AB.
We are going to show that h satisfies the hypotheses of Rolle’s Theorem.
1. Since yAB is a first degree polynomial, yAB is continuous on [a, b]. Thus h is a difference of continuous functions,
and hence h is also continuous on [a, b].
2. Since yAB is a first degree polynomial, yAB is differentiable on (a, b). Thus h is a difference of differentiable
functions and so h is also differentiable
Å on (a,ã b).
f (b) − f (a)
3. Since h(a) = f (a) − f (a) − (a − a) = 0 and
b−a
Å ã
f (b) − f (a)
h(b) = f (b) − f (a) − (b − a)
b−a
= f (b) − f (a) − ( f (b) − f (a))
= 0,
it follows that h(a) = h(b).
Thus h indeed satisfies the hypothesis of Rolle’s Theorem. Hence we may conclude that there is c ∈ (a, b) such
that h0 (c) = 0. Thus
0 = h0 (c)
f (b) − f (a)
= f 0 (c) −
b−a
f (b) − f (a)
⇒ f 0 (c) = . 
b−a
Worked Example 6.4.3. Prove that the function f (x) = x3 − 8x − 5 satisfies the hypothesis of the MVT on the
interval [1, 4] and find a number c ∈ (1, 4) that satisfies the conclusion of the MVT.
Solution. Since f is a polynomial it is continuous and differentiable on R. In particular, f is continuous on [1, 4]
and differentiable on (1, 4). Thus f satisfies the hypothesis of the MVT. Accordingly, there is c ∈ (1, 4) such that
f (4) − f (1)
f 0 (c) =
4−1
27 − (−12)
=
3
39
=
3
∴ f 0 (c) = 13.
Since f 0 (x) = 3x2 − 8,
f 0 (c) = 13
⇒ 3c2 − 8 = 13
⇒ c2 = 7
√ √
⇒ c = 7 or c = − 7.

Since c ∈ (1, 4), we must have c = 7.
120 1st Semester Calculus Lecture Manual 2017 MATH1036

x2 − 4x + 3
Worked Example 6.4.4. Given the function f (x) = in the interval [1, 3] discuss the validity of the
x−2
MVT.
Solution. We see that 2 ∈ [1, 3] but 2 < dom( f ). So f is not defined at 2, and hence not continuous at 2. Thus
f is not continuous on [1,4]. Therefore the hypotheses of the MVT are not satisfied, and the MVT is thus not
f (3) − f (1) 0 − 0
applicable. There may be a value c ∈ (1, 3) for which f 0 (c) = = = 0.
3−1 2
Theorem 6.6. If f 0 (x) = 0 for all x in an interval (a, b), then f is a constant function on (a, b).

Proof. Let x1 , x2 ∈ (a, b) be distinct. We may suppose that x1 < x2 . Since f 0 (x) = 0 for all x ∈ (a, b), f is
differentiable on (x1 , x2 ) and continuous on [x1 , x2 ]. Applying the MVT to f on [x1 , x2 ] we find c ∈ (x1 , x2 ) such
that
f (x2 ) − f (x1 )
f 0 (c) =
x2 − x1
f (x2 ) − f (x1 )
⇒0= ∵ f 0 (x) = 0 for all x ∈ (a, b)
x2 − x1
⇒ 0 = f (x2 ) − f (x1 )
⇒ f (x2 ) = f (x1 ).

Thus f has the same value for any two numbers in (a, b). Consequently, f is constant on (a, b). 

Theorem 6.7. If f 0 (x) = g0 (x) for all x in an interval (a, b), then f −g is a constant on (a, b), that is, f (x) = g(x)+C,
where C is a constant.

Proof. Let F(x) = f (x) − g(x). Then F 0 (x) = f 0 (x) − g0 (x) = 0 for all x ∈ (a, b). By the above theorem, F is
constant on (a, b), i. e., f − g is constant on (a, b). Thus there is C ∈ R such that ( f − g)(x) = C for all x ∈ (a, b),
i. e., f (x) − g(x) = C or f (x) = g(x) + C for all x ∈ (a, b). 

Tutorial 6.4.1. 1. Thomas Exercises 11th ed. 4.2 pp. 282–284: 1, 3, 4, 5, 7–10, 12, 13, 15, 16, 19, 22, 51, 56, 57,
58 or 12th ed. 4.2 pp. 236–237: 1, 3, 5, 6, 11, 12, 15, 16, 18, 19, 21, 22, 25, 28, 57, 66, 67.
2. Show that the functions y(x) = Ce x , C ∈ R, are the only functions which satisfy y0 = y. Hint: Consider the
function y(x)e−x .

6.5 Curve Sketching

LEARNING OUTCOMES:
On completion of this section you should (tick the checkbox when you have mastered the skill)
 1. be able to use the first derivative to find the interval(s) on which a given function is increasing or decreas-
ing,
 2. be able to use the First and Second Derivative Test to determine whether a function has a local maximum
or minimum at a critical number c or not,
 3. be able to explain the concepts of concave up and concave down,
 4. be able to use the second derivative to find the interval(s) on which a function is concave up and concave
down,
 5. know the definition and understand the notion of a point of inflection,
 6. be able to use the second derivative to find the points of inflection of a function,
 7. be able to draw any given curve using the curve sketching guidelines,
 8. know how to solve the examinable tutorial problems and worked out problems from these notes on this
section.
MATH1036 1st Semester Calculus Lecture Manual 2017 121

6.5.1 Increasing and Decreasing Functions and The First Derivative Test

A function f is increasing on an interval I if: ∀ x1 , x2 ∈ I, x1 < x2 ⇒ f (x1 ) < f (x2 )

and
a function f is decreasing on an interval I if: ∀ x1 , x2 ∈ I, x1 < x2 ⇒ f (x1 ) > f (x2 ).

The following theorem indicates how the derivative may be used to determine intervals on which a function is
increasing and decreasing. This theorem is called the I/D Test.

Theorem 6.8 (Increasing and Decreasing Test). Let f be a function that is continuous on a closed interval [a, b]
and differentiable on the open interval (a, b).
1. If f 0 (x) > 0 for each x ∈ (a, b), then f is increasing on [a, b].
2. If f 0 (x) < 0 for each x ∈ (a, b), then f is decreasing on [a, b].
3. If f 0 (x) = 0 for each x ∈ (a, b), then f is constant on [a, b].

Proof. 1. Suppose that f 0 (x) > 0 for all x ∈ (a, b) and consider any two numbers x1 , x2 ∈ [a, b] such that x1 < x2 .
We wish to show that f (x1 ) < f (x2 ).
Since f is continuous on [a, b] and differentiable on (a, b), f is also continuous on [x1 , x2 ] and differentiable on
(x1 , x2 ). We can therefore apply the Mean Value Theorem to f on [x1 , x2 ]. Consequently, by the MVT there is
c ∈ (x1 , x2 ) such that
f (x2 ) − f (x1 )
f 0 (c) = .
x2 − x1
Hence
f (x2 ) − f (x1 ) = f 0 (c)(x2 − x1 ).
Since x2 − x1 > 0 and since by the hypothesis, f 0 (c) > 0, the product f 0 (c)(x2 − x1 ) > 0, i. e., f (x2 ) − f (x1 ) > 0.
Hence f (x2 ) > f (x1 ).
2. The proof is similar to that of 1. above.
3. Theorem 6.6. 

Guidelines for finding the intervals of INCrease and DECrease:

Let f be continuous on (a, b). To find the open intervals on which f is increasing and decreasing, use the
following steps:

1. Find the critical numbers c ∈ (a, b) of f by solving f 0 (x) = 0 AND finding x such that f 0 (x) does not
exist.
2. Use the critical numbers to create a first derivative table.
3. Determine and place the sign of f 0 at one test value in each open interval in the table.

4. Write down the sign of f 0 in the table.


5. On each interval, use the I/D Test to determine whether f is INCreasing or DECreasing.

Worked Example 6.5.1. Find the open intervals where the function f (x) = 4x5 − 15x4 − 20x3 + 110x2 − 120x + 40
is increasing and decreasing.
Solution. Since f is a polynomial, it is everywhere continuous and everywhere differentiable.
f 0 (x) = 20x4 − 60x3 − 60x2 + 220x − 120
= 20(x4 − 3x3 − 3x2 + 11x − 6)
= 20(x + 2)(x − 1)2 (x − 3).
122 1st Semester Calculus Lecture Manual 2017 MATH1036

The critical numbers of f occur only when


f 0 (x) = 0 ⇔ x = −2, 1 or 3.
This produces the 1st Derivative table:

(−∞, −2) −2 (−2, 1) 1 (1, 3) 3 (3, ∞)

20(x + 2) − 0 + + + + +

(x − 1)2 + + + 0 + + +

x−3 − − − − − 0 +

f 0 (x) + 0 − 0 − 0 +

I/D Test INC NO DEC (DEC) DEC NO INC

Hence, f 0 > 0 on (−∞, −2) and (3, ∞) so that by the I/D Test, f increases on (−∞, −2) and (3, ∞). Also f 0 < 0 on
(−2, 1) and (1, 3). Thus by the I/D Test, f decreases on (−2, 1) and (1, 3).

Note. It would also be true to say that f increases on (−∞, −2] and [3, ∞) whilst f decreases on [−2, 3].
The next theorem is a consequence of the I/D Test, which makes locating the local extrema of a function more
explicit.

Theorem 6.9 (The First Derivative Test). Let c ∈ (a, b) be a critical number of a function f that is continuous on
[a, b] and differentiable on (a, b), except possibly at c.
1. If f 0 (x) > 0 for a < x < c and f 0 (x) < 0 for c < x < b, i. e., f 0 (x) changes from positive to negative at c, then
f (c) is a local maximum of f .
2. If f 0 (x) < 0 for a < x < c and f 0 (x) > 0 for c < x < b, i. e., f 0 (x) changes from negative to positive at c, then
f (c) is a local minimum of f .
3. If f 0 (x) > 0 or f 0 (x) < 0 for each x ∈ (a, b) except x = c, i. e., f 0 (x) does not change sign at c, then f (c) is not a
local extremum of f .

y y

f 0 (x) < 0
f 0 (x) > 0 f 0 (x) < 0 f 0 (x) > 0

x x
c c
Local maximum Local minimum

y y

f 0 (x) > 0
f 0 (x) < 0
f 0 (x) < 0
f (x) > 0
0

x x
c c
No max or min No max or min
MATH1036 1st Semester Calculus Lecture Manual 2017 123

First Derivative Test Guidelines for finding local maxima and local minima

1. Complete the I/D Test producing the first derivative table.


2. Based on the change of the sign of f 0 around the critical values, determine the type of local extremum.
Note The following symbolic tables can help to memorize the First Derivative Test.
(a) f 0 changes sign from positive to negative around c ⇒ f (c) is a local maximum.

f 0 (c) + 0 −

Slope

LOC
f INC DEC
MAX

(b) f 0 changes sign from negative to positive around c ⇒ f (c) is a local minimum.

f 0 (c) − 0 +

Slope

LOC
f DEC INC
MIN

(c) f 0 > 0 around c ⇒ f is increasing around c and c is neither a local maximum nor a local minimum.

f 0 (c) + 0 +

Slope

not LOC MAX


f INC INC
not LOC MIN

(d) f 0 < 0 around c ⇒ f is decreasing around c and c is neither a local maximum nor a local minimum.

f 0 (c) − 0 −

Slope

not LOC MAX


f DEC DEC
not LOC MIN
124 1st Semester Calculus Lecture Manual 2017 MATH1036

x
Worked Example 6.5.2. Find local maxima and minima of f (x) = using the First Derivative Test.
x2 +4
Solution.
(x2 + 4) − x(2x) 4 − x2 (2 − x)(2 + x)
f 0 (x) = = = .
(x2 + 4)2 (x2 + 42 ) (x2 + 4)2
The critical numbers only occur when
f 0 (x) = 0 ⇔ (2 − x)(2 + x) = 0 ⇔ x = ±2.
The First Derivative Table is then

(−∞, −2) −2 (−2, 2) 2 (2, ∞)

2+x − 0 + + +

2−x + + + 0 −

(x2 + 4)2 + + + + +

(2 − x)(2 + x)
f 0 (x) = − 0 + 0 −
(x2 + 4)

Slope & → % → &


DEC NO INC NO DEC

Applying the First Derivative Test we see that


1
f 0 changes sign from negative to positive around x = −2 ⇒ f (−2) = − is a local minimum
4
1
f 0 changes sign from positive to negative around x = 2 ⇒ f (2) = is a local maximum
4
1
Worked Example 6.5.3. Use the First Derivative Test to find the local extrema of g(x) = (x4 − 4x3 ).
9
Solution.
1 4
g0 (x) = (4x3 − 12x2 ) = x2 (x − 3).
9 9
Thus g0 (x) = 0 ⇔ x = 0 or x = 3 which are the only critical numbers. The First Derivative Table gives

(−∞, 0) 0 (0, 3) 3 (3, ∞)

4 2
x + 0 + + +
9
x−3 − − − 0 +

4 2
g0 (x) = x (x − 3) − 0 − 0 +
9
& → & → %
DEC (DEC) DEC NO INC

By the First Derivative Test,


g0 changes sign from negative to positive around x = 3
1
⇒ f (3) = [34 − 4(33 )] = 32 − 4(3) = −3 is a local minimum.
9
g0 does not change sign around x = 0 ⇒ f (0) = 0 is neither a local maximum nor a local minimum.
MATH1036 1st Semester Calculus Lecture Manual 2017 125

6.5.2 Concavity and The Second Derivative Test

You have seen that the first derivative locates the intervals of increase and decrease of a function f by providing
the slope of f . This gives a rough idea of the straight line shape of f . The bending or curving of the graph of f ,
giving its true shape, is determined by the second derivative of f . The nature of the bending or curving of f is
called the concavity of f . More precisely,

If the graph of f lies ABOVE all of its tangents on an interval I, then f is CONCAVE UPWARD on I.

x
Concave up

If the graph of f lies BELOW all of its tangents on an interval I, then f is CONCAVE DOWNWARD on I.

x
Concave down

The following theorem shows us how to use the second derivative of a function to determine the intervals on which
the function is concave up (CU) and concave down (CD).

Theorem 6.10 (Concavity Test). Let f be a function whose second derivative exists on an open interval I.
1. If f 00 (x) > 0 for all x ∈ I, then f is concave upward on I.
2. If f 00 (x) < 0 for all x ∈ I, then f is concave downward on I.

Note. In more advanced mathematics, one considers convex instead of concave up. A function f is called convex
if the straight line segment connecting two points on the graph of f lies on or above the graph of f . Every concave
up functions is convex, but the converse is not true. For example, straight lines are convex but not concave up.
126 1st Semester Calculus Lecture Manual 2017 MATH1036

Guidelines for finding the intervals on which f is CU and CD:

To find the open intervals on which f is concave up and concave down, use the following steps:

1. Identify intervals (a, b) on which f is continuous and twice differentiable.


2. Find the numbers c ∈ (a, b) such that f 00 (c) = 0.
3. Use the above results to determine open intervals and a second derivative table.
4. Determine and place the sign of f 00 at one test value in each open interval in the table.

5. Generate the sign of f 00 in the table.


6. Use the Concavity Test to determine whether f is CU or CD on each interval.

1 3
Worked Example 6.5.4. Determine the open intervals on which the graph of f (x) = x4 − x2 is concave up
4 2
and concave down.
Solution. Since f 0 (x) = x3 − 3x,
f 00 (x) = 3x2 − 3 = 3(x − 1)(x + 1).
Then f 00 exists everywhere. Now f 00 (x) = 0 ⇔ x = ±1. The second derivative table gives

(−∞, −1) −1 (−1, 1) 1 (1, ∞)

3(x − 1) − − − 0 +

x+1 − 0 + + +

f 00 (x) = 3(x − 1)(x + 1) + 0 − 0 +

CONCAVITY CU NO CD NO CU

Hence, by the Concavity Test, f is concave up on (−∞, −1) and (1, ∞) whilst f is concave down on (−1, 1). The
graph of f is depicted below to illustrate the upward bend (CU) on (∞, 1) and (1, ∞) and the downward bend (CD)
on (−1, 1).

−3 −2 −1 1 2 3
x

−2
CU CD CU
MATH1036 1st Semester Calculus Lecture Manual 2017 127

x2 + 1
Worked Example 6.5.5. Determine the open intervals on which the graph of g(x) = 2 is concave up and
x −4
concave down.
Solution.
5 −10x
g(x) = 1 + ⇒ g0 (x) = 2 .
x2 − 4 (x − 4)2
Thus
(−10)(x2 − 4)2 − (−10x)(2)(x2 − 4)(2x) 10(3x2 + 4)
g00 (x) = = .
(x2 − 4)4 (x2 − 4)3
There are no points at which g00 (x) = 0, but g00 (x) does not exist when the denominator (x2 − 4)3 is zero, i. e., when
x = ±2. The second derivative table yields

(−∞, −2) −2 (−2, 2) 2 (2, ∞)

10(3x2 + 4) + + + + +

(x2 − 4)3 + 0 − 0 +

10(3x2 + 4)
g00 (x) = + U − U +
(x2 − 4)3

CONCAVITY CU U CD U CU

Hence, by the Concavity Test, g is concave up on (∞, 2) and (2, ∞) whilst g is concave down on (−2, 2). The graph
of g is depicted below to illustrate the concavity.

y
Concave Up Concave Up

−2 2 x

Concave Down
128 1st Semester Calculus Lecture Manual 2017 MATH1036

We notice that in the above examples there are points in the domain of the functions at which the concavity
changes. If the tangent to the graph exists at such points of concavity change, then that point is called a point of
inflection. More precisely,

A point P = (c, f (c)) on the graph of a function f is a point of inflection if there exists an open interval
(a, b) with c ∈ (a, b) with f continuous at c such that one of the following statements hold:
(i) f 00 (x) > 0 if a < x < c and f 00 (x) < 0 if c < x < b, i. e., f changes from concave up to concave down at c
or
(ii) f 00 (x) < 0 if a < x < c and f 00 (x) > 0 if c < x < b, i. e., f changes from concave down to concave up at c.

Three types of points of inflection are shown below. Note that the graph crosses its tangent line at a point of
inflection.

y y
Concave Concave
Down Up

Concave
Up Point of inflection Point of inflection

Concave
x Down x

Concave
Down

Point of inflection
Concave
Up

Worked Example 6.5.6. Find the points of inflection of f (x) = sin x + cos x on [0, 2π].

Solution. Since f 0 (x) = cos x − sin x, f 00 (x) = − sin x − cos x. Then

f 00 (x) = 0 ⇔ − sin x − cos x = 0


⇔ − sin x = cos x
⇔ tan x = −1
π
⇔ x = − + kπ, k ∈ Z,
4
3π 7π
∴ f (x) = 0 ⇒ x =
00
, ∵ 0 ≤ x ≤ 2π.
4 4

The second derivative table will give the points of inflection.


MATH1036 1st Semester Calculus Lecture Manual 2017 129

Å ã Å ã Å ã
3π 3π 3π 7π 7π 7π
0, , , 2π
4 4 4 4 4 4

f 00 (x) = − sin x − cos x − 0 + 0 −

CONCAVITY CD NO CU NO CD

3π 7π
Hence, f changes concavity at and . Since f is continuous at these points, we have points of inflection at
4 4 Å ã Å ã
3π 7π 3π 7π
and . Hence the points of inflection are , 0 and , 0 . Thus the points of inflection occur at the
4 4 4 4
x-intercepts.

y Points of inflection


x
3π 7π
4 4

In addition to testing for concavity, the second derivative can be used to perform a simple test for local maxima
and minima. The test is based on the fact that if the graph of a function f is concave up on an open interval
containing c, and f 0 (c) = 0, then f (c) must be a local minimum. Similarly, if the graph of a function f is concave
downward on an open interval containing c, and f 0 (c) = 0, then f (c) must be a local maximum.

Theorem 6.11 (The Second Derivative Test). Suppose that a function f is differentiable on an open interval (a, b)
containing c and the second derivative of f on (a, b) exists.
1. If f 0 (c) = 0 and f 00 (c) > 0, then f (c) is a local minimum.
2. If f 0 (c) = 0 and f 00 (c) < 0, then f (c) is a local maximum.

Proof. 1. If f 0 (c) = 0 and f 00 (c) > 0, then

f 0 (x) − f 0 (c) f 0 (x)


0 < f 00 (c) = lim = lim .
x→c x−c x→c x − c

Hence
f 0 (x)
>0
x−c
for x , c close to c. If x < c, then x − c < 0 and f 0 (x) < 0 for x close to c. Also, if x > c, then x − c > 0 and
f 0 (x) > 0 for x close to c. Thus, f 0 changes sign from negative to positive at c. The First Derivative Test then
implies that f (c) is a local minimum.
2. Similar to 1. 

We have derived two tests for local maxima and local minima: The First Derivative Test and The Second
Derivative Test. Either one can be used at your preference but note that if f 00 c) = 0 then The Second
Derivative Test is not applicable. In such cases you have no choice but to resort to The First Derivative
Test.

Worked Example 6.5.7. If f (x) = 5x3 − 3x5 , use The Second Derivative Test (where possible) to find the local
maxima and minima of f . Discuss concavity, find the points of inflection and sketch the graph of f .
130 1st Semester Calculus Lecture Manual 2017 MATH1036

Solution.
f 0 (x) = 15x2 − 15x4 = 15x2 (1 − x2 ).
1. Local maxima and local minima: Now f 0 (x) = 0 gives the three critical numbers −1, 0, 1 of f . Then

f 00 (x) = 30x − 60x3 = 30(x − 2x3 ).

Apply the Second Derivative Test at these critical numbers:

f 00 (−1) = 30(−1 − 2(−1)) = 30 > 0 ⇒ f (−1) = −5 + 3 = −2 is a local minimum.


f 00 (0) = 0 ⇒ Test fails, use First Derivative Test
f 00 (1) = 30(1 − 2) = −30 < 0 ⇒ f (1) = 5 − 3 = 2 is a local maximum.

Hence, the Second Derivative Test gives us a local maximum at (1, 2) and a local minimum at (−1, −2). We now
conclude the nature of the extremum at x = 0 by the First Derivative Test:

(−1, 0) 0 (0, 1)

15x2 + 0 +

1−x + + +

1+x + + +

f 0 (x) = 15x2 (1 − x2 ) + 0 +

Slope % → %

INC (INC) INC

Hence, by the First Derivative Test, f (0) is neither a local maximum nor a local minimum.
2. Concavity and Points of inflections: Since

f 00 (x) = 30(x − 2x3 ) = 30x(1 − 2x2 ),

1
we see that f 00 (x) = 0 ⇔ x = 0, ± √ . The second derivative table then yields
2

Å ã Å ã Å ã
1 1 1 1 1 Ä ä
−∞, − √ −√ − √ ,0 0 0, √ √ √1 , ∞
2
2 2 2 2 2
30x − − − 0 + + +

1 − 2x + + + + + 0 −

1 + 2x − 0 + + + + +

f 00 (x) = 30x(1 − 2x2 ) + 0 − 0 + 0 −

CONCAVITY CU NO CD NO CU NO CD

Å ã Å ã
1 1
Hence, by the Concavity Test, f is concave up on −∞, − √ and 0, √ , whilst f is concave down on
Å ã Å ã 2 2
1 1
− √ , 0 and √ , ∞ .
2 2
MATH1036 1st Semester Calculus Lecture Manual 2017 131

1 1
Since the concavity changes around x = − √ , 0 and √ with f continuous here, we have three points of
2 2
inflection.
Since f (x) = 5x3 − 3x5 = x3 (5 − 3x2 ),
Å ã ï ò
1 1 1 3 7
f −√ =−√ 5− =− √ ,
2 2 2 2 4 2
f (0) = 0,
Å ã ï ò
1 1 1 3 7
f √ = √ 5− = √ .
2 2 2 2 4 2
Å ã Å ã
1 7 1 7
Thus − √ , − √ , (0, 0) and √ , √ are points of inflection.
2 4 2 2 4 2

y
Local max
2

POI

−1 POI
x
1

POI

−2
Local min

The graph above does not depict all features and is only a rough sketch. The intercepts are missing. This leads
to the culmination of the applications of the derivative into the procedure to sketch a curve showing all important
features.

Tutorial 6.5.1. 1. Thomas Exercises 11th ed. 4.3 pp. 289–290: 1, 5, 8, 9, 16, 17, 21, 23, 47, 48 or 12th ed. 4.3
pp. 241–242: 1, 5, 12, 19, 26, 27, 33, 35, 67, 68.
2. Thomas Exercises 11th ed. 4.4 pp. 298–301: 1, 2, 3, 5, 67, 68, 75, 77, 79, 83, 84 or 12th ed. 4.4 pp. 251–254:
1, 2, 3, 5, 103, 104, 111, 113, 115, 117, 118.
132 1st Semester Calculus Lecture Manual 2017 MATH1036

6.5.3 A summary of Curve Sketching

Curve sketching guidelines:

1. Find the DOMAIN of f .

2. Determine the INTERCEPTS:


x-intercept: let y = 0 and solve for x,
y-intercept: let x = 0 and evaluate y.
3. Check for SYMMETRY:
f (x) = f (−x) ⇒ even function ⇒ y-axis symmetry,
f (x) = − f (−x) ⇒ odd function ⇒ symmetry about origin.

4. Find the ASYMPTOTES:


Horizontal: Evaluate lim f (x) and lim f (x). If lim f (x) = L or lim f (x) = L then y = L is HA.
x→∞ x→−∞ x→∞ x→−∞
Vertical: Look at all a < dom( f ) with points x near a in the domain of f . If lim+ f (x) = ±∞ or
x→a
lim− f (x) = ±∞, then x = a is VA.
x→a
p(x)
Slant or Oblique (for rational functions): If f (x) = , where p and q are polynomials with deg p =
q(x)
r(x)
deg q + 1, do long division to get f (x) = (mx + b) + with deg r < deg q. Then y = mx + b is SA.
q(x)
5. Find the INTERVALS OF INCREASE and DECREASE: Use the Inc/Dec Test
Regions of increase: f 0 (x) > 0 on interval I ⇒ f is increasing on I,
Regions of decrease: f 0 (x) < 0 on interval I ⇒ f is decreasing on I.
6. Determine the LOCAL MAXIMA and LOCAL MINIMA:
Find the critical numbers of f : Solve for x where f 0 (x) = 0 AND f 0 (x) does not exist.
Use the 1st Derivative Test: If c is a critical number, then
f 0 changes sign from positive to negative around c ⇒ f (c) is a local maximum,
f 0 changes sign from negative to positive around c ⇒ f (c) is a local minimum.
or use the 2nd Derivative Test: If c is a critical number and f 00 (c) , 0, then
f 00 (c) > 0 ⇒ f (c) is a local minimum,
f 00 (c) < 0 ⇒ f (c) is a local maximum.
7. Find the CONCAVITY and POINTS OF INFLECTION: Use Concavity Test. Find f 00 (x).
f 00 (x) > 0 ⇒ CU,
f 00 (x) < 0 ⇒ CD.
P is a point of inflection if f is continuous at P and concavity changes around P.

8. Use the above information to SKETCH the curve giving all points of interest.

Worked Example 6.5.8 (No Asymptotes). Draw the graph of the function f (x) = x5 − 5x3 .
Solution. Let y = f (x).
1. Domain: Since f is a polynomial, the domain of f is R.
2. Intercepts:
x-intercepts: Let y = 0. Then

y = 0 ⇔ x5 − 5x3 = 0
⇔ x3 (x2 − 5) = 0
√ √
⇔x=0 or x= 5 or x = − 5.

y-intercept: Let x = 0. Then y = 0.


MATH1036 1st Semester Calculus Lecture Manual 2017 133

3. Symmetry: Since f (−x) = (−x)5 − 5(−x)3 = −x5 + 5x3 = −(x5 − 5x3 ) = − f (x), f is an odd function, hence
symmetric about the origin.
4. Asymptotes: None.
5. Regions of increase and decrease: We find the critical points: f 0 (x) = 5x4 − 15x2 . Then
f 0 (x) = 0 ⇔ 5x4 − 15x2 = 0
⇔ 5x2 (x2 − 3) = 0
√ √
⇔x=0 or x= 3 or x = − 3.
√ √
Hence − 3, 0 and 3 are the critical values of f .

√ √ √ √ √ √
(−∞, − 3) − 3 (− 3, 0) 0 (0, 3) 3 ( 3, ∞)

5x2 + + + 0 + + +

x− 3 − − − − − 0 +

x+ 3 − 0 + + + + +

f 0 (x) + 0 − 0 − 0 +

Slope % → & → & → %


INC NO DEC (DEC) DEC NO INC

√ √
√ − 3) and ( √3, ∞) since0 f > 0 here.
0
Intervals of increase: (−∞,
Intervals of decrease: (− 3, 0) and (0, 3) since f < 0 here. √ √
6. Local extrema: Using the 1st Derivative
√ Test and the table above we see that f (−√ 3) = 6 √3 is a local
maximum since the sign of f 0 around√− 3 changes from positive to negative. Also f ( 3) = −6 3 is a local
minimum since the sign of f 0 around 3 changes from negative to positive. There is no local extremum at 0.
7. Concavity and Points of Inflection: We consider the equation f 00 (x) = 0. Then
f 00 (x) = 0 ⇔ 20x3 − 30x = 0
⇔ 10x(2x2 − 3) = 0
… …
3 3
⇔x=0 or x= or x = −
2 2
√ √
6 6
⇔x=0 or x= or x = − .
2 2
Ç √ å √ Ç √ å Ç √ å √ Ç√ å
6 6 6 6 6 6
−∞, − − − ,0 0 0, ,∞
2 2 2 2 2 2

10x − − − 0 + + +
√ √
2x − 3 − − − − − 0 +
√ √
2x + 3 − 0 + + + + +

f 00 (x) − 0 + 0 − 0 +

Concavity CD NO CU NO CD NO CU

Ç √ å Ç√ å
6 6
f is concave up on − , 0 and ,∞ .
2 2
134 1st Semester Calculus Lecture Manual 2017 MATH1036

Ç √ å Ç √ å
6 6
f is concave down on −∞, − and 0, .
2 2
√ √ Ç √ √ å Ç√ √ å
6 6 6 21 6 6 21 6
Since the concavity changes around x = 0, − and , the points (0, 0), − , and ,−
2 2 2 8 2 8
are points of inflection.
8. Sketch:

√ √ y
(− 3, 6 3)


6
2
√ √ √ x
− 5 6 5

2

√ √
( 3, −6 3)

5 2
Worked Example 6.5.9 (Vertical tangent). Sketch the graph of f (x) = x 3 + 5x 3 .
1. Domain: dom( f ) = R.
2. Intercepts:
x-intercepts:

5 2
y = 0 ⇔ x 3 + 5x 3 = 0
2
⇔ x 3 (x + 5) = 0
⇔x=0 or x = −5.

y-intercept: x = 0 ⇒ y = 0.
3. Symmetry: None
4. Asymptotes: None.
5. Regions of increase and decrease:

5 2 10 − 1 5 − 1 5(x + 2)
f 0 (x) = x3 + x 3 = x 3 (x + 2) = 1 .
3 3 3 3x 3

The critical values occur when

f 0 (x) = 0 ⇔ x + 2 = 0 ⇔ x = −2 and
1
f 0 (x) does not exist ⇔ x 3 = 0 ⇔ x = 0.

Hence f has two critical numbers 0 and −2.


The 1st derivative table is
MATH1036 1st Semester Calculus Lecture Manual 2017 135

(−∞, −2) −2 (−2, 0) 0 (0, ∞)

5(x + 2) − 0 + + +
1
3x 3 − − − 0 +

f 0 (x) + 0 − U +

Slope % → & U %

INC NO DEC U INC

Hence f increases on (−∞, −2) and (0, ∞) whilst f decreases on (−2, 0).
6. Local extrema: The 1st Derivative Test and the table above shows a local maximum at x = −2 since the sign
of f 0 around x = −2 changes from positive to negative. Since

2 2 √3
f (−2) = (−2) 3 (−2 + 5) = 3(2 3 ) = 3 4

√3
so that (−2, 3 4) is a local maximum.
1
Furthermore, as x → 0, the numerator of f 0 , 5(x + 2), tends to 10, whilst the denominator 3x 3 tends to 0 so that

5(x + 2)
lim | f 0 (x)| = lim 1 = ∞.
x→0 x→0 3x 3

Thus at x = 0 we have a vertical tangent, i. e., at x = 0 we have a cusp which is a local minimum.
5 2 − 31
7. Concavity and Points of Inflection: Since f 0 (x) = x 3 + 10 3 x ,
3

10 − 1 10 − 4
f 00 (x) = x 3− x 3
9 9
10(x − 1)
= 4 .
9x 3

4
Thus f 00 (x) = 0 ⇔ x − 1 = 0 ⇔ x = 1, and f 00 (x) does not exist when x 3 = 0 ⇔ x = 0.

(−∞, 0) 0 (0, 1) 1 (1, ∞)

10(x − 1) − − − 0 +
4
9x 3 + 0 + + +

10(x − 1)
f 00 (x) = 4 − U − 0 +
9x 3
Concavity CD U CD NO CU

f is concave down on (−∞, 0) and (0, 1) whilst concave up on (1, ∞). Since the concavity changes around x = 1,
we have a point of inflection at x = 1. Thus the point (1, 6) is a point of inflection.
136 1st Semester Calculus Lecture Manual 2017 MATH1036

√3
(−2, 3 4) 6

−5
x

5 − x2
Worked Example 6.5.10 (Vertical and Horizontal Asymptotes). Sketch the graph of the function f (x) = .
1 − x2
Solution. Let y = f (x).
1. Domain: dom( f ) = {x ∈ R : 1 − x2 , 0} = {x ∈ R : x , ±1}.
2. Intercepts:
x-intercepts: Let y = 0. Then
5 − x2
y=0⇔
1 − x2
⇔ 5 − x2 = 0

⇔ x = ± 5.
y-intercept: Let x = 0. Then y = 5.
3. Symmetry: Since
5 − (−x)2 5 − x2
f (−x) = = = f (x),
1 − (−x)2 1 − x2
f is an even function, hence symmetric about the y-axis.
4. Asymptotes: Note that x = ±1 are not in the domain of f .
Vertical Asymptotes: As x → 1− , 5 − x2 → 4 and 1 − x2 → 0+ . Hence lim− f (x) = ∞. As x → 1+ , 5 − x2 → 4 and
x→1
1 − x2 → 0− . Hence lim+ f (x) = −∞. Hence x = 1 is a vertical asymptote.
x→1
As x → −1− , 5 − x2 → 4 and 1 − x2 → 0− . Hence lim− f (x) = −∞. As x → −1+ , 5 − x2 → 4 and 1 − x2 → 0+ .
x→1
Hence lim+ f (x) = ∞. Hence x = −1 is a vertical asymptote.
x→1
Horizontal Asymptote:
5 − x2
lim f (x) = lim
x→∞ x→∞ 1 − x2
5
2
−1
= lim x
x→∞ 1
2
−1
Åx ã
5
lim − 1
x→∞ x2
= Å ã
1
lim −1
x→∞ x2
−1
=
−1
= 1.
Similarly, lim f (x) = 1. Hence y = 1 is a horizontal asymptote.
x→−∞
5. Regions of increase and decrease: We find the critical points:
4 8x
f (x) = 1 + 2
⇒ f 0 (x) = .
1−x (1 − x2 )2
MATH1036 1st Semester Calculus Lecture Manual 2017 137

Since f is differentiable on its domain, the critical points occur when f 0 (x) = 0, i. e., when x = 0.
Observing that −1 and 1 do not belong to the domain of f , the 1st Derivative Table is

(−∞, −1) (−1, 0) 0 (0, 1) (1, ∞)

8x − − 0 + +

(1 − x2 )2 + + + + +

8x
f 0 (x) = − − 0 + +
(1 − x2 )2

Slope & & → % %


DEC DEC MIN INC INC

Intervals of increase: (0, 1) and (1, ∞).


Intervals of decrease: (−∞, −1) and (−1, 0).
6. Local extrema: Using the 1st Derivative Test and the table above we see that f 0 changes sign from negative to
positive around x = 0. Thus we have a local minimum at (0, 5).
8x
7. Concavity and Points of Inflection: Since f 0 (x) = ,
(1 − x2 )2

8(1 − x2 )2 − (8x)2(1 − x2 )(−2x) 8(1 + 3x2 )


f 00 (x) = = .
(1 − x2 )4 (1 − x2 )3

Thus f 00 (x) , 0 for all x ∈ dom( f ).


The 2nd Derivative Table is

(−∞, −1) (−1, 1) (1, ∞)

8(1 + 3x2 ) + + +

(1 − x2 )3 − + −

8(1 + 3x2 )
f 00 (x) = − + −
(1 − x2 )3

Concavity CD CU CD

Thus f is concave up on (−1, 1) whilst f is concave down on (−∞, −1) and (1, ∞).
f has no points of inflection since f 00 does not change concavity at any point in the domain of f . Note that even
though the concavity changes around ±1 we do not have points of inflection here since ±1 < dom( f ).
138 1st Semester Calculus Lecture Manual 2017 MATH1036

8. Sketch:

1
y=1

√ √ x
− 5 −1 1 5

x = −1 x=1

x2 − 9
Worked Example 6.5.11 (Slant Aysmptote). Sketch the graph of g(x) = .
2x − 4
Solution.
1. Domain: dom(g) = {x ∈ R : 2x − 4 , 0} = {x ∈ R : x , 2}.
2. Intercepts:
x2 − 9
x-intercepts: g(x) = 0 ⇔ = 0 ⇔ x2 − 9 = 0 ⇔ x = ±3.
2x − 4
9
y-intercept: x = 0 ⇒ g(x) = .
4
3. Symmetry: None.
4. Asymptotes:
Vertical Asymptote: Note that 2 < dom(g). As x → 2− , x2 − 9 → −5 and 2x − 4 → 0− , so that lim− g(x) = ∞.
x→2
Also, as x → 2+ , x2 − 9 → −5 and 2x − 4 → 0+ , so that lim+ g(x) = −∞.
x→2
Slant Asymptote: Note that the degree of the numerator is one more that that of the denominator. Long Division
produces
x2 − 9 1 (2x2 − 4x) + (4x − 8) − 10 1 5
g(x) = = = x+1− .
2x − 4 2 2x − 4 2 2x − 4
Therefore y = 12 x + 1 is a slant asymptote.
MATH1036 1st Semester Calculus Lecture Manual 2017 139

5. Regions of Increase and decrease: From


1 5(−2) 1 5
g0 (x) = − = +
2 (2x − 4)2 2 2(x − 2)2
we see that g0 (x) > 0 for all x ∈ dom(g). Since dom(g) = (−∞, 2) ∪ (2, ∞), g increases on (−∞, 2) and (2, ∞).
6. Local Extrema: Since the sign of g0 does not change in dom(g), g has no local extrema.
7. Concavity and Points of Inflection:
1 1 5 1 5(−2)(1) 5
g0 (x) = + ⇒ g00 (x) = =−
2 2 (x − 2)2 2 (x − 2)3 (x − 2)3
Hence g00 (x) exists for all x ∈ dom(g). For x < 2, g00 (x) > 0, so that g is concave up on (−∞, 2). For x > 2,
g00 (x) < 0, so that g is concave down on (2, ∞). There are no points of inflection (since 2 < dom(g)).
8. Sketch:

1
y= x+1
2

9
4

1
−3
x
−2 2 3

x=2

Tutorial 6.5.2. Thomas Exercises 11th ed. 4.4 p. 299: 21, 25, 27, 29, 30, 33, 39, 41, 42 or 12th ed. 4.4 p.
252–253: 23, 35, 37, 41, 49, 53, 55, 58, 85, 87, 88, 97.
140 1st Semester Calculus Lecture Manual 2017 MATH1036

6.6 Optimization

LEARNING OUTCOMES:
On completion of this section you should (tick the checkbox when you have mastered the skill)
 1. be able to identify and know what is meant by an optimization problem,
 2. be able to set up the function that has to be maximized or minimized in a given word problem about
optimization,
 3. be able to find the absolute extreme values of the optimization function,
 4. be able to solve a given optimization problem,
 5. know how to solve the examinable tutorial problems and the worked out problems from these notes on
this section,

Strategy for Optimization:

1. Carefully read and understand the problem.


2. Draw a diagram, where relevant, and introduce notation on the diagram assigning symbols to all
given and required quantities.
3. Assign a symbol (the optimization function) to the quantity that is to be maximized or minimized.

4. Express the optimization function in terms of one variable, using the geometry of the situation.
5. Find the critical values of the optimization function.
6. Determine the absolute maximum or minimum value of the optimization function.

7. State the final answer, in coherent form, specifying the units that you are using.

Worked Example 6.6.1. You need to fence in a rectangular vegetable patch so that it fits into a right-triangular
plot whose sides measure 4m and 12m. What is the maximum area for the vegetable patch?
Solution. Let x be the length of the rectangle and y be the width of the rectangle.

4 x R
Q

S T
12

Then the area of the rectangle is A = xy. We need A as a function of one variable. We use similar triangles. Since
∆PQR ∼ ∆PS T ,
4−y 4
= ,
x 12
which gives 12(4 − y) = 4x, i. e., x = 3(4 − y). Hence
A = xy = 3(4 − y)y = 12y − 3y2 .
MATH1036 1st Semester Calculus Lecture Manual 2017 141

dA
The critical numbers of A occur only where = 0. Thus
dy

dA
= 0 ⇔ 12 − 6y = 0.
dy

Hence y = 2 is the only critical number.


Since the domain of A is 0 ≤ y ≤ 4, we compare A(0), A(2) and A(4) to find the maximum area using the Closed
Interval Method. Now

A(0) = 0,
A(2) = 12(2) − 3(2)2 = 12,
A(4) = 3(4 − 4)4 = 0.

Thus the maximum area occurs when y = 2. In this case, x = 3(4 − y) = 3(4 − 2) = 6. Hence the largest rectangle
has length 6m and width 2m with area 12m2 .

Worked Example 6.6.2. Two posts, one 6m high and the other 9m high, stand 20m apart. They are to be stayed
by two wires, attached to a single stake, running from the ground level to the top of each post. Where should the
stake be placed to use the least wire?
Solution. 1. Hard Way: Let x be the distance from the 6m post to the stake. Also let y be the distance (representing
the length of wire) from the top of the 6m post to the stake, and z be the distance (representing the length of wire)
from the top of the 9m post to the stake.

z
9
y
6

x 20 − x
20

We need to minimize the length of wire. Let the length of wire be L. Then L = y + z. By Pythagoras,


x2 + 36 = y2 ⇒ y = x2 + 36,

(20 − x)2 + 92 = z2 ⇒ z = x2 − 40x + 481.

Thus the optimization function is

√ √
L=y+z= x2 + 36 + x2 − 40x + 481
142 1st Semester Calculus Lecture Manual 2017 MATH1036

with 0 ≤ x ≤ 20. For the critical values of L,

dL 2x 2x − 40
=0⇔ √ + √ =0
dx 2 x + 36 2 x − 40 + 481
2 2

x x − 20
⇔ √ + √ =0
x2 + 36 x2 − 40x + 481
√ √
⇔ x x2 − 40x + 481 = (20 − x) x2 + 36
⇒ x2 (x2 − 40x + 481) = (20 − x)2 (x2 + 36)
⇔ x4 − 40x3 + 481x2 = x4 − 40x3 + 436x2 − 40x(36) + 400(36)
⇔ 45x2 + 40(36)x − 400(36) = 0
⇔ x2 + 32x − 320 = 0
⇔ (x − 8)(x + 40) = 0
⇔x=8 or x = −40
∴x=8 ∵ 0 ≤ x ≤ 20

We now compare L(0), L(8) and L(20). Since


√ √
L(0) = 36 +
481 > 6 + 21 = 27,
√ √
L(8) = 64 + 36 + 144 + 81 = 10 + 15 = 25,
√ √
L(20) = 436 + 81 > 20 + 9 = 29,

the minimum length of wire required is 25m and the stake should be placed at 8m from the 6m post.
2. Smart Way: Dig a hole and bury one of the posts.

b
9
a
6

a0
6

Clearly, a = a0 , and therefore L = a + b has a minumum if a0 + b = l (reflection principle). But


p
l= 202 + (9 − (−6))2 = 25,

and x is found as the x-intercept of the line

9 − (−6) 3
y = −6 + x = −6 + x.
20 4
MATH1036 1st Semester Calculus Lecture Manual 2017 143

Worked Example 6.6.3. Four meters of wire is to be used to form a square and a circle. How much of the wire
should be used for the square and how much should be used for the circle to enclose the maximum total area?
Solution. Let x be the length of a side of the square and r be the radius of the circle. The total area is A = x2 + πr2 .
Since the total wire available is 4 meters, the total perimeter of the circle together with the square is 4. Thus

4 = (perimeter of square) + (perimeter of circle)


= 4x + 2πr
2(1 − x)
⇒r= .
π
The optimization function is
ï ò2
2(1 − x) 4 1
A = x + πr = x + π
2 2 2
= x2 + (1 − x)2 = (π + 4)x2 − 8x + 4

π π π

restricted by the square’s perimeter, i. e., 0 ≤ 4x ≤ 4, so that 0 ≤ x ≤ 1. For the critical values of A(x) we find

1
A0 (x) = 0 ⇒ [2(π + 4)x − 8] = 0
π
8 4
⇒x= = .
2(π + 4) π + 4

Since
4
A(0) = ,
π
1 4(π + 4) − 16
Å ã ï ò ï ò
4 1 16 32 4
A = − +4 = = ,
π+4 π π+4 π+4 π π+4 π+4
A(1) = 1,

4 2
the maximal area is m which occurs when x = 0, i. e., all the wire is used for the circle.
π
Worked Example 6.6.4. A manufacturer wishes to make a closed box with a square base and rectangular sides.
The base of the box and all four sides are to be made of metal costing R10/m2 . The lid is to be constructed of a
thick glass costing R50/m2 . What are the dimensions of the box of greatest volume that can be constructed for
R720?
Solution. Let b be the length of the base, h the height of the side, C the cost of the box and V the volume of the
box. The objective is to maximize the volume V = b2 h given that the cost is C = 720. Now

C = 50(area of lid) + 4(10)(area of vertical side) + 10(area of base)


= 50b2 + 40bh + 10b2
= 60b2 + 40bh
⇒ 720 = 60b2 + 40bh
⇒ 40bh = 720 − 60b2
720 − 60b2
⇒h=
40b
36 − 3b2
= .
2b
Then V in terms of one variable is

36 − 3b2
Å ã
1
V=b h=b
2 2
= (36b − 3b3 ).
2b 2
144 1st Semester Calculus Lecture Manual 2017 MATH1036

1
Since b ≥ 0, the domain of V is D = {0 ≤ b ∈ R : (36b − 3b3 ) ≥ 0}. Now
2
1
(36b − 3b3 ) ≥ 0 ⇔ 36b − 3b3 ≥ 0
2
⇔ 3b(12 − b2 ) ≥ 0
⇔ 12 − b2 ≥ 0 ∵b≥0
2
⇔ b ≤ 12

⇔ b ≤ 12 ∵b≥0

Hence the domain of V is D = [0, 12]. For the maximal volume,
dV 1
= 0 ⇒ (36 − 9b2 ) = 0 ⇒ b2 = 4 ⇒ b = 2 or b = −2 ⇒ b = 2 ∵ b ≥ 0.
db 2

We compare V(0), V(2) and V( 12) to determine the maximal volume.

V(0) = 0
1 1
V(2) = (36(2) − 3(2)3 ) = (72 − 24) = 24,
2 2

V( 12) = 0.

36 − 3b2 36 − 12
Thus b = 2 gives the maximal volume. Then h = = = 6. Thus b = 2m and h = 6m gives the
2b 4
maximal volume of 24m3 .

Worked Example 6.6.5. A right circular cylinder is inscribed in a right circular cone so that the centrelines of the
cylinder and the cone coincide. The cone has height 6m and radius of base 3m.
(a) Express the volume of the cylinder as a function of its radius.
(b) Find the dimensions and the volume of the cylinder with maximal volume.
Solution. Let r be the radius and h the height of the cylinder.

6 6

r 3−r
3
3

(a) The volume of the right cylinder is V = πr2 h. Using similar triangles, we see that
h 6
= = 2 ⇒ h = 6 − 2r.
3−r 3
MATH1036 1st Semester Calculus Lecture Manual 2017 145

Hence,
V = πr2 (6 − 2r) = 2π(3r2 − r3 ).
Since the radius of the cylinder cannot exceed the base of the cone we have

V(r) = 2π(3r2 − r3 ), 0 ≤ r ≤ 3.

(b) For the critical values of V,

V 0 (r) = 0 ⇔ 2π(6r − 3r2 ) = 0


⇒ r(2 − r) = 0
⇔ r = 0 or r = 2.

Comparing V(0), V(2) and V(3) we get

V(0) = 0,
V(2) = 2π(3(22 ) − 23 ) = 8π,
V(3) = 0.

Thus the maximal volume of the cylinder occurs when r = 2. In this case h = 6 − 2(2) = 2. Hence, the cylinder of
maximal volume has height 2 m and radius 2 m with volume 8π m3 .

Tutorial 6.6.1. 1. Thomas Exercises 11th ed. 4.5 pp. 309–315: 4, 7, 8, 11, 16, 18, 19, 22, 24, 33, 43, 44, 52a,
59a or 12th ed. 4.6 pp. 268–273: 4, 7, 8, 11, 16, 18, 19, 22, 24, 39, 51, 52, 60a, 67a.
2. Thomas Chapter 4 Additional and Advanced Exercises 11th ed. pp. 348–350: 15, 17, 18, 26 or 12th ed. pp.
294–295: 15, 17, 18, 26.

Sn=
Chapter 7

Hyperbolic Functions

This chapter is a self-study. You will not be given any formal lectures on this examinable chapter. The reference
for the section on hyperbolic functions is Thomas, Section 7.4, pages 523–534.

LEARNING OUTCOMES:
On completion of this section you should (tick the checkbox when you have mastered the skill)
 1. know the definitions of the hyperbolic functions sinh x, cosh x, tanh x, sech x, coth x and cosech x and
know the sketches of the graphs of sinh x, cosh x and tanh x given on page 524 of Thomas,
 2. know the hyperbolic identities and their derivations given on page 525 of Thomas,
 3. know the derivatives (and the proofs) of the hyperbolic functions given on pages 525–526 of Thomas,
 4. know the inverse hyperbolic functions sinh x, cosh x and tanh x and their sketches given on page
−1 −1 −1

527 of Thomas,
 5. know how to express the inverse√hyperbolic functions in terms√ of natural logarithms, i. e.,Åknow how
ã to
1 1+x
prove and use sinh−1 x = ln(x + x2 + 1), cosh−1 x = ln(x + x2 − 1) and tanh−1 x = ln ,
2 1−x
 6. know the derivatives (and the proofs) of the inverse hyperbolic functions given on pages 528–529 of
Thomas and know how to use the inverse hyperbolic functions in integration techniques as given on page
530 of Thomas,
 7. know how to solve the examinable tutorial problems and the worked out problems from Thomas on this
section.

Tutorial 7.0.2. 1. Thomas Exercises 11th ed. 7.4, pp. 530–531: 1, 3, 5, 6, 7, 9–12, 14, 15, 17, 21, 23, 25, 27, 31,
37, 40, 41, 45, 55, 61, 62 or 12th ed. 7.3, pp. 441–442: 1, 3, 5, 6, 7, 9–12, 14, 15, 17, 21, 23, 25, 27, 31, 37, 40,
41, 45, 55, 61, 62.
2. Prove the following formulas:

(a) sinh−1 x = ln(x + x2 + 1),

(b) cosh−1 x = ln(x + x2 − 1),
1+x
Å ã
1
(c) tanh−1 x = ln .
2 1−x

146

You might also like